Anda di halaman 1dari 265

LAW DECKS

COMPREHENSIVE FLASHCARDS TO HELP YOU PREPARE FOR THE MULTISTATE BAR EXAMINATION.

LAW DECKS
TOPICS INCLUDE:

Constitutional Law
Contracts CRIMINAL LAW Property Law Torts Evidence

Thank you for purchasing Law Decks. We feel that the use of Law Decks as a supplement to a study program for the Multistate Bar Exam will enable you to have foundation for excellent exam performance. All rights reserved. No part of these cards may be reproduced in any way, or by any means, without permission from Law Decks, Inc. The purchaser agrees not to resell, copy, rent, or transfer any part of this product. This product has been prepared from sources the author believes to be accurate and reliable. However, the possibility of human and/or mechanical error does exist. The users of these cards are encouraged to secure additional information, as needed, to assist them in reaching excellent performance. If you have any questions, comments, or recommendations about Law Decks, please contact us.

Law Decks, Inc.


"The Decks Corporation" 4065 Quakerbridge Road Princeton Junction, NJ 08550 (609) 919-9400 (877)-763-3257 www.LawDecks.com
GOOD LUCK ON YOUR EXAM!

NOTICE
The authors and the publisher of this volume have taken care that the information and recommendations contained herein are accurate and compatible with the standards generally accepted at the time of publication. However, as in any text, some inaccuracies and ambiguities may occur; therefore, if in doubt, please consult your references. The authors and the publisher disclaim any liability, loss, or damage incurred as a consequence, directly or indirectly, of the use and application of any of the contents of this volume.

AUTHORS
Vincent P. Loccisano, JD Based on years of analyzing and forecasting the topic coverage, fact patterns and question formats covered on the MBE, as well as the Decks Corp. approach to flashcards, Loccisano has developed this series of flashcards you have before you. Using this systematical and scientific approach to studying, he has passed numerous professional licensing exams, including the Massachusetts Bar Exam and the Federal Patent Bar Exam, all on the first try. Mr. Loccisano currently practices patent law in Boston, Massachusetts. Loccisano holds a J.D. from the Intellecutal Property Law Concentration program of Suffolk University School of Law and a B.S. in Mechanical Engineering from Lehigh University. Candace Lombardi, MFA Lombardi is responsible for the creative attempts at humor, satire and diversity in your questions. She holds an M.F.A. in Creative Writing from Emerson College and a B.A. in English from Lehigh University. When she isn't thinking up creative ways to grasp your attention, Lombardi is a freelance travel writer, member of the Davis Square Writers Group, screenwriter, and children's book author.

Property Law
Che and Cedric own adjoining pieces of land in a rural community, but the boundary line between the 2 properties was never clearly marked. Twenty-five years ago Che dug a well on a piece of the property that he believed to be his. He has used the well continuously since then and has taken care of the area surrounding it. Fifteen years ago, Cedric was defined as mentally ill and found incompetent. He died today and the executor of his estate filed suit to throw Che off the land with the well. The adverse possession requirement is 20 years. Which best summarizes the status of the well property? A. Che has acquired title by adverse possession. B. Che cannot claim title as an adverse possessor since he never entered with hostile intent. C. Che is in adverse possession but does not hold title, since Cedric's incompetence 15 year ago stopped the possession clock from running. D. None of the above

Answer: A
This is a simple adverse possession case. Che had title due to his open, exclusive, continuous, and hostile use for a period longer then the statute. Since Cedric did nothing to eject Che during the running period of the statute, title to the property vests in the possessor, and the owner is barred from suing for ejectment. An adverse possession claim requires: 1) open and notorious use, 2) actual and exclusive use, 3) ccotinuatis use throughout the statutory period and 4) the possessor must occupy the property and enter without the owner's permission. Here, Che has met all of these requirements. B is incorrect. Hostile intent merely requires a possessor to take possession of the land without permission of the true initial owner, which results in a loss of Cedric's rights. C is incorrect. The only time in which the statute of limitations for adverse possession would be tolled is if the true initial owner is mentally incompetent when the possessor initially takes the land. Here, Cedric's mental illness occurred years after Che initially took possession. D is incorrect in light of the correct answer. Answer A clearly and accurately states that Che has taken right and title by adverse possession.

2007-2008 Law Decks

Fropertg Law
Kazuo owns a huge hunk of property in a rural district of Oregon. He builds a home on the northernmost region, and grows Christmas trees on the southern region. Kazuo becomes ill from pesticide poisoning and spends his last years in a hospital bed. Right before dying, he gives the northern parcel with his house to Henry stating: "To Henry, for life and then to his widow for her life, and then to Henry's surviving children." The southern region goes to Roy, a friend. Kazuo dies leaving a single heir, his brother Tanaka. After failing to rent the house on the northernmost region of the property, Henry decides to level it and plant Christmas trees for income. The property would be worth much more this way, but Tanaka has a sentimental attachment to the home and wants Henry to leave it as is. This jurisdiction has the common law rule against perpetuities. Does Tanaka have standing to enjoin destruction of the house? A. Yes, as Tanaka has a reversion that will occur when Henry's widow dies. B. Yes, Tanaka has an executory interest that will become possessory in the event that

Henry dies without surviving children.


C. No, Tanaka has no interest in this parcel of the property. D. No, so long as Henry is married with living children at the time of the conveyance.

Answer: A
Under the rule against perpetuities, the remainder of Henry's interest is void and Tanaka has a reversionary interest. Since Tanaka has inherited this reversionary interest from Henry, he has standing to sue. Under the rule against perpetuities, interest in property is valid unless it must vest, if at all, for longer than 21 years after one or more of the lives in being at the creation of the interest. The analysis for this rule is done at the time the interest is created. We have a potential problem with the general word "widow." "Widow" is defined as the person to whom someone is married at the time of his death, so Henry's widow can only be determined upon Henry's death. Henry's "widow" has a contingent remainder in a life estate because, until her identity and existence are ascertained, there is no one to take possession upon Henry's death. Henry's children have a contingent remainder in fee simple, as their interest is contingent upon outliving both Henry and his widow. This remainder is in violation of the rule against perpetuities, as Henry may get divorced (assuming that he is married) and remarry someone younger who was not yet born at the time that the interest was created. He could then have a new child with his new wife, at which time everyone connected with the vesting of the interest could die. This widow could live for more than 21 years after the death of all lives in being at the time the interest was created and leave their child. Here, the interest of Henry's widow and child could vest outside the perpetuities period (21 years) rendering the interest void. In light of this, when Kazuo convened the northern parcel of his land, he had a reversion, which was passed to his brother Tanaka upon his death. Tanaka, therefore, has standing to enjoin the destruction of house. B is incorrect since Tanaka actually holds a reversion interest, and not an executory interest. By definition, an executory interest is a future interest created in a transferee that is not capable of taking on the natural termination of a preceding life estate. An executory interest follows a gap in possession which does not occur here. In the present

facts, Tanaka gets his interest in the property through his inheritance rights from his brother Kazuo.

C incorrectly states that Tanaka has no interest in the northern parcel. In light of the correct answer, Tanaka has a reversionary interest and can sue to enjoin destruction of the house. D is incorrect. Based upon the analysis given in the correct answer, there is a possibility that the Rule of Perpetuities could be violated.
2007-2008 Law Decks

Fropert9 Law
Bartlett owns an apartment building in New Hampshire and decides to execute a deed to his son that states: "To my son, Franklin, for his life, and upon his death to his children, except if Franklin becomes a Republican; then I leave it to my daughter, Mary, and then to Mary's children upon her death." At the time of the grant, Bartlett is alive and healthy and a Democrat. Assume that Franklin becomes a Republican and Mary takes the property. How would the interest in Mary's 2 living children best be described? A. A contingent remainder B. A vested remainder subject to total divestment C. An executory interest D. A vested remainder subject to open

Answer: D
Here, Mary has a possessory interest in the property for her life; and then she sees that the property passes to her children. Her children's interest is a remainder as it comes into being once Mary dies. It is a vested remainder as it has no condition precedent that it is subject to and the takers are known. The remainder is subject to open as Mary may have additional children and they would be eligible to take from the interest. A is wrong. This is not a contingent remainder as there is no condition precedent and the takers (child 1 and child 2) are known. B is wrong as an interest subject to total divestment means that child 1 and child 2 may lose their interest. This is not the case. At worst their interest may be reduced upon Mary having more children. C is wrong. This is not an executory interest since it follows Mary's life estate rather than divests the previous estate. Note: If Mary has not yet taken possession (i.e., Franklin remains a Democrat) then this

would be an executory interest since it would not naturally follow from Franklin's life estate, but would divest only if Franklin changed political affiliation.

2007-2008 Law Decks

Froperty Law
Aarti owns 10,000 acres. She wishes to develop 5,000 of those acres for a housing development. Aarti intends to hold the remaining acres for a long time, hoping that they will become more valuable once the housing development is completed. While selling the houses, Aarti markets the remaining acres to the current potential buyers as a wildlife refuge, where they can hike, mountain bike and ride mopeds; and is okay with doing this so long as she can sell the remaining 5,000 acres in 20 years. What is the best legal remedy Aarti can use to implement her plan? A. A covenant B. An easement C. A leasehold D. A personal contractual obligation of owner

Answer: B
The best way Aarti can do this is via a 20 year easement, wherein the holder can use the land for a special purpose, but has no right to possess or enjoy the land. Upon granting an easement, Aarti can limit its scope (as she chooses) to the home buyers in the 5,000 acre development. At the end of the 20 years, the homeowners loose all rights and Aarti can do with the land as she pleases. A is incorrect. A covenant is a promise to do, or refrain from doing, something on one's own property that one would otherwise be privileged to do. Here, a covenant would not grant the homeowners a right to use Aarti's land. C is incorrect. A leasehold is a possessory interest in land and in granting the lease Aarti could, in fact, be giving up her right to possess the land. Using the easement approach, Aarti retains the right and ability to essentially do with the property as she pleases, while retaining the most control possible.

death all rights of the homeowners would be lost. The same loss of rights would occur if Aarti

D is incorrect. A personal contract with each homeowner is a poor choice since upon Aarti's

decided to sell. In the case of a personal contract, the homeowners might be in a position to acquire payment for damages; but they still could not successfully sue for continued use of the land. In an easement, however, the homeowners' continual use of the land for the 20 year period would be protected in the event of Aarti's death or sale of the land.
2007-2008 Law Decks

Property Law
Suppose that Ramirez owns a farm in Utah and sells it to Johnson. Johnson pays a fair price based on the market value but fails to record the deed before leaving for his honeymoon. Ramirez is then hit by a runaway tractor and dies. Her will is probated and it reads: "I leave the farm to my daughter Sandra." As Sandra knows nothing about the sale to Johnson, she records the deed the following day. She decides to start raising sheep, and (needing cash) goes to WaterBank who give her a loan for $100,000 with the farm as collateral. WaterBank checks and sees that Sandra is the only owner of the farm on record. The sheep farming fails and WaterBank requires that the farm is sold in foreclosure, the proceeds of which will repay Sandra's bank loan. Just before the sale, Johnson returns with his executed deed. Suppose that there is a Utah statute that reads: "No conveyance or mortgage of an interest in land is valid against any subsequent purchaser for value without notice thereof, unless it is recorded." In light of this, can WaterBank foreclose and take the proceeds? A. Yes, WaterBank is the successor to Sandra's rights.
B. Yes, because WaterBank is a mortgagee for value.

C. No, because Sandra never actually owned the land. D. No, since WaterBank should have known that Sandra's business would likely fail and should have never granted the loan in the first place.

Answer: B Here, we have a notice statute. Therefore, a purchaser for value without notice of the prior conveyance is protected by that state statute. In light of a notice statute like this, a subsequent bona fide purchaser will prevail over a prior purchaser who failed to record his purchase. Note: A bank, as a mortgagor, is considered a purchaser over a prior buyer who failed to record their deed. Here, WaterBank had no notice of the prior sale, making them a proper purchaser whose rights are protected. A is incorrect. Sandra's rights would not protect WaterBank as Sandra was not a bona fide purchaser, since she failed to pay value for the farm. C is wrong. Johnson could have challenged Sandra's rights since she was not a bona fide purchaser. However, WaterBank is a bona fide purchaser who properly recorded the deed. WaterBank stepping in trumps Johnson's rights due to his failure to record. D is wrong for offering an irrelevant answer to the question and facts presented. In light of the facts, the assumption that the loan may have been imprudent is meaningless. All that matters, here, is that WaterBank was a bona fide purchaser who properly recorded the deed.
2007-2008 Law Decks

Fropert9 Law
Enrique and Wendy are a married couple who own Casa Riverside as a tenancy by the entirety. Enrique decides to transfer his interest to Paul by a quitclaim deed. When Enrique does this, Wendy is away visiting her mother. When she gets back, she decides to mortgage her interest in Casa Riverside to Sue. What interest does Paul have? A. No interest B. An undivided % interest with right of survivorship C. An undivided % interest held in tenancy in common D. The entire fee

Answer: A
Paul has no interest, since Enrique and Wendy held the property as a tenancy by the entirety, which is only applicable to married couples. This tenancy can only be terminated upon the death of a spouse, divorce of the spouses, mutual agreement, or execution by a joint creditor of both spouses. In light of this, Enrique had no interest that he alone could have convened to Paul. B is wrong as Enrique had no interest he alone could have transferred. This is a tenancy by the entirety and the property is essentially held by the couple as one. C is wrong. Enrique acting alone cannot sever the tenancy by the entirety and turn it into a tenancy in common. D is wrong for all of the above reasons, and, additionally, because Enrique did not own the entire fee himself and, therefore, could not convey the entire fee himself.

2007-2008 Law Decks

Fropertg Law
Serena owns a baseball field. Bette owns a bumper car park which adjoins the baseball field and is surrounded on 3 sides by quicksand. Bette has an easement to cross the baseball field for access. After 15 years, Bette sells the bumper car park to Serena, who uses the two lots as one common lot. After 10 years, Serena sells the bumper car park to Barry. Does Barry have an easement over the baseball field? A. Yes, an easement in gross. B. Yes, because his only access to the bumper car park is across the baseball field. C. No, Serena's purchase of the bumper car park from Bette extinguished the easement. D. No, Barry needs to use the property for a longer time to gain an easement by prescription.

Answer: B
There is an easement by necessity over Serena's baseball field, since this is the only way to access the bumper car park. Upon the sale of the bumper car park from Serena to Barry, there was an implied grant of an access easement since (given that quicksand surrounds the other 3 sides of the bumper car park) the baseball field was clearly the only access to a public road. A is wrong. An easement in gross does not have a tenant associated with it, but rather it gives the right to use to the servient tenant, regardless of ownership or possession of an adjoining piece of land by the easement holder. Here, we have an easement that is attached to the bumper car park purchased from Serena by Barry. Thus, the easement is appurtenant, not in gross. C is wrong. Upon sale Bette's easement was extinguished, but Barry acquired a new easement by necessity upon purchase. D is wrong. Barry gains an easement by necessity that results from a certain fact pattern (quicksand on 3 sides), not based on a time of possession. Here, the lack of access by any means but through Serena's baseball field, results in the creation of the easement.

2007-2008 Law Decks

Fropert9 Law
Jos6 inherits Little Quinta from Angel. Liz owns Giant Ranch, which adjoins Little Quinta. Liz tries drilling for natural gas on Giant Ranch, but finds nothing. She approaches Jos6 and asks for a grant to try drilling on his property, but Jos refuses. Liz then goes to the border between their properties and begins drilling at an angle that puts her under Little Quinta. She finds gas there. Jose then discovers that gas is coming from beneath his land. Can Jose sue Liz for damages? A. Yes, Liz's actions invaded Jose's subterranean rights. B. Yes, if Liz's drilling interferes with Jos's use and enjoyment. C. No, because gas is a free-flowing substance and can be captured wherever it flows. D. No, Liz's acts do not prevent Jos from drilling on Giant Ranch property himself.

Answer: A
Liz is committing a trespass, which is a tangible physical intrusion that interferes with the possessor's right to exclusive possession of the land. Damages can be awarded based on this trespass. Here, Jose is entitled to such damages as the drilling was on property for which he had exclusive possession. The trespass occurs regardless of whether Liz struck gas or not. B is incorrect for only addressing the nuisance grounds for recovery and neglecting the trespass grounds of recovery. A nuisance is an activity that substantially and unreasonably interferes with a possessor's use or enjoyment of their property. While Jose could possibly recover under a nuisance theory, he would have to show that his enjoyment was interfered with. A trespass theory recovery does not require such a showing. C is a true statement of law and would be applicable if Liz had drilled straight down into Giant Ranch, and the gas was flowing from Little Quinta to Giant Ranch where Liz was

pumping it out. Here, however, Liz has drilled a slanted well that necessarily goes under
Little Quinta in order to hit gas, making this a trespass. D is incorrect as being irrelevant. The fact that Jose's right to drill was interfered with is meaningless. Jose can recover for trespass regardless of the interference with is rights.
2007-2008 Law Decks

Fropert9 Law
Kirti buys a house from Barbara only to learn that it was built improperly with outdated wiring techniques requiring $10,000 to fix. Kirti sues Barbara for the cost to fix the place. Which fact, if true, would best help Kirti's chance of winning? A. The improper wiring makes the house unsafe or uninhabitable. B. Barbara was the builder. C. Kirti took the house by warranty deed. D. Kirti had no knowledge of the defect when she bought the house and it was not immediately and reasonably evident.

Answer: B
Generally speaking, the conveyance of property has no warranty associated with it when between ordinary parties. However, if Barbara was the builder, there is an implied warranty that the house was built in a "workmanlike" manner and suitable for human habitation. A is wrong. The fact that the house was unsafe and uninhabitable is meaningless unless (as set forth in the correct answer) Barbara was the actual builder. C is wrong. A warranty deed only contains covenants for title. It is not related to, or a protection for, defects in construction. D is wrong as knowledge is not a relevant fact here. In many states the acts of a seller who actively attempts to hide a defect can open the door to liability, but here there is no indication that Barbara acted in a dishonest manner at all. In light of this, the only way that Barbara can be liable for these deficiencies is if she is the actual builder (in light of

the implied warranty).

2007-2008 Law Decks

Property Law
Pasquale enters into an option agreement to buy Philip's house and land. Pasquale is to pay a fixed monthly rent, plus taxes and maintenance charges for 5 years. This option can be exercised at any point during that 5 year term by giving Philip 30 days notice. Three years into the lease, Philip dies, leaving his estate to Rick, his brother. Just before Philip's death, Pasquale assigned his interest in land to Ashley using a written agreement. The option terms are set forth, but Pasquale failed to mention the payment of the real estate taxes. Months later, Rick is informed that there is a tax lien on the house. Rick pays off the lien. Ashley goes on to assign the lease to Biff, who fails to pay the taxes or rent. Can Rick sue Ashley to recover? A. Yes, Ashley is still in privity of estate with Rick. B. Yes, because Ashley is still in privity of contract with Rick . C. No, Ashley is not in privity of estate with Rick. D. No, since Pasquale was allowed to assign the lease, Ashley is allowed to assign the lease.

Answer: C
liable for covenants in the original lease after they assign. Answer C, therefore, makes Answer A clearly incorrect.

Absent an express assumption, or language otherwise, an assignee does not remain

B is wrong because an assignee is not in privity of contract with the lessor unless the assignee expressly assumes the lease obligations. D is incorrect as the ability of Ashley to subsequently assign is unrelated to the issue of liability. Ashley's original liability comes about under the basis of privity of estate. Once Ashley assigns the lease to Biff; the privity of estate ends.

2007-2008 Law Decks

Property Law
Roman leases Metro Pond to Mitch for 99 years. Mitch builds a casino on Metro Pond, and sells the casino to Lorna, with Roman consenting to the assignment of the lease. Ten years into the lease, Roman conveys the property to his 2 sons, Harry and Lance. A month later, Roman has a huge fight with his sons and demands that they return the deed. The sons do so and Roman destroys the deed. Harry learns that he is dying and conveys to Tim "any interest I may have in Metro Pond." He then dies. After Harry's death, Roman sells Metro Pond to Reva. Assume that the deed from Roman to his sons was never recorded. Further assume that the state recording statute reads: "No conveyance or mortgage of an interest in land is valid against any subsequent purchaser for value without notice thereof whose conveyance is first recorded." If Reva were to bring a quiet title action and win, which answer explains the result? A. Roman, as owner, was allowed to convey to Reva. B. Harry and Lance never recorded the deed they took from Roman. C. Since Harry and Lance never recorded, Reva was not aware of their interest when she paid fair market value for the property. D. Harry and Lance never recorded. Reva paid fair market value and then immediately recorded.

Answer: D
Since Roman had no interest to convey, Answer A is wrong. If Reva was a bona fide purchaser for value who recorded first, then the deed to Harry and Lance would be no good against her. B and C are incorrect because they do not contain every element necessary for Reva to prevail.

2007-2008 Law Decks

Fropert3 Law
Jung owns land in fee simple. In 1990 she devises land to Ty for life with a written instrument that states: "To Ty for life; if Ed and Ned return home from looking for the Northwest Passage prior to Ty's death, to Ed and Ned and their heirs. If they do not return home before Ty dies then the remainder is to go to my heirs." In 2004 Jung and Ty both die in a tragic smoking accident. Ty's will left the estate to a pro-smoking club, and Ty's remaining relative is his brother, Jake. Assume for this question that Ed and Ned do find the Northwest Passage. Assume that Ty is dead and that Ed and Ned are also dead. Ed predeceased Ned by 2 months. Ned has a son, Scott. What is Scott's interest after the death of his father Ned? A. Ownership of all Jung's land in fee simple. B. Ownership of 1/2 of Jung's land in fee simple. C. Ownership of an undivided 1/2 interest in Jung's land along with the heirs of Ed who

own the same interest.


D. Ownership of 1/2 of Jung's land as a tenant in common with the heirs of Ed.

Answer: C
When Ed died, his % interest past to his heirs. The same occurred when Ned died, namely to Scott. In light of this, Scott owns % interest in Jung's land and Ed's heirs own 1 /2 interest in fee simple. Answer A is wrong because Scott only owns 1/2 of the land in fee simple, the remaining half is owned by Ed's heirs. B is wrong because Scott holds an undivided % interest in, namely a % interest in all of Jung's land. Scott does not own % of the land (a full interest in % of the land). Answer D is also wrong based on similar logic to Answer B.

2007-2008 Law Decks

Fropert9 Law
Luna partners with Lupe to develop Wolf Moon Estates in 1990. In 1999 they acquire all of the required permits and begin to build Wolf Moon's residential development. The first portion is comprised of 100 houses on 250 acres of land. The second portion is a restaurant complex built on an adjoining 50 acres of land. Lupe owns an adjoining piece of land (100 acres) on which Tom wants to build a condo complex. These condos would result in the blockage of the residents' of Wolf Moon Estates view of the mountains. What effect would the original Wolf Moon residential development have on Tom's proposed condo project? A. Tom, who has taken the property from Lupe, can only use the property for constructing single family ranch houses. B. The owners of Wolf Moon Estates homes have an implied easement for light and air against Tom, and can enjoin Tom's construction.

C. The owners of Wolf Moon Estates homes have a right of first refusal to buy the land
sold from Lupe to Tom. D. Tom can do as he pleases, as the owners of Wolf Moon Estates homes have no enforceable rights against Tom.

Answer: D
The original development plan has no common scheme regarding the land owned by Lupe, therefore, Tom can do with the land as he pleases. In light of this, Answers A and C are clearly incorrect as they are not supported by the facts. Answer B is further incorrect for being a blatant misstatement of the law.

2007-2008 Law Decks

Fropert9 Law
Felix builds Cara Mia Estates, and includes the following in each lot deed, "Grantee agrees for himself/herself and assigns to only use this property for a single family home construction, and to pay the monthly service charges to Cara Mia Estates for upkeep and security. Grantee also agrees not to put up any fences so that the indigenous cats may wonder the grounds as they please. Grantee will also agree never to paint a house yellow & brown since it is a bad combination." Felix sells lots to Minx, Luis, and Nan, and each deed is recorded. Minx resells to Mike, but this deed has no language noting the restrictions and Mike has no notice of them. Mike then starts selling junk scrap metal out of his garage and keeps it on the side of his house. Luis resells to Kendra, who is aware of the monthly fees but refuses to pay based on the suggestions of a ghost she claims to have met in her new house. Nan resells to Cindy who builds a 12 foot cinder block wall around her property and digs a mote, unaware of the fence restriction. Answer based on common law requirements. Assume that Kendra sues Cindy to remove the wall and mote. Who wins? A. Cindy wins, as there is no privity between Kendra and Cindy. B. Cindy wins, because she built the wall and mote in good faith and without knowledge of the restrictions. C. Kendra wins, because Kendra holds vertical probity with Luis. D. Kendra wins, since the deed language about fencing results in an equitable servitude, which is enforceable against Cindy.

Answer: D
Kendra wins, as we have an equitable servitude enforceable against Cindy. The deed language showed intent to allow the restriction to be enforceable by and against assignees. The burden of the covenant clearly touches and concerns the land occupied by Cindy, as it results in a restriction of what Cindy can, in fact, do with the land. The benefit of the covenant touches and concerns Kendra's property as it allows her cats to roam freely. Cindy further has constructive notice of the restriction based upon the recorded deed between Felix and Nan resulting in record notice. Cindy may additionally have been on inquiry notice due to the general makeup of the surrounding properties. As this was part of a general plan for the entire subdivision, a neighbor can enforce the deed restriction. We have all of the equitable servitude requirements, which can be enforced by Kendra against Cindy. B is incorrect since Cindy had record notice, and, possibly, inquiry notice as set forth in correct answer. A and C are both red herrings. We are dealing with an equitable servitude. Privity is not necessary because the servitude is enforced as an equitable property interest in the land itself, and not as an in personam right against the owner of the servient tenement.

2007-2008 Law Decks

Fropert9 Law
Edie and Simon own adjoining land. Edie's property is west of Simon's. Adjoining Simon's property on the other side to the east is Hot Spring Caves. In 1980 Edie buys inflatable rubber duckies for her children to use while soaking in the hot springs. She asks Simon if it would be okay for her and her children to walk across a 10 foot expanse of his property to gain access to the hot springs. Simon knows that the only other access is a circuitous 30 minute car ride over 15 miles, so he agrees to let Edie and her kids use the 10 feet of his property to get to the springs. Edie and her family use this strip for 19 years, at which point Simon sells his property to Art. Art sees Edie walking the strip every Monday, but says nothing, knowing that this is, in fact, his property. On the 9th month of noticing this, Art speaks up and prohibits Edie from using the strip. Edie and her family continue to use the land, regardless of the fight, and continue for 5 more years, at which point Art gets an injunction. The adverse possession period is 20 years. Assume that a court of law has determined that Edie holds a valid license for use of the 10 foot strip to Hot Spring Caves. Which answer best describes the easement? A. An easement in gross

B. An easement appurtenant C. An easement by necessity


D. An easement by implied reservation

Answer: B We have an easement appurtenant since the right to use the 10 foot strip results in a special benefit conferred upon Edie to enjoy a piece of land (here, her own tract) based upon the rights to special use. A is incorrect as an easement in gross is acquired independent of the ownership and enjoyment of another tract of land. An easement of necessity is not applicable for two reasons: Hot Spring Caves is unlikely to be found a public way that Edie needs access to; and Edie does have an alternative route that could gain her access to the hot springs if she so chooses. Therefore, Answer C is incorrect. D is incorrect. We have no facts indicating that Edie ever owned the entire parcel of land and conveyed a portion of it to Simon, while reserving an easement for herself with the conveyance.

2007-2008 Law Decks

Fropert9 Law
Suppose that Higuchi owns a 100 acre farm in Kansas City. Suppose that Kansas City annexes his land to build a residential shopping complex, and rezones it from farming to residential. This project is spearheaded by Hattie, a builder who buys the 100 acres from Higuchi on behalf of the city for a fair price. She files a subdivision plan to build 200 homes and 2 "city parks" and begins selling lots left and right. In each deed of transfer is a covenant, restrictions and conditions relating to the nature of homes in relation to the subdivision. One covenant is that there is to be no commercial or industrial activity. Hattie sells all but 10 lots and one of the designated parks. All of her sales include the covenants. The remaining 10 lots are then sold to CoCompany, without any of the deed restrictions. They resell the 10 lots (absent deed restrictions) and the remaining designated park to Clear & Free, a nuclear power company. The final sale to Clear & Free was made 2 years after Kansas City's purchase of land from Higuchi. Clear & Free is about to start trucking in uranium. Suppose that until now, Kansas City has done nothing with the park. Wilson, an owner of one of the lots purchased by CoCompany, is a holistic healer. Wilson puts a small sign in the front window advertising his services and starts seeing patients in a discrete manner. Wilson's actions are in compliance with local zoning rules. Assume that Chase, an owner of one of the original lots sold by Hattie seeks to enjoin this activity. Will he prevail? A. No, since Wilson, has no notice of the restriction in his deed from CoCompany.

B. No, if Wilson relied on the zoning regulations before purchasing his property.
C. Yes, since we have a restrictive covenant that runs with the land, binding Wilson to the covenant. D. Yes, if Chase can show that a reciprocal negative servitude is implied due to the original subdivision common scheme.

Answer: C A covenant runs with the land if it touches and concerns the land and is intended to run. Here, the covenant regarding use of land for commercial purposes clearly touched and concerned the land. Furthermore, it is apparent that the original party intended this to run with the land, as the land was part of a subdivision with a common scheme. Therefore, Wilson is bound even though his deed did not include this covenant and restriction, as he was on constructive notice. In light of this, Answer A is incorrect. B is incorrect. Meeting zoning requirements is meaningless in light of the covenant relating to the land. Here, Wilson is bound by the covenants regarding zoning running with the land. D is incorrect. Chase does not need to show a servitude implied from the original common scheme, as there was a recorded covenant regarding the land, and Chase is free to enforce this covenant in equity as an equitable servitude.

2007-2008 Law Decks

Fropert9 Law
Murray and Rodney are adjoining land owners. Murray decides to develop a campground on CampySwamp, and submits a development plan outlining the camping areas. The plan has access via a road around CampySwamp, which Murray is going to construct and turn into a public road that he refers to as CampySwamp Road. The plan is recorded. Rodney owns GolfClub Acres, which only has small dirt road as access to main roadways. Rodney sells GolfClub Acres to Kathleen. Kathleen's deed has similar language to prior recorded deeds, but contains additional language directed to road access which states, "bordered on the south by the new CampySwamp Road, soon to be constructed by Murray, CampySwamp's owner," and references Murray's plan for a public road. Kathleen seeks a building plan giving her access from Murray's new road to GolfClub, bypassing GolfClub's poor dirt road. Murray objects, stating that he never gave Rodney any rights originally, and that Kathleen, therefore, has no rights to use the new CampySwamp Road. Kathleen's best argument is: A. The deed from Rodney to Kathleen referenced Murray's recorded plan, thereby, creating rights in Kathleen to use this public street. B. The recording of the plan by Murray gives CampySwamp Road over to the public, which may subsequently be accessed by Kathleen anyway based on Murray's proposed public use of

the road.

C. Kathleen can gain access by way of necessity to CampySwamp Road.


D. Kathleen can only gain access to GolfClub Acres by spending a lot of money to update the existing

dirt road. Therefore, there is sufficient need for access to Murray's CampySwamp Road.

Answer: B
Generally, when a city approves plans such as these, they have accepted the dedication, allowing Kathleen to gain access to the new public road. A is wrong. Rodney simply referring to the recorded plan does not give Kathleen the right to use the plan (and road). C is wrong. A necessity argument is not proper as Kathleen does, in fact, already have access, although not as convenient as the proposed new road, to GolfClub Acres. D is wrong because there is already a means by which Kathleen can gain access to GolfClub Acres. The fact that it is costly to update the dirt road, does not give rise to an easement by necessity.

2007-2008 Law Decks

Fropert9 Law
Mame is affluent and in her will dated March 1, 1950 leaves everything to Patrick. Mame dies in 1991. When Patrick meets with Mame's executors, Patrick learns that 2 pieces of property were deeded to others; namely, the Rough Ride Ranch to Brigitta in 1975, and the Vino Vicarage to Rosalind in 1990. The deed to the ranch includes the clause: "for so long as the property is used to house sick parakeets. If the property is used for any other purpose, then it goes to the Able Aviary Association." The deed to Vino Vicarage includes: "as long as the vicarage is used as a headquarters in campaigning for a tax cut for the wealthy to pass by 2004. However, if the tax cut is passed before 1990, then the property should pass to Patrick at that time." Mame died leaving only one son, Fred, who was a bastard who never called his mother and so received nothing at all in Mame's will. This is a common law jurisdiction, and the state's probate laws provide that future interests, or estates in real property, may be passed by will or descent in the same manner as present or possessory interests. (Note that neither Brigitta, nor Rosalind qualify as a charitable institution.) In 1995, Rosalind and Fred join together to sell the vicarage in fee simple to Jack. Patrick sues to prevent this sale. A court will find for: A. Rosalind and Fred, as together they own a fee simple interest in the building.

B. Rosalind and Fred, as the proposed restriction on the use of the property is in violation of the rule
against perpetuities. C. Rosalind and Fred as the deed restriction is an impermissible restraint on alienation. D. Patrick, as he never signed the contract for sale.

Answer: D
Here, Patrick is an interested party in the sale, as he has an interest in the property. Rosalind's interest is only for as long as Rosalind is working for the tax cut by the year 2004. Since there is no provision as to what to do with the vicarage if Rosalind stops working on the tax cut issue before 2004, Patrick may gain this interest via a possibility of reversion to Mame, which is passed to Patrick with the will. Therefore, the property cannot be sold without Patrick's signature. In light of this, Answer A is clearly wrong. B is wrong because the interest in the estate will pass to Fred, if at all, within the 21 year time allowed by the rule against perpetuities and is, therefore, permissible. C is wrong because Rosalind could potentially pass a defeasible fee to an interested party.

C)2007-2008 Law Decks

Fropert9 Law
e

Romeo owns ocean front property and conveys it, "To the woman who is the most precious and special person I have every known, Juliet, for life, then to the heirs of my brother, Monty." At the time of the conveyance, Monty has not yet married, preferring the swinging singles life, and has no children that he knows of. At the time of Juliet's death, Romeo is still alive and Monty is unmarried and childless. When Juliet dies, to whom should her interest go? A. Monty B. Romeo C. Juliet's heirs D. Monty's heirs

Answer: B
Here, we have a reversion interest in the grantor, which occurs when the grantor conveys a lesser vested estate than he has. In a situation where the reversion was not expressly retained, operation of law will result in a reversion being created. Since Monty is still alive, his heirs cannot be ascertained (heirs are only after death) so Romeo takes a reversion until Monty's heirs can be ascertained and the property can pass to them. Note: This assumes a modern day law application in which the destructibility doctrine is not applicable to the interest of a not yet ascertained set of heirs. A is wrong because Monty has no interest, only his heirs have an interest in Romeo's ocean front property. C is wrong because Juliet's interest terminates upon her death since it is only a life estate. There is no ocean front property, then, for Juliet to pass on to her heirs since she loses her interest upon death. D is wrong because, during Monty's life, he has no heirs. Heirs only occur after death.

2007-2008 Law Decks

Fropert9 Law
Jack owns Stone's Skip, a large rugged piece of land. Stone's Skip has a river running through it with fish, but it is not navigable. The river cuts Stone's Skip in half, making a northern portion and a southern portion. Jane is interested in the northern portion for raising wild dogs. Jack and Jane agree on a price and Jack conveys, "the northern part of Stone's Skip, as separated by the river, to Jane." The deed states that this 100.25 acres, but upon a survey the actual amount is 101.57 acres. Which of the following is true in a dispute between Jack and Jane regarding the mistake? A. The deed is invalid because of the mutual mistake of the parties. B. The deed is invalid unless the court allows the Parol Evidence Rule to bring in evidence as to the amount of acreage conveyed. C. The deed is valid, Jane owns 100.25 acres.

D. The deed is valid, Jane owns 101.57 acres.

Answer: D
The physical description always takes precedence over a quantity description, unless there is a reason for reformation of the deal. Here, the physical description is applicable, giving Jane the 101.57 acres. In light of this, Answer C is clearly incorrect. The facts, here, are such that we are dealing with a bargain between Jack and Jane for the northern portion of Stone's Skip, not a bargain for an exact amount of land at issue. Therefore, there appears to be no grounds for reformation. Answers A and B are wrong since a conflict in description does not invalidate the deed between Jack and Jane.

02007-2008 Law Decks

Fropertg Law
Raphael buys land in 1975 from Michael by warranty deed. Raphael does not know that Michael really doesn't own Purple Plume Hill, but that it is actually owned in fee simple by Leo. After buying from Michael, Raphael builds a house and digs a pool on the northern tip of Purple Plume Hill. The remaining parts of the property are vacant. In 1980, Raphael builds a garage on a 10 acre plot that he thinks is part of Purple Plume Hill. In reality, these 10 acres belong to Leo. Leo lives out of state and doesn't know that a garage was built on his property. In 1990, Leo dies, having not been back to Purple Plume Hill since Raphael built the garage. Leo leaves his property to his daughter, Lisa, a 6 year old, via a will. In 2000, Raphael dies leaving all his property to David. The adverse possession period in this state is 20 years. In 2002, David learns that the 10 acres with his garage are not part of Purple Plume Hill. Assume that in 1977 Leo learned of Raphael's purchase, and refused to let Raphael reenter his land. Raphael and Leo go to court. Raphael calls Michael and says, "You need to straighten this situation out for me. I'm paying bills left and right for nothing' Michael says, "Sorry, but I'm not getting involved." If Raphael sues for damages stemming from breach, which of the following is most applicable? A. The covenant of quiet enjoyment only.

B. The covenant of seisin, right to convey, quiet enjoyment, warranty, further assurances, and the
covenant against encumbrances. C. The covenant of seisin, right to convey, quiet enjoyment, warranty, and further assurances. D. The covenant of seisin and right to convey only.

Answer: C Based on the facts, Michael does not own, nor was he acting as Raphael's agent, when dealing with Purple Plume Hill. Therefore, Michael has clearly broken the covenant of seisin and right to convey when Michael conveys to Raphael. Answer A, therefore, is incorrect. B is incorrect. There are no facts to suggest that the property was originally encumbered, so a breach of the covenant against encumbrances is clearly wrong. D is wrong. Under the covenant of quiet enjoyment, warranty, and further assurances, Michael promises Raphael that: 1) Raphael's possession of the land would not be disturbed; 2) Michael would defend Raphael against lawful title claims; and 3) Michael would do whatever necessary to perfect Raphael's title. Clearly, Michael has breached these, making Answer D wrong and Answer C the correct answer.

2007-2008 Law Decks

Property Law
Colby conveys land in Maine to Pat from valid consideration that is the fair price. The deed adequately defines the land. The land itself is divided by a road into 2 plots: the northern plot being full of trees and a stream, the southern plot being farmland. Both Colby and Pat (once he owned it) used the southern plot for blueberry farming. Neither Colby, nor Pat ever used the northern plot for anything more than the occasional walk. Unknown to Colby or Pat, Colby got his deed through a forged deed. Ten years after Pat has bought the land, Ling arrives. It was Ling's name that had been forged on Colby's deed. Suppose that the Maine adverse possession clause has a 5 year requirement. Ling admits that Pat has now acquired the southern plot, but asserts that the northern plot is still hers. Who will a court determine the northern plot belongs to? A. Ling, since Pat never officially occupied the northern plot. B. Ling, since Pat cannot obtain color of title through a forged deed

C. Pat, since by farming the southern plot of land he has constructively occupied it all.
D. Pat, since Colby had no knowledge of the forged deed.

Answer: A
An adverse possessor will gain title only to land that he actually occupies. Occupation must be the kind that a true owner would have made, thereby giving the true owner notice of the trespass. Here, since Pat never went to the northern plot, he failed to put Ling on notice as he failed to occupy the land.

B is wrong for two reasons. By definition, "color of title" merely means possession of a document claiming to give title to the land. Pat clearly has this. Additionally, under an adverse possession claim, color of title is not necessary. In most jurisdictions, a person does not need to believe that they have a right to possess land. They can be a trespasser and still gain adverse possession of the land.
C is wrong because constructive possession of the northern plot, based upon actions in the southern plot, is insufficient to avoid an adverse possession claim. Most jurisdictions will claim that if an adverse possessor has possessed a significant portion of the whole, they will be granted the entire piece if it is a unitary piece of land. Here, the possession is 50% and
we are not dealing with a unitary piece (due to the road cutting it into northern and southern

plots), making a successful adverse possession claim by Pat for the entire land unlikely.
D is wrong since in reality Colby had no title to convey to Pat. The only way that Pat can
have possession to any portion of the land is via an adverse possession claim, not one based upon color of title.
2007-2008 Law Decks

Fropert9 Law
Kumar and Kim have an undivided % interest in Faulk Island. Their agreement states that each has the right to possess all portions of the island and neither has the right to exclusive possession of any part. One day, Kim ousts Kumar from the island wrongfully. If Kumar sues, he can recover: A. The fair rental value of the property for the time excluded.
B. 1 /2

of the fair rental value of the property for the time excluded.

C. 1/4 of the fair rental value of the property for the time excluded. D. Nothing, because each co-tenant has the right to possess all portions of the property and neither has the right to exclusive possession of any part.

Answer: B
If a co-tenant ousts another co-tenant, the ousted tenant is entitled to % of the fair rental value of the property for the time he was wrongfully deprived of possession. Here, there was clearly a wrongful ousting; making Kim liable to Kumar for % the rental value for the time that he was excluded. A is incorrect as Kumar is only entitled to his fair share (% the value). C is incorrect again, as the fair share is % not %. D is a correct statement but it only addresses the right to possession. The share of ownership determines the share of rental value.

2007-2008 Law Decks

Property Law
Karen borrows $30,000 form Caton Bank to start a business, pledging her house and property as collateral. There are no other loans on the house and property. Months later, in a booming economy, Tim offers Karen $200,000 for her house and property. Karen accepts instantly and they execute a written land sale agreement. Closing date is June 24th. On June 1st Tim's attorney does a title search and learns of the mortgage to Caton Bank as collateral. He informs Tim, who in turns calls Karen. Karen says, "I'm using the cash from the closing to pay off the loan. It's no big deal at all." Tim gets cold feet and never shows up to the closing on the 24th. If Karen sues demanding specific performance and wins, what is the likely reason? A. A mortgage used to secure a debt does not constitute a legal "encumbrance." B. The vendor of real property need not have marketable title until the time of the closing. C. The mortgage was unenforceable ab initio, because a preexisting debt is not

adequate consideration.
D. A mortgage of the type described does not follow the land and Tim would have taken the land free of the mortgage even if Karen did not pay it off.

Answer: B
So long as Karen was willing to deliver marketable title to Tim at closing, she will likely win. A seller of property only needs marketable title at the time of closing, not before. For example, a seller can eliminate any existing encumbrances at closing using the proceeds from the sale. As long as the proceeds are sufficient to cover the encumbrances, the seller is free to clear the title to the property at closing. Therefore, Tim cannot claim that the title is unmarketable and must go through with the closing as planned. A is wrong as any mortgage, regardless of type, is an encumbrance on a property. C is wrong as the mortgage here is still enforceable. The fact that the mortgage was granted for an antecedent debt only results in a lack of protection under the recording statute, but this is meaningless to the present facts. Here, Tim had notice of Cat& Bank's interest and could not cut off their interest by placing his first. The loss of protection by Cat& Bank would only be against any potential lien holder that came in front of them, of which there are none. D is wrong as all mortgages follow the transfer of land. If the mortgage was not paid off Karen is no longer personally liable for the debt and the land could be foreclosed against by CatOn Bank.
2007-2008 Law Decks

rropert9 Law
Julia is a rich rock star who owns several properties. She often works from one of her homes via the internet. One summer, she decides to work from her Seal Harbor property in Maine. Ordinarily, she doesn't work from this location in August, since she usually spends August in New Mexico. Unknown to Julia, for years Marta (her neighbor in Seal Harbor) has been using a portion of Julia's land to have an end of summer clambake for the entire town. Marta has been doing this for 14 yrs. This year Julia notices and asks Marta to stop, who promptly refuses. Julia files suit to enjoin Marta from doing this. Assume that the clambake is totally legal and that the term for adverse possession in Maine is 10 yrs. Will Julia prevail in her suit? A. No, because Marta has acquired this portion of Julia's land by adverse possession. B. No, because Marta has, in effect, obtained an easement by prescription. C. Yes, because Marta's use was not continuous. D. Yes. Because Julia had no prior notice of Marta's use, Julia cannot be deemed acquiescent to Marta's use.

Answer: B
Here, Julia looses. Marta has an easement by prescription, which is a right to use another's land for a special purpose, with no right to possess and enjoy the property. An easement by prescription is similar to an easement by adverse possession, requiring the use of the land to be: open and notorious; adverse and under claim of right; and continuous and uninterrupted for the statutory period. Mata's use clearly meets these requirements. Note: Continuous adverse use does not mean constant use. A is incorrect as Marta did not entirely possess Julia's property and, therefore, fails to meet the continuous possession requirements of adverse possession. Here, Marta did not possess, but rather just used the portion of land once a year. C is wrong as Marta's use was often enough to acquire a prescriptive easement. All that is required for a prescriptive easement are periodic acts that put the true owner on notice. D is incorrect as Marta's use of the land was open and notorious enough to put Julia on notice. The fact that Julia is typically not there each August is meaningless. Marta's acts were enough for the easement requirements.
2007-2008 Law Decks

Fropert9 Law
Lorraine owns a tattoo parlor and a dry cleaner. They both share a common driveway that is technically located on the property of the tattoo parlor, but used by the merchants and customers of the dry cleaner. Lorraine then sells the dry cleaner to her friend Cynthia. Cynthia continues to use the common driveway for years. Lorraine then sells the tattoo parlor to Leo, who also continues with the status quo. Years later the driveway has huge deep potholes from years of ice and wear. Cynthia is concerned, and approaches Leo, asking him to have the driveway fixed and offers to pay half the cost herself. Leo refuses and storms off. Cynthia, fearing injury to her customers or damage to their cars, gets an estimate to fix the driveway anyway. Cynthia takes the estimate to Leo and this time offers to pay the entire cost. Again, Leo refuses. If Cynthia sues Leo, can she compel him to fix the driveway? A. Yes, Cynthia has an easement, which implies the power to repair. B. Yes, Cynthia has a right to guard against tort suits, making repairs to the dangerous potholes in the driveway necessary and acceptable.

C. No, because Cynthia's interest in the driveway is only for the reasonable lifetime of that
driveway. D. No, Cynthia has no right of entry to Leo's property.

Answer: A
Cynthia has an easement by implication, which is created by operation of law rather than by written instrument, and has a right to repair the driveway. This easement by implication is an exception to the Statute of Frauds, and does not need to be in writing. This type of easement is implied if, prior to the time when the property is divided, a use exists on the "servient part" that is reasonably necessary for the enjoyment of the "dominant part," and the parties intend for the use to continue after the property is subsequently divided. Note: The use must be apparent at the time the property is divided. In determining reasonable necessity a court will look to the cost, as well as the difficulty, of the alternatives. Here, the requirements of the easement by implication were met when Lorraine conveyed the dry cleaner to Cynthia, which in turn passed to Leo when Lorraine conveyed the tattoo parlor to Leo. B is wrong for stating a right that clearly does not exist. Absent an easement as set forth in the correct answer, Cynthia has no rights to enter Leo's property. C is wrong. An easement is generally for a perpetual duration unless there is some writing in existence that limits the length of the easement's duration. Here, there are no facts to indicate a limit on the duration of the easement. D is wrong in light of the correct answer as Cynthia has an easement implied from an existing user, which gives Cynthia the right to enter Leo's property.
@2007-2008 Law Decks

Froperty Law
Kazuo owns a huge hunk of property in a rural district of Oregon. He builds a home on the northernmost region, and grows Christmas trees on the southern region. Kazuo becomes ill from pesticide poisoning and spends his last years in a hospital bed. Right before dying, he gives the northern parcel with his house to Henry stating: "To Henry, for life and then to his widow for her life, and then to Henry's surviving children." The southern region goes to Roy, a friend. Kazuo dies leaving a single heir, his brother Tanaka. Kasuo's house is in disrepair as he did not have the means to fix it while he was sick. Because of this, Henry is not able to rent it to anyone. Henry decides to level the house and plant Christmas trees for income. The property would be worth much more this way, but Tanaka has a sentimental attachment to the home and wants Henry to leave it as is. This jurisdiction has the common law rule against perpetuities. If Tanaka has standing to sue, which of the following actions could a court go forward and take? I. Enjoin the leveling of the house. II. Order Henry to continue to pay the property taxes. III. Order Henry to make any necessary repairs. IV. Allow Henry to level the house, but in turn require that he pay damages to Tanaka. A. B. C. D. IV only II and III only I, II, and III only I only

Answer: D Here, Henry is a life tenant and has the requirements not to change the premises in a
way that the holders of subsequent estates have a reasonable ground to object to. The

only permissible changes are those that result from a change in the neighborhood wherein the land is no longer good in its current form. Here, there are no facts indicating this to be the case, and Henry will not be permitted to destroy the house in light of Tanaka's interest. A is wrong. Tanaka suffers no monetary reduction or damages if the house is leveled, as the house has no value in its present condition. If a court refused to stop the destruction, they cannot award damages. B and C are wrong because a court cannot force a life tenant to pay taxes or make repairs. When a life tenant receives a piece of property, they are not obligated to make repairs at the time the property is passed to them, regardless of whether the property needs it. A life tenant is only obligated to preserve the land and structures in a

reasonable state of repair, and to pay ordinary taxes to the extent of rents and profits collected from use of the property. Here, Henry is not occupying the land and cannot
gain any rent due to the poor condition of the house. Therefore, he cannot be forced to pay the repair bills and taxes.
02007-2008 Law Decks

Property Law
Bruce owns Icebellow Farm. He is nearing retirement, but his kids show no interest in farming the earth and raising grain. He puts his highest hopes in his son Murray, a cheesemaker, thinking that he will someday come around, after a few years. Bruce is diagnosed with a week to live, based upon a tetanus infection, and drafts his will. It reads: "I leave lcebellow Farm to my best friend and good neighbor, Wong, for life, then to my eldest son Murray in fee simple absolute." The remainder of Bruce's property is to be divided among his remaining children, his only heirs at the time of his death. Bruce passes away and his estate is dispersed. Weeks later, Wong discovers that Icebellow Farm has diamonds beneath the surface and begins to start mining the property and digging tunnels. Murray asks Wong to stop the mining, but Wong (who has 3 kids in college) refuses. Murray sues seeking to stop Wong. The court will: A. Partition the property.
B. Enjoin Wong from mining and award Murray with damages.

C. Order a sale of the property and a division of the proceeds between Murray and Wong. D. Allow Wong to keep on mining and keep the profits.

Answer: B
A court will enjoin Wong from mining and make him liable for damages. He is simply a life tenant and a life tenant cannot consume or exploit natural resources on the property without it being considered voluntary waste. In such a situation the remainderman (Murray) may sue for damages and an injunction. A is incorrect. A partition declaration is generally only applicable to a co-tenant, not to those who hold successive estates. At best, a life tenant may ask for judicial sale in a partition proceeding if a court finds that the land itself is worthless in the condition that the life tenant received it in. However, this is clearly not the case here. C is incorrect because a partition sale is not appropriate (see above).

D is incorrect. Depleting the natural resources on a piece of property is considered waste as it diminishes the value of the property and hurts the remainderman. In light of this, generally speaking, a life tenant cannot exploit the natural resources of the piece of property.

2007-2008 Law Decks

Fropert9 Law
Van owns Roadside Estate, which butts against the road. Vin owns PiperGate Glen, an undeveloped lot that does not have road access but butts against Roadside Estate. The easiest access to PiperGate is across Roadside. Van grants Vin an easement to "pass through," allowing Vin to drive along one side of Van's property to get to PiperGate. However, when Vin seeks to have electrical cables run underground along that same area of Van's property, Van refuses. The electric company refuses to run the cable across Van's property (even in light of eminent domain) without Van's approval. Can Vin force Van to allow the running of the underground electrical cables? A. Yes, by exercising the electric company's right of eminent domain. B. Yes, there is an easement of necessity relating to the need for electricity. C. No, because Vin's easement from Van is for a different purpose.

D. No, because the property is undeveloped and there is no established necessity for electrical service.

Answer: C
Vin has no rights to force Van to allow the installation of underground electrical cable to PiperGate Glen, as a change in the basic understood nature of the easement is not permissible. A is incorrect because Vin has no right to exercise the eminent domain rights of the electric company. B is incorrect because easements, by necessity, only address easements of way and have no bearing on easements to allow an electric company access. Note: The existence of an easement by necessity arises when the owner of a single piece of land sells a portion to another party and the second portion has no road access. Here, there are no facts indicating that both lots were ever owned by the same person. D is incorrect. Even if we assumed an easement by necessity, necessity is determined at the time that the land is divided. Here, we have no information as to whether the land

was ever a single piece, making a easement by necessity analysis difficult or impossible.

2007-2008 Law Decks

Property Law
Tensin owns Bella Field and has properly recorded his deed and title with the state of South Carolina. Tensin conveys Bella Field to Mike by warranty deed. Mike fails to record. Later that week, Tensin conveys Bella Field to Plum, for "one dollar" via a written contract. On the closing date of the contract, Plum hands over $1 via certified check and Tensin hands Plum a quitclaim deed conveying Bella Field to Plum, following which Plum records with the state of South Carolina. Suppose that the applicable statute in South Carolina is: "Any conveyance of an interest in land is not valid against any subsequent purchaser for value without notice thereof whose conveyance is first recorded." What is the greatest problem that Plum is going to face in establishing ownership of Bella Field? A. A quitclaim deed is not a valid transfer of title. B. Proving that the transaction was a transfer for value.

C. Warranty deeds are superior in right to quitclaim deeds.


D. A quitclaim deed is not a recordable instrument.

Answer: B
To be a bona fide purchaser for value the purchase must pay actual fair consideration, not a nominal amount like $1. Here, it looks like Plum is more a donee than purchaser. The consideration need not be adequate or the market value, but it must be of substantial pecuniary value. Plum will have a difficult time proving that her $1 meets this test. A is wrong. Title can be conveyed by quitclaim deed. C is wrong because a warranty deed is not superior in right to a quitclaim deed. The only difference between the two is that a warranty deed normally contains covenants for title, the breach of which gives rise to a cause of action against the grantor, while a quitclaim deed contains no assurances by the grantor. D is wrong because all instruments affecting title to real property are recordable and should be recorded.

2007-2008 Law Decks

Fropert9 Law
Essex has a will which reads "I leave Pine Castle Estate to my loving valet, Bernard, for life, the remainder to my niece, Qwara, so that she may live out her days going on fox hunts." Pine Castle Estate is a 100 acre parcel with a mansion, pool, dock and extensive hunting grounds. When Essex dies, Bernard is 50 years old and Qwara is 18 and living off of her $1 billion trust fund. Pine Castle Estate has a large mortgage outstanding and money from renting the hunting grounds to wealthy locals barely pays the taxes. Qwara, being of a generous animal-loving nature, agrees to pay the mortgage herself so that the animals on the hunting ground can remain in peace. She tells Bernard that she is looking forward to her retirement and living on the grounds later in life. Fifteen years after Essex has died, a commuter rail from the local metropolis is run through the county. Many young people begin to move out to the rolling countryside now that there is easy access to the city. Bernard is offered $10 million from Hump, a land developer, for 10 acres of Pine Castle Estate (including part of the hunting grounds) on which he plans to build hunt-theme condos and a hotel. Hump knows that Bernard is only a life tenant, but figures that he can keep him alive long enough to make back the money it would cost to level the mansion and build the condo-park plus profits. Bernard tells Qwara all about this and offers to share the proceeds, but Qwara refuses, wanting to preserve the land. Bernard goes ahead anyway, and Qwara sues seeking to enjoin construction. What is Qwara's best argument?

A. The eventual use of the property by the remainderman will be as a residence.


B. Destruction of the house and hunting grounds is waste. C. Since Qwara has been paying the mortgage, Bernard must subrogate Qwara's rights to the land. D. Bernard has the right to transfer his life estate.

Answer: B
A waste argument is the best and most applicable. Here, a life tenant is free to any ordinary use of land, but cannot act in a manner that severely alters the estate that a remainderman will take, even if the proposed actions will make the estate more valuable. A life tenant is only allowed to alter or level preexisting buildings if 1) the market value of the future interests is not diminished and either 2) the remainderman does not object, or 3) a substantial and 'permanent change in the neighborhood conditions has deprived the property in its current form of reasonable productivity or usefulness. Here, Qwara is clearly objecting, making option 2 incorrect. Furthermore, there is insufficient factual basis to assume that the neighborhood has changed enough to allow conversion of the residence into a multiple-housing project and hotel. A is wrong as Qwara's intended use of the property is meaningless. Even if Qwara intended to build a resort, she has a right to receive the property in the same condition as it was left to Bernard. C is wrong. The mortgage payments do not grant Qwara a greater right, as the payments

were made simply to protect Qwara's rights.


D is wrong. A life estate, absent language in the grant otherwise, is freely alienable. Note: A grant of life estate to a person is only in effect for the duration of the life of the life tenant.
2007-2008 Law Decks

Fropert9 Law
Lilly changes the zoning of her property to commercial. She records the zoning changes along with a plan to subdivide the land into a spa, with numerous homes (including condos, apt buildings and multifamily houses) having access to the spa. An entire gated community is developed with an automotive theme; which is included in the development plan as covenants, conditions and restrictions, wherein each house and store must have a car motif, approved by the merchants association. Twenty acres in the center of the community are set aside as a racetrack, deemed a "public sports arena." Ten years later, Lilly sells a number of the lots in the spa to various homeopathic healers that agree to maintain the car motif: Acupuncture Juncture, Manual Shift Massage, Manifold Manicures, etc... She also sells numerous residential lots, each of which has the car motif restrictions set in their deed. One spa lot in the center of the spa region is sold to Tom, who 5 years later sells it to Gary without the deed restriction. Over a weekend, Gary converts the lot into a safari-themed bicycle shop. Soo owns Acupuncture Juncture in the spa. It neighbors Gary's spa lot and is in compliance with the car theme deed restriction. On Monday, Soo sees Gary's safari motif and instantly seeks an injunction compelling him to demolish the entire store since he failed to obtain consent from the merchant's association. Should the court issue the injunction? A. No, because destruction of the store is a waste of resources. B. No, the deed Gary obtained had no deed restriction. C. Yes, because the restrictive covenant runs with the land. D. Yes, unless Gary can establish that his safari-themed bicycle shop has as much merit as the car motif stores.

Answer: C
A covenant such as this runs with the land, therefore, the court should issue the injunction. A covenant such as this is enforceable under a theory of equitable servitude allowing a covenantee, covenantor, or successor to enforce the covenant via the use of an injunction. A party must prove that: 1) the covenant in a writing satisfies the Statute of Frauds, 2) touches and concerns the land and 3) indicates an intention that the servitude exists, and that 4) notice is given to future owners of the burdened land. Here, we clearly have a written covenant, which touches and concerns the land, benefiting all of the lots and burdening all of the lots. The intention to create is established via the writing, as well as the implied common theme of the entire development. Notice was found, as all of the original deeds of Lilly had the covenant in them. A is incorrect. While an injunction is equitable in nature, merely claiming that it would be a waste of assets is not a defense. B is incorrect. Gary clearly had record notice of the deed restriction as it was in the originally prepared deed from Lilly to Tom. This deed was clearly in Gary's chain of title

and could easily be known to him had he looked.


D is incorrect. Note: A court will enforce a covenant, or potentially choose not to enforce a covenant; but a court will not get involved in modifying a covenant.
2007-2008 Law Decks

Property Law
Maya owns 2 adjoining lots, Lot 1 and Lot 2. Lot 1 abuts the road, and has street access. Lot 2 does not and uses the parking lot across Lot 1 to gain access to the road. Maya runs a gas station on Lot 1 and an ice cream parlor on Lot 2. Since the ice cream parlor is seasonal, it is only open May to September. Maya sells Lot 2 to Buddy and deeds an easement to use Lot 1 parking lot for access. Buddy promptly records the deed. Two years after the sale to Buddy, Maya sells the gas station to Cliff, but the deed fails to mention the easement. Cliff demolishes the gas station and builds a diner, blocking the parking lot to Buddy's ice cream parlor. This occurs in December, and Buddy has no idea that it is happening as the ice cream parlor is closed for the winter. In April, Buddy arrives to ready the ice cream parlor for spring opening and finds that he no longer has any access to his store. Buddy brings an action to compel Cliff to relocate a portion of the diner to allow the parking lot access. The shortest route for Buddy to the street is through Cliffs lot. There is an alternative, but it would require passing through 6 parcels of land to get to the street. How should a court rule on Buddy's motion? A. For Cliff, as his deed had no mention of the easement.

B. For Cliff, as once he built the diner and blocked the driveway Buddy's rights were terminated.
C. For Buddy as he owns the easement and has a right to access his lot. D. For Buddy, as there is no reasonable alternative access to his lot from the street.

Answer: C The court should rule for Buddy. Buddy has an easement which is the right to use Cliffs property for access. Absent language to the contrary, once an easement is granted it is perpetual in duration. Therefore, the transfer of the property from Maya to Cliff does not terminate Buddy's easement. Furthermore, Cliff has notice, as Buddy recorded the easement.

A is incorrect since the recorded deed to Buddy is deemed constructive notice for Cliff,
the purchaser. Therefore, Maya does not need to mention the easement when selling to Cliff or put it in Cliffs deed.

B is wrong. The statutory period for terminating the easement is typically the same period as necessary to obtain an easement by prescription. To terminate an easement, Cliff would have to have openly interfered with Buddy's easement for the statutory period. Generally, this statutory period is always at least 10-20 years. D is incorrect. While there might be an easement by absolute necessity (i.e., Buddy has
no other reasonable access); it is inapplicable as an answer since Buddy has an express easement from Maya.

2007-2008 Law Decks

Fropert9 Law
Soledad sells the mortgage and note on Four Rivers Manor to WaterBank. WaterBank goes ahead and resells the mortgage and note to Bank-a-lot. Bank-a-lot properly records the sale of the mortgage as required under state statute. Bank-a-lot, a small bank, decides to use WaterBank as the collection agent for the Four Rivers Manor mortgage as payments come due. Bank-a-lot, therefore, leaves the mortgage and note in the hands of WaterBank. WaterBank is owned by Chainley, a shady business man, who is running WaterBank into the ground. Desperate, Chainley sells the Four Rivers Manor mortgage to TriBank. This helps, but it's not enough to keep WaterBank afloat and they go under. When wrapping up WaterBank's affairs, Bank-a-lot learns of the sale of the mortgage to TriBank. Bank-a-lot further learns that WaterBank never told TriBank about Bank-a-lot's interest in the mortgage. Bank-a-lot sues WaterBank for return of the mortgage and note. The court will decide that: A. Bank-a-lot owns the mortgage and the note.

B. TriBank owns both the mortgage and the note. C. Bank-a-lot owns the mortgage, TriBank owns the note.
D. TriBank owns the mortgage, but Bank-a-lot owns the note.

Answer: A
By definition, a mortgage is a security interest in a property and a note is evidence of the underlying debt. One need not have physical possession of the mortgage and note to own them. In light of this, Bank-a-lot stills owns both the note and mortgage as Banka-lot has bought the note and recorded it, even though it left possession with WaterBank. TriBank has no interest in the mortgage or note, as there is record notice from Bank-a-lot's recording should they have chosen to investigate. In light of this, TriBank has no interest in either the note or mortgage. B is wrong because, under a first in time statute, Bank-a-lot was the first in time to purchase and record the mortgage and note, making them valid holders of both. C and D are wrong because (as stated above) the mortgage and note are a package, and are never separated. Note: When facing a question which separates the mortgage and note, instantly rule out answers that find different parties holding different pieces. A mortgage and note travel

together.

2007-2008 Law Decks

Frope rt 9 Law
Kendis owns Villa Morris in fee simple, and following her husband's death she conveys Villa Morris: "To my only child Canda, for life, remainder to the children of Ace in fee simple." Ace was Canda's only child at the time that conveyance was made. Note, however, that both Canda and Ace were married, but Ace had no children at the time of conveyance. With regards to Ace's own children (Kendis's great-grandchildren), what interest would they hold on conveyance? A. None B. A springing use C. A contingent remainder D. A vested remainder subject to divestment

Answer: C
By definition, a remainder is a future interest in a transferee that can be taken upon the of the preceding estate. A remainder must be expressly created in the instrument creating the prior possessory estate. Here, we clearly have a remainder, as the interest in Ace's children follows the termination of Canda's estate. The remainder is contingent since we cannot ascertain right now if there will be any takers. Note: When dealing with created remainders that would apply to unborn people, such remainders are always contingent.
termination

A is incorrect since Ace's children clearly have an interest, namely the contingent remainder set forth in Answer C.

B is incorrect. A springing use serves to cut short the estate that is held by the grantor and a remainder does not cut short the estate of that grantor. D is incorrect because a vested remainder requires the existence of at least one

remainderman at the time when it is created. Here, we do not have the required
remainderman ascertained at the time of the creation of the remainder, making D incorrect.

2007-2008 Law Decks

Fropert9 Law
Max owns Belfry Field which contains a house that Max built himself out of recycled tires and bat guano. Max agrees to sell Belfry Field to Latanya for $100,000 and signs a contract for the sale. The contract states that Max will deliver title to Latanya, subject to all applicable zoning laws and covenants or restrictions of record. There is a zoning law stating that no structure can exceed 20 feet. One week before closing, Latanya has a survey done and Max's house stands 20 feet 6 inches tall at the belfry, its highest point. Furthermore, there is a restriction in the deed stating that no house can exceed 15 feet, as set forth by the original developer when he subdivided the land. Latanya refuses to go forward with the sale, on the basis that the title is not marketable and that she will be subject to lawsuits if she goes through with the purchase. If Max sues to ask for specific performance, how will a court rule? A. For Max, as any suits regarding the height of the house would be frivolous. B. For Max, as the height is a de minimis non curat lex violation. C. For Latanya, as any violation, regardless of how small it is, is a breach of contract. D. For Latanya, as she may be exposed to a lawsuit, making the title unmarketable.

Answer: D
Latanya will prevail as the title here is unmarketable. Implied in each land sale is an implication that the seller will deliver marketable title (i.e., that the title that is free of potential litigation). Here, we have a height that is in violation of the deed restriction of the original developer, making it quite possible that the owner (either Max, or Latanya should she purchase) would be subject to a lawsuit. Additionally, the belfry 6 inches over the 20 foot zoning ordinance further makes the title unmarketable due to another potential suit. A and B are, therefore, incorrect. C is also wrong as being a total misstatement of the law.

2007-2008 Law Decks

Fropert9 Law
Suppose that Oklahoma City owns a field used as an unpaved parking lot by its residents. They build a new parking garage in a more convenient location, so the field is no longer used by the city residents for parking. Jamar owns the property adjoining the field. He notices that people stop parking there and fences in a part of the field. He uses this part of the field to grow crops of tomatoes for years and years, well past the time necessary for adverse possession. If Jamar decides to bring an action to establish his rights to the parcel on which he grows tomatoes, which of the following answers must he prove? I. A municipality can lose its real property interests via the actions of adverse possession. II. Jamar's use of the parcel serves as proof that he has asserted dominion over the parcel for years. III. A municipality cannot abandon fee interests in real property without an official vote. IV. The municipality's lack of use created an irrevocable license in Jamar for his use. A. I and II only

B. I and IV only C. II and III only


D. III and IV only

Answer: A
Jamar is clearly asserting an adverse possession claim. Therefore, we must establish
whether Jamar can, in fact, adversely possess land from a municipality (Statement I).

Note: Government land, generally, cannot be obtained through adverse possession. The second step necessary is whether Jamar's use constituted the assertion of dominion required for adverse possession (Statement II). Here, this is clearly the case based on the facts present. C and D are wrong for misstating the requirements of adverse possession. Jamar does not have to show abandonment by the record owner to prove adverse possession (Statement III). B and D are wrong because a license (Statement IV) is not necessary. Additionally, a showing of a license serves to make an adverse possession claim impossible, as a license would indicate that Jamar had Oklahoma City's permission and the taking then would not be hostile.

2007-2008 Law Decks

Fropert9 Law
Quiterie owns Glass House Cliff, and tells, Colette, her favorite cousin, "You've loved me like a sister. When I pass on to that big terrarium in the sky, Glass House Cliff is all yours." Colette has an outstanding debt due to Jung, and to repay him decides to deliver a general warranty deed of Glass House Cliff. Jung records the deed and moves in to Glass House Cliff. Quiterie then dies. In her will Glass House Cliff goes to Colette. Colette is again broke, so she sells Glass House Cliff to Lloyd by general warranty deed. Lloyd records instantly, and pays Colette a fair price for the property. Lloyd never moves in to Glass House Cliff as he saw it as an investment property. He has no idea that the property has an actual glass house on it which Jung is now living in. In regards to Colette's deed to Jung, which best describes the status of this deed at the time it was recorded? A. It was ineffective as a conveyance of either legal or equitable interest.
B. It effectively conveyed legal title since it was recorded.

C. It effectively conveyed equitable title. D. It was effective as a quitclaim, thereby, conveying (to Jung) Colette's expectancy of title.

Answer: A
Here, Colette owned nothing until Quiterie died, so her conveyance of title before she actually possessed anything is ineffective. In light of this, Answers B and C are clearly erroneous. Answer D is possibly a decent answer, but in light of the clearly correct answer set forth in A, D is incorrect.

2007-2008 Law Decks

Fropert9 Law
Aarti owns 10,000 acres. She wishes to develop 5,000 of those acres for a housing development. She intends to hold the remaining acres for a long time, hoping that they will become more valuable once the housing development is completed. While selling the houses, Aarti markets the remaining acres as a wildlife refuge to the current potential buyers, where they can hike, mountain bike and ride mopeds; and is okay with doing this so long as she can sell the remaining 5,000 acres in 20 years. At the end of 20 years, Aarti sells her remaining 5,000 acres to TireBurners Inc, who burn old tires in an effort to make the biggest smelling cloud. In this jurisdiction, the statute of limitations for adverse possession is 10 years. Assume that the homeowners seek to enjoin TireBurners' use of the 5,000 acres. How will a court find? A. Prevail for the homeowners, since they have acquired an easement by prescription for recreational use. B. Prevail for the homeowners, because they have acquired the tract by adverse possession. C. Not prevail for the homeowners, because the homeowners have no interest in the property. D. Not prevail for the homeowners, since the correct remedy is damages.

Answer: C
Here, the homeowners have absolutely no enforceable property interest and will lose in court. Looking at the incorrect answers we see that:
A is wrong because the homeowners have not acquired an easement by prescription, which requires open and notorious, adverse, and continuous and uninterrupted use for the statutory period. Here, the use was with Aarti's permission and, thus, was not adverse.

B is wrong. Adverse possession has the same requirements as an easement by prescription, but additionally requires use that is actual and exclusive. Here, the homeowners lack actual, adverse or exclusive use. D is wrong. The homeowners have no claim at law, making damages their only applicable remedy, not an injunction.

2007-2008 Law Decks

Fropert9 Law
Kesey owns a large piece of land adjoining to Red's land and the boundary between them is never properly surveyed and set forth. In 1950, Kesey puts huge windmills on a portion of land that he believes he owns. He also runs wires across a portion of land that he knows Red owns with Red's consent. Kesey goes into the business of selling electricity to the local electric company. In 1965, Red goes insane due to self-inflicted shock treatments and is diagnosed as mentally incompetent. He dies in 1970 and the executor of his estate sues to remove Kesey from the land owned by Red via ejectment. The statute of limitations for ejectment is 20 years. With regards to the land on which the windmills are turning: A. Kesey cannot use the adverse possession claim, as Red's mental incapacity tolls the statute of limitation period for adverse possession. B. Kesey cannot claim title by adverse possession as his use was not open and notorious. C. Kesey has title to the land under the theory of adverse possession.
D. None of the above.

Answer: C
Since Kesey has used the land as his own for longer than the statute period, it is his under an adverse possession claim. In light of this correct answer, D is clearly wrong. A is wrong because Red's disability does not toll the period for an adverse possession statute of limitations. B is wrong, in light of the facts, as Kesey's use was clearly open and notorious.

2007-2008 Law Decks

rrope rt 9 Law
Suppose that Rory owns a 50 acre parcel of land in Rhode Island. Forty acres she inherited, and 10 acres she acquired from adverse possession over Tyrell. Rory enters into a signed contract with Zane to convey the entire 50 acres. The contract states that Rory will provide good and marketable title. Zane pays the purchase price, and Rory hands over a deed. Zane records. Tyrell learns of the sale and sues for quiet title. Zane realizes that there are no covenants for title in his deed and proceeds to sue Rory. What is the likely outcome? A. Zane wins, as Rory breached the contract to provide marketable title. B. Zane wins, as Rory misrepresented the size of the land. C. Rory wins, since the terms of the deed she gave to Zane control the liability she is subject to.

D. Rory wins, since Zane acted negligently in not checking the covenants at closing.

Answer: C Rory wins since it is the terms of the deed that control the liability that she is exposed to, not the terms of the contract. The merger doctrine states that the contract merges into the deed, and the terms of the contract are meaningless. The fact that the contract states "good and marketable title" is meaningless. It is the deed that controls. A is wrong as the deed will not incorporate the terms of an earlier contract. B is wrong as there are insufficient facts to support this conclusion. D is wrong as Zane's possible negligence is meaningless in the present case.

2007-2008 Law Decks

Fropert9 Law
Wong buys a new house, but decides to keep his old house, awaiting a bump in the Wisconsin real estate market where both houses are located. Dunn sees Wong moving out and asks if Wong will rent to him. Wong states that he is intending to sell, and once he gets the right buyer Dunn would have to move. Dunn agrees and moves in for a weekly rent of $1,000, which he drops off under Wong's door each Saturday. The following month, Wong's business flops due to outsourced labor to India. He decides to downsize quickly and puts his new house on the market for only the amount of money outstanding on his mortgage. Wong quickly gets a qualified buyer and sells his new place. He then informs Dunn that he must move out so that Wong can move back in to his old house. Dunn refuses and on Saturday shows up with his weekly rent. Wong refuses to take it. Wong then brings suit to evict Dunn. Suppose that the Wisconsin statute states that 5 days notice to evict must be provided, prior to an eviction proceeding, for any tenant whose lease is less than month-to-month or without a long term contract. Here, Dunn has been given no notice. To gain immediate possession, Wong's best bet would be to argue that Dunn is: A. A tenant from month-to-month B. A tenant at sufferance C. A licensee D. A trespasser ab initio

Answer: C
Wong's best argument is that Dunn is a licensee. A licensee agreement allows a party to enter a licensor's property, but that same license may be revoked at any time by the licensor's intent to end it. A tenant at sufferance would be entitled to the statutory notice. Furthermore, a monthto-month tenant is awarded the common law right of notice equal to the tenancy (i.e., one month). Therefore, A and B are wrong. D is wrong. Dunn is not a trespasser ab initio, as his entry inside the property was with the permission of the owner.

2007-2008 Law Decks

Fropert9 Law
Toby inherits Bamboo Field from his father in 1960. The land sits vacant until 1990 when Toby decides to subdivide Bamboo Field into 20 plots for a housing development. In the center of the plots is a 50 acre parcel with rare bamboo trees destined to be "a wildlife preserve," which remains vacant. Toby prints up brochures, and sells the plots around the perimeter of the preserve at a premium, with the explanation that they have the desirable views of the wildlife preserve and rare bamboo trees. He sells 18 plots, and each deed has a restriction: "no satellite dishes may be installed." In 1999, Toby sells the remaining 2 plots, along with the undeveloped 50 acre wildlife preserve parcel to Charlie, for $1 million. In the sale, the deed restriction against installation of satellite dishes is contained. Charlie sells the 2 lots to Fitz and Zoe, respectively. He sells the 50 acre parcel to Lee. None of these deeds contain the plan for a wildlife preserve, or the deed restrictions against satellite dishes. Lee states that he is going to build a business complex on the 50 acre parcel. Ling, who owns a plot abutting the bamboo preserve parcel, brings suit to enjoin Lee. If Ling wins it is likely because: A. Lee would have been deemed to have taken the property with notice due to the location of the parcel. B. Ling can bring a suit in equity to enforce the proposed wildlife preserve plan against Lee. C. Ling can be viewed as a creditor beneficiary of Toby's promise to make the 50 acre parcel a wildlife

preserve.

D. Ling received an economic benefit from the wildlife preserve's existence in her development. Being adjacent to the 50 acres with rare bamboo trees, the wildlife preserve is an asset of her plot.

Answer: B
The only way Ling can enjoin Lee is if Ling has an equitable servitude. A is wrong as location alone is not enough to put Lee on notice. Ling can, however, enforce the restriction as an equitable servitude, as Lee had record notice in the deed from Toby. Had Lee investigated the deed properly, he would have been aware of the restrictions. C is wrong since Ling dealt directly with Toby and is, therefore, not a 3rd party beneficiary. D is not sufficient to be the basis for granting an injunction.

2007-2008 Law Decks

Fropert9 Law
Grace and Kiri inherit Virendell Nursery from their parents as joint tenants. One-quarter of the land is currently farmed and another % is an extensive propagating greenhouse. The remaining 1/2 is vacant. Kid decides that in order to make more money off the land, they should plant the remaining half with several varieties of perennials. She goes ahead and does this. Grace refuses to participate, and Kid sues seeking 1/2 the costs of the improvements that she spent on planting. What is the result of the suit? A. Kiri will be denied her request. B. Kid will be granted her request. C. The answer depends on whether Grace will receive an economic benefit. D. The result depends on whether or not the additional planting will generate enough income to pay the outlaid costs.

Answer: A
A co-tenant such as Kiri has no duty to improve the property and, following an improvement, cannot force the other co-tenant to pay for the improvements. In light of this, Answer B is wrong. C and D are wrong. The outcome of the improvements are meaningless, the rule is simply that a co-tenant cannot force other co-tenants to pay for improvements made to a piece of property.

2007-2008 Law Decks

Fropert9 Law
Assume that Hall Turner and Veata Day have lived together for 15 years in West Virginia, but were never actually married. Veata refers to herself as "Mrs. T" Over the 15 years, they lived together, filed joint tax returns and kept a joint bank account. Turner buys a house, and the deed reads "Hall Turner and his wife, Veata Day, and their heirs and assigns forever as tenants by the entirety." Hall puts down $50,000 and takes a mortgage for the remainder. Both Hall and Veata sign the mortgage as husband and wife. Hall makes all the payments until they get in a fight and Hall leaves Veata. Veata continues to make payments until she runs out of cash, and files a suit against Hall to partition the house. Assume that West Virginia does not recognize the common law marriage. Veata's partition request should be: A. Granted, since the estate generated by the deed signing was not, in fact, a tenancy by the entirety.
B. Granted, because the tenancy by the entirety was severed when Hall left.

C. Denied, a tenant by the entirety has no right to partition. D. Denied, as absolute title to the property is held by Hall.

Answer: A
A tenancy by the entirety only exists between a legally married husband and wife, making the estate, here, likely a joint tenancy with right of survivorship or a tenancy in common. Both of these types can easily be partitioned. B is wrong. If we assume this was, in fact, a tenancy by the entirety, the simple act of abandonment by one spouse does not severe the estate. C is wrong as we are not dealing with a tenancy by the entirety. Note: A tenant by the entirety cannot obtain a partition. D is wrong since the facts serve to establish that Hall cannot have absolute title.

2007-2008 Law Decks

Fropert9 Law
Ignatius conveys land: "...to Jetmin , his heirs and assigns, so long as the premises are used for a drive-in theater, then to Fergal, her heirs and assigns." As a consequence of the conveyance, Fergal's interest in the land is: A. A right of entry B. A possibility of reverter C. A fee simple absolute, since we are dealing with a violation of the rule against perpetuities. D. Nothing

Answer: B
Fergal's executory interest (as given to her by Ignatius) is void since it could possibly vest more than 21 years after a life in being, making this a rule against perpetuities violation. A court would, therefore, strike the gift to Fergal and just look to the remainder of the conveyance. C is incorrect. The use of language such as "so long as" results in the creation of a fee simple determinable and the possibility of a reverter, making Answers A and D incorrect.

2007-2008 Law Decks

Fropert9 Law
Zogby leases an apartment from Jadyn for 1 year. At the end of the year, Zogby continues to live in the apartment and pay Jadyn the usual $500/month rent. Zogby continually complains about a drain that keeps backing up, but Jadyn ignores his requests to have it fixed. Zogby gets angry and calls the board of health, who states that Jadyn must make a $10,000 repair to the plumbing. Jadyn is angry, calls Zogby and tells him that she will be increasing his rent to $1,000/month. (Other tenants in the complex with the same size apartment still only pay $500/month.) Jadyn further gives Zogby the required statutory notice that his tenancy will be terminated at the earliest possible time. Zogby contests Jadyn's right to terminate. If Zogby succeeds, it will likely be because: A. The jurisdiction has a doctrine prohibiting retaliatory eviction. B. There is a periodic tenancy created by implication. C. A landlord must generally charge the same rent for all units in an apartment complex. D. Jadyn gave no valid reason for raising the rent.

Answer: A
Here, we are clearly dealing with a retaliatory eviction, which is typically prohibited in most jurisdictions. B is wrong and counterintuitive. The fact that this is a periodic tenancy does not prevent Zogby from being evicted, but rather allows his eviction following the proper notice. C is wrong since freedom of contract allows a landlord to charge whatever she likes. D is wrong. A landlord generally does not need to establish a valid reason for raising rent.

C)2007-2008 Law Decks

rropertg Law
Sophia, a widow, owns Villa Contessa in fee simple. She is happy with the work of her gardener, Marcello, who has been the caretaker of Villa Contessa for forty years. Sophia decides to give him the land via a deed presented to Marcello on his birthday. Marcello never records the deed. A year later, Sophia realizes that she is broke and decides to sell Villa Contessa. Sophia humbly asks Marcello to destroy the deed, and he does so at her request. As they are leaving the grounds, a huge oak tree falls on their car, killing them both. If Sophia's and Marcello's estates each lay claim to Villa Contessa, whose estate will likely win?

A. Sophia's, as she was the owner of Villa Contessa following Marcello's destruction of the deed. B. Sophia's estate owns Villa Contessa since the destruction of the deed serves to eliminate the existence of the original transfer to Marcello.

C. Marcello's. His estate owns Villa Contessa since the physical deed was nothing more
than evidence of Marcello's title, and its destruction failed to cause the title to pass back to Sophia. D. Marcello's estate owns Villa Contessa, since the destruction of the deed was under the undue influence of Sophia.

Answer: C
The deed is simply a physical manifestation of title, and its literal destruction did not result in the transfer of land back to Sophia. To transfer Villa Contessa back to Sophia, Marcello would have had to draft a new deed and deliver this deed to Sophia. In light of this, Answers A and B are wrong. D is wrong since it is not supported by the facts. There are no facts indicating that Sophia used any undue influence in getting Marcello to destroy the deed.

2007-2008 Law Decks

rropert9 Law
Nick and Jake are not married, but have been living together for 2 years. They decide to buy a house. Each puts down 1/2 of the down payment, taking title as tenants in common. They promise each other that if one dies the other should take title of the house. They live in their house for 5 years, splitting all the bills directly in half. While driving to a movie, their car falls into a sinkhole. Jake is sucked underground and is never found. Nick is pulled form the sinkhole, but dies in the hospital weeks later. Neither of them had a will. Nick is survived by his mother, Lilly. Jake is survived by his brother, Bobby. Who owns the house? A. Lilly owns the entire house. B. Bobby owns the entire house. C. Lilly and Bobby each have a 1/2 interest as tenants in common.

D. Lilly and Bobby each have a % interest as joint tenants.

Answer: C
When Nick and Jake died, their 1/2 interest passed to their estates. The oral agreement is not binding, as it fails to meet the requirements of the Statute of Frauds and the Statute of Wills. Furthermore, there is no right of survivorship in the tenancy in common of Nick and Jake. Additionally, there are no simultaneous death issues at stake. In light of the correct answer, Answers A and B are clearly wrong.
D is wrong since the tenancy in common of Nick and Jake does not convert into a joint tenancy upon their death.

2007-2008 Law Decks

Fropert9 Law
Jag wants a garage built and hires Besnik. He wants the garage on the backmost region of his property right near the property line. Jag tells his neighbor, Lala, about the garage: "It's going to be 6 cars wide, so there will be plenty of room. You are more than welcome to park your car in it, too, once it is finished if you like." When Besnik starts digging for the foundation, Lala is home and takes great interest watching the workers dig. She is friendly as she stands around and watches, but makes it clear to Besnik that he and his workers may not go onto her property. After several hours of excavating, part of Lala's yard collapses into the excavated foundation. She falls into the sinkhole herself and is okay, but outraged. Assume that Lala sues for damages caused to her yard. How will the court rule? A. For Lala, with absolute liability attached to Jag. B. For Lala, only if Beznik's excavation was done in a negligent manner.

C. For Jag, since it was only land that was damaged and not Lala's house.
D. For Jag, as he had no duty to provide support for Lala's land.

Answer: A
A property owner has a right to support of his land, even if there are no buildings. If a neighbor causes collapse of that land, they would be absolutely liable. Here, Jag's contractor did just this, making Jag absolutely liable. B is wrong. Since this as an absolute liability question, negligence is meaningless. C is wrong because damage to the land yields an actionable recovery if support has been taken way. D is wrong in light of the correct answer. Neighboring landowners have a right to support even if there is no building on that particular part of the land.

2007-2008 Law Decks

Fropert9 Law
Raphael buys land in 1975 from Michael by warranty deed. Raphael does not know that Michael really doesn't own Purple Plume Hill, but that it is actually owned by in fee simple by Norman. After buying from Michael, Raphael builds a house and digs a pool on the northern tip of Purple Plume Hill. The remaining parts of the property are vacant. In 1980, Raphael builds a garage on a 10 acre plot that he thinks is part of Purple Plume Hill. In reality, these 10 acres belong to Leo. Leo lives out of state and doesn't know that a garage was built on his property. In 1990, Leo dies, having not been back to Purple Plume Hill since Raphael built the garage. Leo leaves his property to his daughter, Lisa, a 6 year old, via a will. In 2000, Raphael dies leaving all his property to David. The adverse possession period in this state is 20 years. In 2002, David learns that the 10 acres with his garage are not part of Purple Plume Hill. Raphael brings quiet title against Lisa. Who wins? A. Lisa, since Raphael never paid the property taxes on this part of land. B. Lisa, as a minor the adverse possession statute will be tolled against her until she reaches the age of majority. C. Raphael, as he had an honest belief that the 10 acres were part of Purple Plume Hill. D. Raphael, as he was in continuous possession for the adverse possession period.

Answer: D
Since Raphael meets all the requirements for adverse possession (in possession for over the period of 20 years, was open and notorious about it, etc...), he gets to keep the land. A is incorrect because only a minority of states require that the adverse possessor pay property taxes on the land in question. B is incorrect. The existence of a disability such as 6 year old Lisa's minority status, does not toll the adverse possession period. The existence of a disability must have existed at the time the adverse possession began for a tolling to occur. (For example, if Leo was mentally ill when Raphael built the garage.) C is incorrect because the majority view makes Raphael's state of mind irrelevant. Even if he knew he was a trespasser, Raphael could still gain the land under an adverse possession claim.

2007-2008 Law Decks

Fropert9 Law
Mame is affluent and in her will dated March 1, 1950 leaves everything to Patrick. Mame dies in 1991. When Patrick meets with Mame's executors, Patrick learns that 2 pieces of property were deeded to others; namely, the Rough Ride Ranch to Brigitta in 1975, and the Vino Vicarage to Rosalind in 1990. The deed to the ranch includes the clause: "for so long as the property is used to house sick parakeets. If the property is used for any other purpose, then it goes to the Able Aviary Association." The deed to Vino Vicarage includes: "as long as the vicarage is used as a headquarters in campaigning for a tax cut for the wealthy to pass by 2000. However, if the tax cut is passed before 1990, then the property should pass to Patrick at that time." Mame died leaving only one son, Fred, who was a bastard who never called his mother and so received nothing at all in Mame's will. This is a common law jurisdiction, and the state's probate laws provide that future interests, or estates in real property, may be passed by will or descent in the same manner as present or possessory interests. (Note that neither Brigitta, nor Rosalind qualify as a charitable institution.) At the time of Mame's death, Patrick's interest is describes as: A. An executory interest B. A power of termination or right of entry C. A possibility of reverter D. A contingent interest

Answer: C
Note that the Able Aviary Association interest is void as it violates the rule against perpetuities since it may vest beyond the 21 years after the death of a life in being requirement. In light of this, Mame receives a possibility of reverter in Rough Ride Ranch. An interest such as this is freely devisable and, therefore, passes to Patrick upon Mame's death. A is wrong because Patrick's interest is an interest in the grantor and is not executory. Note: A grantor cannot hold an executory interest, and an executor interest is held by someone other than the grantor. Here Patrick's interest is inherited from Mame, the grantor. B is wrong. A right of entry, as opposed to a possibility of reverter that always follows a fee simple determinable, must be expressly retained to be effective. D is wrong. The possibility of reverter is always a vested interest and never runs afoul

of the rule against perpetuities.

2007-2008 Law Decks

Fropert9 Law
Roman leases Metro Pond to Mitch for 99 years. Mitch builds a casino on Metro Pond, and sells the casino to Lorna, with Roman consenting to the assignment of the lease. Ten years into the lease, Roman conveys the property to his 2 sons, Harry and Lance. A month later, Roman has a huge fight with his sons and demands that they return the deed. The sons do so and Roman destroys the deed. Harry learns that he is dying and conveys to Tim "any interest I may have in Metro Pond." He then dies. Assume that Lorna has failed to make any rent payments, and has failed to pave the dirt road leading to the casino, which was part of the original lease agreement. Against whom does the landlord of Metro Pond have a cause of action? A. Lorna for the rent only, as a rent covenant runs with the land. B. Lorna, for both the rent and the road, assuming that Lorna assumed performance for all covenants in the original agreement.

C. Mitch and Lorna for both the rent and the road.
D. Lorna for the rent and Mitch for the road.

Answer: C
An assignment does not release the tenant from his contractual obligations to the landlord, as an assignee is in privity of estate with the landlord. Therefore, Mitch remains liable for all of the lease provisions including rent and the road. In light of the correct answer, Answer D is clearly wrong. Note: The assumption of the duties in an assignee is implied and does not need to be expressed as part of the assignment. Further note that covenants to pay money run with the land, as do covenants to perform physical acts on the property; making both Mitch and Lorna liable. This also makes Answers A and B wrong.

2007-2008 Law Decks

Fropert9 Law
Track is a railroad company that has run cargo from the city of Chicago to the city of New Orleans for the last 100 years. A bit of their rail line runs across Willie's property, based on an easement that Track received from Willie's predecessor years ago. Willie was aware of the easement when he purchased the property in 1968, but as of late Track has not been using the rail line, as that route fell into disfavor. In 1995 Willie builds numerous structures on his property for raising precious color-genetic sheep who, under the right conditions, produce extra-soft blue/gray wool. In 1999, Willie learns that Track intends to reincarnate their heavy rail project across his land, and that huge freight trains will pass through for hours on end. Willie fears that the horns and rumbling trains will upset his sheep and, as a result, the stress will cause them to produce scratchy grey wool. He brings action to enjoin Track trains from entering his property. In this jurisdiction, the statutory prescriptive period is 10 years. How should the court rule? A. For Willie, as Track abandoned the easement. B. For Willie, as Track's use of the easement would unreasonably interfere with his use of his land for raising sensitive color-genetic sheep. C. For Track, as they still have possession of the easement. D. For Track, since Willie has failed to use the land governed by the easement and cannot assure that the easement has been abandoned.

Answer: C
Easements are generally viewed as being of perpetual duration, and cannot be abandoned by simply not using them. To abandon, Track would need to have showed an intention to permanently abandon the easement; such as removing the train tracks. In light of this, Answer A is clearly incorrect. B is wrong for stating the wrong result, as the facts in this answer have nothing to do with abandonment. Why would you choose an answer having to do with abandonment? The question is not specific to that. D is a misstatement of law. Once an easement is abandoned, an easement cannot be revived solely based on the fact that the owner of the underlying fee failed to take an affirmative action.

2007-2008 Law Decks

Fropert9 Law
Luna partners with Lupe to develop Wolf Moon Estates in 1990. In 1999 they acquire all of the required permits and begin to build Wolf Moon's residential development. The first portion is comprised of 100 houses on 250 acres of land. The second portion is a restaurant complex built on an adjoining 50 acres of land. Which is the best method by which a court will enforce the residential character of the Wolf Moon Estates? A. Each deed should contain a negative easement. B. Each deed should contain a condition subsequent. C. All deeds should include covenants. D. Develop the area in accordance with all applicable residential zoning laws.

Answer: C
The best approach is Answer C, as the use of a negative easement in accordance with Answer A is too limited. , B is a possibility, but when face with a court-appointed forfeiture of an owner's estate, the court may not go forward with such a plan; especially in light of the fact that covenants in the deeds would accomplish the same end result. D is wrong because the applicable zoning laws may, in fact, change.

2007-2008 Law Decks

Fropert9 Law
Luis owns Shin Mountain and in 1970 decides to subdivide it into 20 lots of 1 acre each. He keeps Lot 1 and sells Lots 2-20 with the deed restriction stating: "All lots will be used for single family residential construction." Luis builds a house, as do owners of Lots 220. Luis lives in his home on Lot 1 until his death, at which time it is sold without any restrictions to Al. In 2001, Al dies and Lot 1 is sold to Henna without any deed restriction. In 2002, Henna decides to turn her house on Lot 1 into an adult video store and bondage emporium. Tasha tells Henna that she cannot do this and mentions the original deed restriction to her. Assume that the restriction is applicable to Lot 1 (which Henna now owns). It will be applicable because the restriction is: A. An equitable servitude B. A covenant running with the land C. A reciprocal negative servitude D. Part of a general plan

Answer: C
In a development and subdivision setting as applicable here, where some pieces of property have deed restrictions and others do not, an implied negative covenant or equitable servitude is applicable under the theory of reciprocal negative servitudes. Answer A is almost a correct answer, but not as thorough as Answer C. Answer B is wrong as a covenant requires a showing of privity of estate. Furthermore, the applicable remedy for a covenant is damages, not an injunction. D is incorrect, as it is an incomplete answer.

2007-2008 Law Decks

Fropert9 Law
Leroux owns Lorien Gardens in fee simple and in 1970 converts the land "to Jamal for life and to the first child of Jamal's brother Kurt, who shall reach the tender age of 21 years." At the time of the conveyance in 1970, Kurt is unmarried. In 1976, Kurt marries and in 1978 Kurt has a daughter, Loriel. In 1999 Loriel reaches the age of 21. Jamal is still alive and living at Lorien Gardens. Loriel seeks to obtain possession of Lorien Gardens and seeks to eject Jamal. Jamal's best defense is: A. Jamal has a life estate that was not subject to termination during his life. B. Loriel has no claim to Lorien Gardens since she was not living at the time of the original conveyance from Leroux to Jamal. C. The conveyance to Loriel violates the rule against perpetuities.
D. At best, Loriel has a contingent remainder, contingent upon the death of Jamal.

Answer: A
The conveyance at issue clearly creates a life estate in Jamal. Therefore, Jamal maintains possession for the duration of his life. B is wrong since being alive at the time of the conveyance is not necessary for Loriel. C is incorrect. Since Loriel's interest vests within 21 years of a life in being (namely, her father, Kurt), it is a valid interest under the rule against perpetuities. D is incorrect because Loriel's interest is a vested remainder.

2007-2008 Law Decks

Fropert9 Law
Suppose that Florida has the following recording statute: "Any conveyance of an interest in land is not valid against subsequent purchasers without notice thereof who first records." Grant owns Citrus Harbor and desperately needs money for his divorce from Sasha. Grant goes to WaterBank and takes out a loan against Citrus Harbor for $100,000 to pay off Sasha. He does not mention his divorce to WaterBank. The official papers are signed recording the mortgage on January 1st. They are passed to Jiri, a WaterBank clerk, on January 2nd to be taken to the Florida recorder's office. Jiri has done this numerous times in the past, but somehow manages to misplace one of the papers. On February 10th Jiri finds the paper stuck in the copier and finally records the mortgage. While there, Jiri learns of a subsequent conveyance from Grant to Lamar filed on February 1st and recorded on February 2nd. This conveyance was for $200,000 and Lamar had no reason to believe that the property had any liens against it, as at the time WaterBank's interest had not yet been recorded. WaterBank seeks a court declaration stating that Lamar owns Citrus Harbor subject to the $100,000 mortgage. Will this be upheld? A. Yes, WaterBank had an interest that was acquired at a date prior to that of Lamar. B. Yes, the mortgage was simply security for a loan. C. No, because Lamar has recorded first. D. No, unless Florida is, in fact, a title theory state.

Answer: C
Lamar has recorded and, as set forth in the Florida statute, he has recorded first without knowledge of WaterBank's mortgage and is a bona fide purchaser. Here, we have a race-notice statute, under which a subsequent purchaser for value without notice of any prior conveyance is protected if he records before the prior grantee. Here, Lamar satisfied that statute and holds Citrus Harbor free of WaterBank's interest. A is incorrect because, to prevail, the statute here states that WaterBank must have actually recorded prior to Lamar. The date at which they acquired an interest in the property is meaningless. Under Florida's statute, it is the date at which the record is made that counts. B is incorrect. While the mortgage is, in fact, a security interest for the loan; it is also an instrument creating an interest in Citrus Harbor held by WaterBank and falls within the scope of Florida's recording statute. D is incorrect for offering a red herring type answer. Supposing for this question that Florida is a lien theory or a title theory state is irrelevant. Under both theories, WaterBank's mortgage is creating an interest in Citrus Harbor, and is, thus, subject to the recording statute.
02007-2008 Law Decks

Property Law
Gaston and Teresa Curtis are a married couple looking for an expensive home in Mendham. The required down payment is astronomical, and Gaston's mother, Vida, offers to lend the young couple some initial cash. Gaston offers to put her name on the title until they repay the debt and Vida agrees. They buy the house and the title reads: "Gaston and Teresa Curtis, and Vida Curtis as joint tenants with right of survivorship." Vida pays half of all expenses for the house over the years. Teresa dies unexpectedly young and Teresa's will reads: "All my interest in real property to my son from my first marriage." The question becomes, in probate, what interest of the house shall pass to Teresa's son. Gaston offers to prove that Vida's interest was only placed in the title as a security interest. How should the court rule regarding title to the property? A. The property belongs to Gaston as sole tenant. B. The property belongs to Gaston and Vida as joint tenants.

C. The property belongs to Gaston and Vida as tenants in common.


D. The property belongs to Gaston, Vida and Teresa's son as tenants in common.

Answer: B
A joint tenancy carries the right of survivorship. Therefore, when a tenant dies the property held by this tenant is subsequently divided among the remaining tenants, making Gaston and Vida each owning 1/2 shares. A is incorrect. As stated above, Vida does, in fact, have an interest. This answer could be correct if Gaston could prove that Vida merely had a security interest. However, the fact that she paid upkeep and bills tends to indicate that her interest was more than a simple security interest. C is incorrect since the joint tenancy was not severed. A testamentary conveyance such as this does not sever the joint tenancy. If there were only 2 parties, however, a conveyance by one would, in fact, sever the joint tenancy resulting in a tenancy in common. (But that is not the case in this present set of facts.) D is incorrect. A will has no effect on a joint tenancy since once the party dies their

interest in the property disappears and is subdivided to the remaining tenancy (Gaston and Vida). Therefore, Teresa's son has no interest to take.

2007-2008 Law Decks

Fropert9 Law
20 years ago Pow-Pow Power Company built a dam across a river. At the time, Pow-Pow procured easements from all of the local land owners with property abutting the river by paying fair compensation for the river land. Most were large estates with over 100 acres, but Tierre owned a small farm with only 10 acres. When he signed the easement he said: "I really don't want to do this, but I'm being strongarmed by the large estate owners." The easement allows Pow-Pow Power to flood the land if necessary to control the river and produce electricity. In the 20 years since signing the easement, Tierre has never had his small property flooded. Yesterday, all the easement owners were informed by registered letter that their lands will be flooded, including Tierre. Tierre consults with an attorney, fearing the loss of his livelihood. Tierre's attorney realizes that Tierre has made no huge improvements to his land; but while investigating the easements, he realizes that all of the easements but Tierre's were properly recorded. Tierre's easement lacked acknowledgement by power. Tierre's attorney calls PowPow Power and states, "Sorry, the easement on Tierre's land is no good. Better not flood a thing." Can Pow-Pow Power flood Tierre's land? A. Yes, Pow-Pow Power does have a proper easement and flooding is permissible under the easement. B. No. Since Pow-Pow Power has failed to exercise its rights for years, they have lost use of the easement. C. No, because the easement was not properly acknowledged and recorded. D. No, unless the state has an adverse possession statute that requires hostile occupation for a period longer than 30 years.

Answer: A Pow-Pow Power Company has a valid easement, as an easement by express grant requires that there is a writing signed by the grantor. If validly created, an easement lasts for a perpetual period of time, and all of these requirements were met by the present set of facts. B is incorrect because mere non-use will not terminate an easement. To terminate an easement, the non-use must rise to the level of abandonment, wherein the easement holder makes some physical act that manifests their intended abandonment. C is incorrect because the improper recording of Tierre's easement does not extinguish the right of the original parties to the easement. Failure to record properly, at best, results in lack of constructive notice to subsequent parties, but it does not effect the rights of the original parties to the easement.

not met in the present fact pattern. To extinguish an easement by prescription, the owner

D is incorrect as the requirements for extinguishing an easement by adverse use, are

of the servient tenement must interfere in a manner that creates a cause of action in favor of the easement holder, wherein the interference is open, notorious, continuous, and non-permissive for the prescriptive period. Here, Tierre has done nothing that would result in the termination of the easement.
2007-2008 Law Decks

Fropertg Law
Jung owns land in fee simple. In 1990 she devises land to Ty for life with a written instrument that states: "To Ty for life; if Ed and Ned return home from looking for the Northwest Passage prior to Ty's death, to Ed and Ned and their heirs. If they do not return home before Ty dies then the remainder is to go to my heirs." In 2004 Jung and Ty both die in a tragic smoking accident. Ty's will left the estate to a pro-smoking club, and Ty's remaining relative is his brother, Jake. Assume that we are in a jurisdiction that has codified common law principles. The status of Ned's and Ed's interests, at the time the deed was executed, are best described as: A. Mere expectancies B. Contingent remainders C. Absolutely vested remainders

D. Vested remainders subject to a condition subsequent

Answer: B Here, we have a condition precedent to Ed and Ned taking possession. Thus, their interests are contingent rather than vested remainders and that makes Answers C and D incorrect. Note, however, that the original deed constituted a proper conveyance of a future interest, thereby resulting in something more in Ed and Ned than a mere expectancy. This makes Answer A incorrect.

.02007-2008 Law Decks

Frope rt 9 Law
Enrique and Wendy are a married couple who own Casa Riverside as a tenancy by the entirety. Enrique decides to transfer his interest to Paul by a quitclaim deed. When Enrique does this, Wendy is away visiting her mother. When she gets back, she decides to mortgage her interest in Casa Riverside to Sue. What is Sue's interest? A. A lien against the entire property. B. A secured interest against Wendy's 1/2 interest as a tenant in the entirety with Paul. C. A secured interest against Wendy's 1/2 interest as a tenant in the entirety with Enrique. D. No interest

Answer: D
Sue has no interest, as this is a tenancy in the entirety and both spouses must join together to get a mortgage on the property. In light of this, Wendy has no power to grant a mortgage against the property to Sue. A is wrong in light of the correct answer. This answer is also wrong because it is impossible for Wendy to convey a greater interest than the one she holds (namely, a 1/2 interest in the property). B is wrong. It is impermissible for Wendy to mortgage the property absent Enrique's consent, just as it is impermissible for Enrique to convey the property absent Wendy's consent. Additionally, a tenancy by the entirety is only applicable to a married couple and we have no facts indicating that Paul and Sue are married; making this answer wrong for yet another reason.

consent. Furthermore, it is impossible to hold the property as a tenancy by the entirety


between Enrique and Sue, as they are not married.

C is wrong since it is impossible for Wendy to convey her interest without Enrique's

2007-2008 Law Decks

Fropert9 Law
Edie and Simon own adjoining land. Edie's property is west of Simon's. Adjoining Simon's property on the other side to the east is Hot Spring Caves. In 1980 Edie buys inflatable rubber duckies for her children to use while soaking in the hot springs. She asks Simon if it would be okay for her and her children to walk across a 10 foot expanse of his property to gain access to the hot springs. Simon knows that the only other access is a circuitous 30 minute car ride over 15 miles, so he agrees to let Edie and her kids use the 10 feet of his property to get to the springs. Edie and her family use this strip for 19 years, at which point Simon sells his property to Art. Art sees Edie walking the strip every Monday, but says nothing, knowing that this is, in fact, his property. On the 9th month of noticing this, Art speaks up and prohibits Edie from using the strip. Edie and her family continue to use the land, regardless of the fight, and continue for 5 more years, at which point Art gets an injunction. The adverse possession period is 20 years. When Simon gave Edie permission to use the 10 foot strip to cross to the hot springs, Edie's interest could best be described as: A. An easement in gross

B. An easement appurtenant
C. An easement by necessity D. Not an easement

Answer: D
Here, Edie has nothing more than a "license" to use the land. We do not have a written agreement. Therefore, the Statute of Frauds requirements for the creation of an express easement were not met. Additionally, because this is permitted use by Simon, Edie's use of the land was not hostile. This means that the adverse requirements of adverse possession were not met and, therefore, there is no easement by prescription. In light of this Answers A, B and C are wrong.

2007-2008 Law Decks

Froperty Law
Buck owns a 200 acre plot of land. One hundred acres were given to him by his father upon his father's death. The other 100 were acquired from Chin through adverse possession. On January 1st, Buck enters into an agreement to convey the entire 200 acres to Roberto, a builder, for a housing community. Closing is to be April 30th and the property description is clearly set forth in the agreement signed by both parties. The nature of the title, however, is not expressed in the agreement. On April 30th, at the closing, Roberto conveys the cash and Buck conveys the deed to Roberto. Six months later, Chin comes along and sues Roberto, seeking to eject him from his 100 acres. Roberto in turn sues Buck. Which of the following best describes Roberto's rights against both parties? A. Roberto's rights are based on an implied covenant contained in a marketable title. B. Roberto could bring an action for reformation of the deed with a price adjustment.

C. The terms of deed control the liability of Buck.


D. Roberto can sue Buck for fraud.

Answer: C The terms of the deed control. Here, the contract did not express the type of deed to be conveyed, so one must imply that a marketable title is to be conveyed. (Why else would one buy a plot of land?) Note: A title based on adverse possession is not marketable. Therefore, Roberto can refuse to perform. However, once Roberto accepted the deed, the doctrine of merger states that the contract is merged into the deed. Any contract provisions for quality of title which may have been either expressed or implied, therefore, are no longer effective. In light of this, Answer A is clearly incorrect and Roberto must look to the terms of the deed to understand his rights. B is incorrect because a court will never step in and actually rewrite a deed. Answer D is incorrect as there are no facts to indicate that Buck has made any type of misrepresentation.

2007-2008 Law Decks

Fropert9 Law
On tax day (April 15th), Caesar and Brutus contract in writing for Brutus to buy Caesar's house for $100,000. Closing is scheduled for 1 month later (May 15th) at Caesar's attorney's house, and Caesar will deliver marketable title, free and clear of all encumbrances. On May 15th Caesar offers up his deed but Brutus refuses, having learned that Marcus, an interior decorator, has a lien against the property based on an outstanding bill. The lien is for $10,000. Caesar swears that he has never heard of the lien, but states that he will set aside money from the sale to cover the lien to ensure that that matter is resolved. Brutus still refuses. If Caesar sues Brutus for specific performance, what is the likely result? A. Brutus wins, as the title to the property was not marketable as of the date of closing. B. Brutus wins, since there is an encumbrance on the title (which existed upon the date of proposed closing between Caesar and Brutus) that remained subject to a possible litigation.

C. Caesar wins, as the Doctrine of Equitable Conversion states that Brutus was the owner of the property when the lien was placed against it and, therefore, the lien was improper.
D. Caesar prevails, because a contract such as this has an implied term that Caesar can use the proceeds of the sale to cover any outstanding liens or encumbrances.

Answer: D
In a contract for the sale of real property, it is assumed that the seller of the land can use the proceeds of the sale to clear title, as long as he can ensure that the purchaser of the property will be protected. Here, Caesar was willing to do just this, making Answer A incorrect. B is incorrect. Since the potential litigation attached to the property was only for monetary damages, it did not have an effect on the title, as there was no outstanding claim that could affect the title to the land. Therefore, Brutus's interest in the land cannot be harmed. C is incorrect for misapplying the Doctrine of Equitable Conversion. Equitable conversion is only applicable to a seller and buyer. It cannot affect the rights of a 3rd party when the 3rd party has attached property due to some debt outstanding.

2007-2008 Law Decks

Property Law
Mary and Ann have lived together for 20 years. On their 21st year, Mary has a heart attack and Ann continues to live with Mary and look after her. Mary tells Ann that she wants to take care of Ann if she dies, and deeds the house to Ann via quitclaim deed. Ann does not record, but puts the deed under her mattress. Two weeks later, Denise, Mary's daughter, learns of this deed transfer to Ann. Denise offers to buy the house from her mother and allow her to live there until her death. She offers to pay 20% over market value for this. Mary tells Ann of her change in mind and that she wishes to leave the house to her daughter. Ann agrees to destroy the deed. Two days later, Ann burns the deed in the trash basket. On their way to the doctor's office, Mary and Ann are run over by a stray monster truck performing at the local fairgrounds. Both Mary's and Ann's estates sue seeking claims to the house. A court will likely find: A. Ann owns the house, since Mary failed to disclose why she was revoking the house. B. Ann remains the owner, as Ann has yet to transfer the house back to Mary. C. Mary is the owner, as Ann agreed to return the title by destroying the deed and, in fact, did just that when she burned it. D. Mary is the owner. Ann is nothing more than a donee who has a lesser degree of rights as compared to Denise, the bona fide purchaser.

Answer: B
A deed is nothing more than an evidentiary marker that a person holds title to a piece of property. Destruction of the deed, therefore, has no effect on actual title. Here, Ann then retains title in the property even after burning the deed. Ann is, therefore, the true owner, which in turn makes Answer C incorrect.
A is wrong. Once Mary transferred the deed to Ann, she could no longer make any

changes to the ownership of the land as she no longer owned it. In light of this, Mary could not subsequently sell to Denise, regardless of the truth or misstatement of the reason she gave Ann for revoking the deed.
D is incorrect since Denise is not a bona fide purchaser, as Denise knew of the deed to

Mary. A bona fide purchaser is a purchaser that buys without notice and for value. While Denise clearly paid a "value," she fails to meet the notice requirement, as she clearly was aware of the deed to Ann.

2007-2008 Law Decks

Fropert9 Law
Sonny rents a house from Darius for a long-term lease spanning 5 years. After 5 troublefree years, it is time to renew the lease. Sonny is an ideal tenant paying $500/month, but market value is now $1,000/month. Darius still elects to keep the price at $500, as Sonny is a great tenant. The new lease is for 4 years and has an express clause stating that no assignment of the lease shall occur without Darius's approval. When Sonny gets married, he moves in with his rich older wife, and sublets the house to Ling for $750/month without Darius's permission. Assume that Darius brings an action to either reject Ling, or recover damages from Sonny. What will likely happen? A. Darius will be able to recover damages and eject Ling. B. Darius will be able to eject Ling only, because he has suffered no monetary damages. C. Darius will not be able to eject Ling, because Sonny had no right to sublet under the lease agreement he entered into as he did not get Darius' approval. Darius can recover the full rent paid by Ling, since allowing Sonny to benefit form the sublease is improper. D. Darius has no cause of action for either ejectment or damages.

Answer: D
Here, we are dealing with restraint on alienation. Restraints on alienation are traditionally strictly construed. Here, this prohibition against assignment will not include a prohibition against subletting. In light of this, Darius has no cause of action against Sonny, making Answers A, B and C incorrect.

2007-2008 Law Decks

Fropert9 Law
Wilfred is old and in a nursing home. He asks his attorney, Jennie, to draft a deed giving his high-rise apartment building in Manhattan to his son Terence. Jennie drafts the deed, it is executed by Wilfred and recorded. Jennie also notifies Terence, figuring that he will be ecstatic. Terence promptly rushes to the nursing home, grabs his father and says, "I'm no landlord. I don't know a thing about collecting rent. I hate math and I hate New York. Take the damn building back. I don't want it. I just want to go back to my log cabin in the woods." On his drive back up to his cabin Terence is blindsided by a coffee truck and killed, leaving Trish as his only heir. Wilfred seeks to bring an action of quiet title against Trish for the building. Assume that Wilfred is successful. What is the likely reason? A. Terence's statement was a constructive re-conveyance. B. The presumption of delivery arising from Jennie recording was meaningless, as Terence had no knowledge of it. C. Trish is subject to a constructive trust to carry out the intent of Terence. D. Terence never accepted delivery of the deed.

Answer: D The only way that Wilfred can prevail is upon a showing that Terence never accepted delivery of the deed. A is a bogus rule of law, as there is no such thing as a "constructive re-conveyance" of land. It sounds good, but is actually a totally fake answer. B is a close choice, but recall that this is the minority rule of law. The majority rule would be that acceptance is presumed so long as the conveyance is beneficial to the person on the receiving end. Knowledge in the grantee is not necessary. C is wrong. A constructive trust is not appropriate, as there is no indication that Terence or Trish were guilty of wrongful actions. A constructive trust is, therefore, an inappropriate mechanism. D is the best answer. Even in those states that presume acceptance, the presumption is rebutted when the grantee expressly refuses to accept the conveyance.

2007-2008 Law Decks

Fropert9 Law
Antoinette owns 2 homes on a plot of land that spans 10 acres. She enters into the following agreement with Samir, "I sell one house to Samir for $1 million, with enough land attached to build a garage and sauna. Received from Samir is a $1,000 deposit. Antoinette DeSanto, March 1st." On March 15th, Samir enters into an agreement with Marie to resell the house to Marie for $1.5 million. After buying the house, Samir believes that he can get more than $1.5 million for the property and decides not to sell to Marie. Further assume that Marie seeks specific performance. How will a court hold? A. Judgment for Marie, since she had no idea that Samir's purchase from Antoinette was below market value. B. Judgment for Marie, since Samir had a marketable title at the time Marie sought to enforce the contract for the sale of $1.5 million. C. Judgment for Samir, since he had no title to the house at the time he sought to sell to Marie. D. Judgment for Samir. His agreement with Marie was fraudulent at the time he entered into it in regards to Antoinette, and is, therefore, unenforceable.

Answer: B It is essential to realize that the agreement between Samir and Marie has an implied condition that Samir will deliver marketable title to Marie at the time of closing. It is not essential that Samir actually has a marketable title at the time that he enters into the contract with Marie. Answer C is wrong for this reason. A is a totally irrelevant answer that is not applicable and clearly wrong. D is wrong. It is perfectly permissible to go ahead and buy property for the purpose of resale.

2007-2008 Law Decks

Fropert9 Law
Nina inherits a huge chunk of property from her late husband, Ho. It totals 100 acres. Half the land is a forest, and the other 1/2 is a diamond mine. Ho ran the mine up until his death. He often thought about selling firewood from the trees he cut down, but never got around to it. Two years after his death, Nina transfers the land to their daughter, Jen, for life, the remainder to go to a national wildlife society. Jen begins to mine diamonds and cut trees to sell for firewood. The national wildlife society sues on a theory of waste, seeking to stop Jen from doing both activities. How will the court most likely rule? A. The national wildlife society can stop both activities. B. The national wildlife society cannot stop any activity. C. The national wildlife society can prevent the tree cutting, but Jen is free to mine diamonds.

D. The national wildlife society can stop the diamond mine, but not the tree cutting
firewood sales.

Answer: C
A life tenant is generally not allowed to exploit natural resources on a property. There is one exception, namely the "open mine" exception that is applicable to the present facts, wherein the life estate holder can continue with actions that occurred at the time of the grant. Here, the mine previously existed, and Jen will likely be allowed to continue mining diamonds as the grantor likely intended this activity to go forward. The firewood sale of cut trees, however, was not occurring at the time of the grant and the "open mine" exception is not applicable. In light of this, Jen cannot cut trees, making Answer C the correct choice, and Answers A, B and D incorrect.

2007-2008 Law Decks

Fropert9 Law
Bradley owns Buckland Shore. He builds his home on the northern parcel and develops a pear orchard on the southern parcel. There is a country road leading to his home on the north part of the northern parcel of land. Bradley extends this road from the northern parcel down to his southern part of land. He additionally runs above ground power lines on poles across the southern portion to get power to his house up north. After years of raising pears, Bradley gets pesticide poisoning. Just before dying, he deeds the northern part to Gary and the southern part to Carla. The deed to Gary reads, "To Gary for life and then to his widow for her life, remainder to Gaty's children then alive." The deed to Carla is simply, "To Carla, Enjoy raising pears and don't drink the pesticide." Carla finds no recorded document allowing an easement for the power lines across her orchard to the house on the northern parcel and takes them down one afternoon. Which would the most beneficial fact to help Gary? A. Carla knew about the land and the power lines when she accepted the deed from Bradley. B. The only other available access to power for Gary is going to cost 1,000 times what he currently pays and is difficult to get.

C. Carla originally told Bradley that she was "totally okay with the power" lines spanning across the
southern property. D. Carla is acting out of spite, as Gary accidentally blocked the road to Carla's orchard one day when he left his truck in the driveway sticking out too far. She lacks a good faith belief that she has a right to remove the line.

Answer: B
The best set of facts for Gary is that the only other power access is both expensive and difficult to get. This helps to prove that there is, in fact, an easement implied by operation of law ("quasi-easement"). Such an easement can be implied if prior to the time the tract is divided, a use exists on the "servient part" that is reasonably necessary for the enjoyment of the "dominant part." Additionally, a court must further determine that the parties intended the use to continue after division of the property. A party must show that the use was apparent and continuous at the time the tract was divided. Gary can, in fact, prove these conditions (the house has and will continue to need electrical power) of an implied easement for a use such as this, making Answer B the correct answer. A is wrong. Actual knowledge of the line is irrelevant. Carla does not need to be aware of the use, it is only necessary to show that the use is apparent. Here, we have above ground power lines on poles, that could easily be discovered should Carla look up, making the power lines clearly apparent. C is wrong. An oral statement made to the grantor at the time of grant is meaningless in

view of the implied easement set forth in the correct answer.


D is wrong because motive is also irrelevant. If there is no easement, Carla is free to remove the lines for any reason she likes. If there is an easement, however, Carla cannot remove the lines no matter how good of a reason exists.
2007-2008 Law Decks

Fropert9 Law
Pasquale enters into an option agreement to buy Philip's house and land. Pasquale is to pay a fixed monthly rent, plus taxes and maintenance charges for 5 years. This option can be exercised at any point during that 5 year term by giving Philip 30 days notice. Three years into the lease, Philip dies, leaving his estate to Rick, his brother. Just before Philip's death, Pasquale assigned his interest in land to Ashley using a written agreement and set forth the option terms. Lynn fully performs under the lease. Can Lynn exercise the option to purchase? A. Yes, because both the burden and benefit of the covenant to convey run with the land. B. No, because the covenant to convey between Pasquale and Philip does not touch or concern the land. C. No, because the option to purchase was personal to Pasquale and Pasquale alone.

D. No, because the burden of the covenant to convey does not run with the land.

Answer: A
A covenant to convey, both touches and concerns the leasehold and runs with any future interest in the land, such as Ashley's interest. In light of this, Answer B is clearly incorrect. C is incorrect. The option to purchase, as set forth in the question, has no additional information noting that the option is personal in nature. D is incorrect because the burden of the covenant to convey does, in fact, run with the land.

2007-2008 Law Decks

Property Law
Suppose that Higuchi owns a 100 acre farm in Kansas City. Suppose that Kansas City annexes his land to build a residential shopping complex, and rezones it from farming to residential. This project is spearheaded by Hattie, a builder who buys the 100 acres from Higuchi on behalf of the city for a fair price. She files a subdivision plan to build 200 homes and 2 "city parks" and begins selling lots like hotcakes. In each deed of transfer is a covenant, restrictions and conditions relating to the nature of homes in relation to the subdivision. One covenant is that there is to be no commercial or industrial activity. Hattie sells all but 10 lots and one of the designated parks. All of her sales include the covenants. The remaining 10 lots are then sold to CoCompany, without any of the deed restrictions. They resell the 10 lots (absent deed restrictions) and the remaining designated park to Clear & Free, a nuclear power company. The final sale to Clear & Free was made 2 years after Kansas City's purchase of land from Higuchi. Clear & Free is about to start trucking in uranium. Suppose that until now, Kansas City has done nothing with the park. Chase, an owner of one of the lots, brings action against Clear & Free nuclear, asserting that a power plant violates the "no commercial or industrial activity" covenants of the subdivision. Should the injunction be issued? A. Yes, Clear & Free is not a bona fide purchaser.
B. Yes, Chase is a creditor 3rd party beneficiary of the promise made by Hattie with respect to plots within

the subdivision.
C. Yes, Chase has an equitable servitude concerning use of land within the subdivisions. D. No, because the covenant banning commercial activity is only applicable to purchasers of individual lots, not the 10 lot block sold off at the end of Hattie's dealings.

Answer: C
Chase holds an equitable servitude, namely a right to enforce the covenant in equity. Note that Chase is NOT a 3rd party beneficiary, but rather a direct promisee. In light of the correct answer, Answer B is clearly wrong. It does not matter that Clear & Free was a bona fide purchaser, since they would be bound by the covenant because of its presence in public records. This also makes Answer A incorrect. Answer D is further wrong, as the original covenant was not limited to any one type of lot (i.e., a residential home lot) and is applicable to all lots within the development.

2007-2008 Law Decks

Fropert9 Law
Quiterie owns Glass House Cliff, and tells, Colette, her favorite cousin, "You've loved me like a sister. When I pass on to that great terrarium in the sky, Glass House Cliff is all yours." Colette has an outstanding debt due to Jung, and to repay him decides to deliver a general warranty deed of Glass House Cliff. Jung records the deed and moves in to Glass House Cliff. Quiterie then dies. In her will Glass House Cliff goes to Colette. Colette is again broke, so she sells Glass House Cliff to Lloyd by general warranty deed. Lloyd records instantly, and pays Colette a fair price for the property. Lloyd never moves in to Glass House Cliff as he saw it as an investment property. He has no idea that the property has an actual glass house on it which Jung is living in. With regards to status of the title at the time that Quiterie dies, which answer best sets forth the true nature of the title? A. Jung has title to property.
B. Jung has title as long as Quiterie knew of the transfer from Colette.

C. Colette has title, but Jung is free to sue to recover under the theory of estoppel. D. Colette has title. Jung has no claim that is enforceable

Answer: A Under the estoppel by deed theory, most states would find that the title passes to Jung under operation of law. B is wrong because estoppel by deed is applicable, regardless of whether Quiterie knew of the transfer or not. C is wrong for misstating the minority position in regards to the estoppel by deed theory. Under the minority law Jung would have to bring suit to recover title, as compared to the majority view where title passes automatically. D is wrong in light of the passing of title by operation of estoppel by deed, as outlined in the correct answer.

2007-2008 Law Decks

Property Law
Van owns Roadside Estate, which butts against the road. Vin owns PiperGate Glen, an undeveloped lot that does not have road access but butts against Roadside Estate. The easiest access to PiperGate is across Roadside. Van grants Vin an easement to "pass though," allowing Vin to drive along one side of Van's property to get to PiperGate. However, when Vin seeks to have electrical cables run underground along that same area of Van's property, Van refuses. The electric company refuses to run the cable across Van's property (even in light of eminent domain) without Van's approval. Does the electric company need Van's permission to go forward with the underground installation of the electrical cables? A. Yes, since there is no easement allowing them to do so without permission. B. Yes, unless they are willing to post a bond against the possibility of any damage that they may do to Van's property.

C. No, they have the power of eminent domain.


D. No, so long as there is no other means or route that they could take for the power lines.

Answer: C
The electric company can install lines using its power of eminent domain. It is only because of this power that Vin and the electric company may use Roadside to lay underground electrical cable to PiperGlen.
A is incorrect for failing to take into account the electric company's right to eminent

domain. If that right did not exist, however, this answer would be good (and a correct choice) given the facts. B is incorrect since the bond issue is meaningless. It is only because of the electric company's power under eminent domain that the may use the land Roadside to run lines. D is incorrect since an easement of necessity and an easement of way are both inapplicable. Additionally, these types of easements only occur when a single tract of land is parceled into 2 or more. Here, we have no facts indicating that this was ever a

single piece of land that was parceled off.

2007-2008 Law Decks

Fropert9 Law
Felix builds Cara Mia Estates, and includes the following in each lot deed, "Grantee agrees for himself/herself and assigns to only use this property for a single-family home construction, and to pay the monthly service charges to Cara Mia Estates for upkeep and security. Grantee also agrees not to put up any fences so that the indigenous cats may wonder the grounds as they please. Grantee will also agree never to paint a house 'Bright Orange' since it is distracting." Felix sells lots to Minx, Luis and Nan, and each deed is recorded. Minx resells to Mike, but this deed has no language noting the restrictions and Mike has no notice of them. Mike then starts selling junk scrap metal out of his garage and keeps it on the side of his house. Luis resells to Kendra, who is aware of the monthly fees, but refuses to pay based on the suggestions of a ghost she claims to have met in her new house. Nan resells to Cindy who builds a 12 foot cinder block wall around her property and digs a mote, unaware of the fence restriction. Answer based on common law requirements. Assume that Felix still has unsold lots, and sues Mike. Which answer best describes the covenant restricting lots to single-family homes?

I. Enforceable, even though there was no contractual agreement between Felix and Mike, as this is a covenant running with the land. II. Enforceable, as Mike has constructive notice when he bought from Minx. III. Enforceable, even if it is contained in a deed signed only by the grantor developer and not by

grantee Mike.
A. B. C. D. II only I and Ill only I only I, II, and III

Answer: D
All of the statements are correct. The single-family covenant runs with the land, thereby binding Mike as if he himself had agreed to it. The original granting language "grantee and his assign" indicates that Felix and Minx intended all further parties to be bound. Mike also had constructive knowledge of the covenant, as the original deed between Felix and Minx was recorded and this deed was in Mike's chain of deeds had he investigated. It is necessary to evaluate whether this covenant runs with the land using the following analysis: horizontal privity existed between Minx and Felix as they both shared interest in the land independent of the covenant. Mike further holds the entire interest originally held by Minx; thus, there is vertical privity. Finally, the covenant touches and concerns the land since it restricts the holder of the servient estate by limiting the means by which that holder can use the land (single-family home construction only). In light of this, all requirements for the covenant to run have been met. Consequently, Statement I is a correct statement and Answer A is incorrect.
Statement II is correct since there is an equitable servitude and Mike had constructive notice. An equitable servitude is a covenant that equity will enforce against assignees of the burdened land who have notice of the covenant. Equitable servitudes are only created by mechanism of a covenant that is in writing and that is enforceable in a court of equity. For the burden to run with the land, the covenanting parties (Felix and Minx) must have intended the servitude be enforceable by and against assignees. The covenant language indicates that Felix and Minx did intend this covenant to run with the land. Since

Mike clearly has constructive notice of the fact that the land is to be used for single-family home use
only via the chain of deeds, as required in order for the covenant to run; we clearly have a covenant that touches and runs with the land. Thus, Answers B and C are incorrect.
Statement III is also correct since acceptance of a deed signed by the grantor alone is enough to bind the land to an equitable servitude or covenant.
2007-2008 Law Decks

Fropert9 Law
Rufus owns a small business selling ice in Alaska. He is quickly loosing all of his cash with no customers or sales. As the planet slowly warms, Rufus's business is slowly getting better, but he needs cash to stock his freezer with inventory. He goes to Mia for a loan. She agrees to the loan as long as Rufus puts his house and property up as collateral. Rufus's property was inherited from a rich uncle and is worth $1 million. Rufus gives Mia a deed and she promptly records. Mia gives Rufus $500,000 and a promissory note for the remaining $500,000 within 6 months. Rufus continues to live on the property and Mia agrees to re-convey the property back to Rufus upon repayment of the $500,000 advance. Rufus continues to have no business luck and within 6 months his ice business is going broke. He asks for an extension but Mia refuses, realizing that Rufus has no business prospects left. If Mia seeks to take the property as possession, will she win? A. Yes, because she has clear record title. B. Yes, following the institution of foreclosure proceedings and assuming that Mia wins the foreclosure hearing. C. No, the $500,000 is below the fair market value of Rufus's property. D. No, Mia can only seek a contract recovery. There is no space in the law for a security agreement such as this.

Answer: B
Mia needs to proceed with a foreclosure hearing, and can take possession of the property assuming that she is the winner of that foreclosure sale. The deed to Mia, in reality, is nothing more than an equitable mortgage for Rufus's property, wherein the deed was given in security for the loan. A court then requires a foreclosure sale. An equitable mortgage is illustrated by showing: 1) the existence of a debt or promise of payment by the deed's grantor; 2) the grantee's promise to return the land if the debt is paid; 3) the fact that the amount advanced to the grantor/debtor was much lower than the value of the property; 4) the degree of the grantor's financial distress; and 5) the parties' prior negotiations. Based upon this, it is clear that we have an equitable mortgage at issue, requiring a foreclosure sale.

A is incorrect in light of the proper answer. This is not a deed conveying property, but rather
an equitable mortgage instrument. Mia, therefore, does NOT possess clear record title of Rufus's property. C is incorrect. The sum of money under fair market value is not enough, when standing

alone, to indicate that this is an equitable mortgage. We must meet the requirements of the
test set forth in the correct answer before assuming that we have an equitable mortgage at issue.

D is incorrect. As previously discussed, the court will treat this as an equitable mortgage.
2007-2008 Law Decks

Property Law
Carerra and Andy live together as a couple, but are not married. They live in a common law marriage state. Andy depends on Carerra for monetary support, but Andy gets nervous as Carerra is older and is quickly losing his instincts as a lion tamer. Andy worries that he may become maimed or worse, and that his children from his first marriage will cut her out of any inheritance. Carerra understands her concerns and, a week later, shows Andy the deed to a hotel that he owns. He has made Andy the signed grantee on the deed. The deed is properly executed. Carerra, in the presence of his lawyer (Wallace) and Andy, says, "Wallace, I want it to be clear that you are to hold this deed until I die. Upon my death, please give it to Andy." Wallace accepts the deed and promises to do as Carerra wishes. One month later, Carerra and Andy have a huge fight and Carrera gets Wallace to return the deed to him. Assume that Andy sues. Who will win? A. Andy, because the deed gave her a present right to possess the hotel. B. Andy, because the facts indicate that Carerra intended Andy to have a property right in the
hotel when he handed the deed to Wallace.

C. Carerra, as the transfer lacked lawful consideration and was, therefore, freely revocable by him. D. Carerra, since the agreement failed to meet the burden of the Statute of Frauds.

Answer: B Andy wins as there clearly was an intention in Carerra for Andy to have an interest in the property. A transfer of a deed requires delivery. Delivery requires words or conduct evidencing the grantor's intention that the deed have some effect; regardless of the fact that the right of possession may be postponed until the future. In a situation where the grantor gives the deed to a 3rd party, most courts hold that the grantor cannot get the deed back because his intent was to presently convey a future interest to the grantee. The facts, here, indicate that Carerra intended to turn the hotel over to Andy at some later date, making his revocation of the deed from Wallace impermissible.

A is incorrect. The present facts show no intention by Carerra to transfer a present


interest to Andy, but rather to transfer a future interest at a later date. Therefore, Andy wins on the basis that she has a future interest in the property, not a current one. C is incorrect as consideration is not required to render a deed valid or irrevocable.

D is incorrect. Instructions to a custodian can be oral and the Parol Evidence Rule
allows the introduction of these conditions and terms which evidence the deposition of the deed with a 3rd party.

2007-2008 Law Decks

Fropert9 Law
Kesey owns a large piece of land adjoining to Red's land and the boundary between them is never properly surveyed and set forth. In 1950, Kesey puts huge windmills on a portion of land that he believes he owns. He also runs wires across a portion of land that he knows Red owns with Red's consent. Kesey goes into the business of selling electricity to the local electric company. In 1965, Red goes insane to due self-inflicted shock treatments and is diagnosed as mentally incompetent. He dies in 1970 and the executor of his estate sues to remove Kesey from the land owned by Red via ejectment. The statute of limitations for ejectment is 20 years. With regards to the land on which the wires run across: A. Kesey has acquired title by adverse possession. B. Kesey has a prescriptive easement.
C. Kesey has both title via a prescriptive easement and adverse possession.

D. Kesey has not acquired title via adverse possession, nor by a prescriptive easement.

Answer: D
Here, we have a use of the land with Red's consent, and no facts indicating that Kesey had intent to claim the land as his own. In lieu of this, the "hostile" requirement of proscriptive easement and adverse possession are lacking, making Answers A, B and C incorrect.

2007-2008 Law Decks

Fropert9 Law
Toby inherits Bamboo Field from his father in 1960. The land sits vacant until 1990 when Toby decides to subdivide Bamboo Field into 20 plots for a housing development. In the center of the plots is a 50 acre parcel with rare bamboo trees destined to be "a wildlife preserve," which remains vacant. Toby prints up brochures and sells the lots around the perimeter of the preserve at a premium, with the explanation that they have the desirable views of the wildlife preserve and rare bamboo trees. He sells 18 plots, and each deed has a restriction: "no satellite dishes may be installed." In 1999, Toby sells the remaining 2 plots, along with the undeveloped 50 acre wildlife preserve parcel to Charlie for $1 million. In the sale, the deed restriction against installation of satellite dishes is contained. Charlie sells the two lots to Fitz and Zoe, respectively. He sells the 50 acre parcel to Lee. None of these deeds contain the plan for a wildlife preserve, or the deed restrictions against satellite dishes. Lee states that he is going to build a business complex on the 50 acre parcel. Fitz decides to put a satellite dish on the house he is building on his lot. Sam, who owns a neighboring lot, objects. By the time Sam objects the dish is already mounted on to Fitz's new house. Sam seeks to have the dish removed. Who will win? A. Sam, as the land had a restrictive covenant and Sam's deed runs with the land.

B. Sam, since his land is not benefited by the existence of a satellite dish on his neighbor's property.
C. Fitz, since Charlie's deed did not have a restrictive covenant in it. D. Fitz, since he had no notice of the restriction regarding the satellite dish.

Answer: A
Here, there is a restrictive covenant that runs with Sam's lot, benefits that lot and also burdens Fitz's lot. A covenant must touch and concern the lot in question, and be intended to run with the property. Here, these requirements are fulfilled. In light of this, B is clearly wrong. C is wrong because the restriction was contained in a previous deed relating to the property. D is wrong. Fitz had constructive notice of the deed restriction had he looked at the recorded deed history.

2007-2008 Law Decks

rropert9 Law
Sadie owns Eatmore Estate in Rhode Island. In 2000 she executes the following deed: "All of Eatmore to Jude, my best friend and confidant." Sadie puts the deed in the kitchen cupboard and tells her trusted attorney, Wallace, to turn the deed over if Jude survives Sadie. By 2004, Sadie hates Jude, and gives Eatmore to Julia in a quitclaim deed for $1,000. Sadie told Julia about the earlier deed and further told her that she would take great pleasure in burning it that evening. Sadie never did burn it, however. She dies in 2005. Jude is the only surviving heir, and Wallace hands the deed over to Jude as instructed to do so by Sadie when she was alive. Jude builds another house on Eatmore and moves in. The house costs $100,000 to build, but it is an ugly house and adds no value to the land. Note that Eatmore was the only property Sadie ever owned. Which best describes the state of the deed from Sadie to Jude? A. Effective as of 2000, as Wallace accepted the deed on behalf of Jude.
B. Effective as a conveyance of title upon Sadie's death in 2005.

C. Ineffective as a conveyance, since Sadie never delivered it to Jude. D. Ineffective as a conveyance, because the description of the land was inadequate.

Answer: C
Here, the deed lacks valid delivery and is, therefore, ineffective. Absent a delivery by a grantor during her lifetime, a deed is ineffective. Note that "delivery" is based upon the grantor's intent. As long as the grantor wanted the deed to have an operative effect, it is deemed delivered. Delivery requires that the grantor relinquished some amount of control over the property. Here, Saide has never delivered and has relinquished no control. Therefore, the deed was not delivered and not valid. A is wrong in light of the correct answer. There was no delivery of the deed as there was no intent to pass the deed to Jude. Note: An agent can pass title, but Wallace was not acting as an agent and, even if found to be an agent, Wallace never actually delivered to Jude. B is wrong. Even if we overlook the delivery requirements, there was no intent in Sadie to relinquish control, making this an ineffective transfer. At the time of Sadie's death, Julia owns the property and there is nothing that can pass from Sadie to Jude. D is wrong. This property description, in light of the fact that Sadie owns no other land, is likely sufficient to convey Eatmore.

2007-2008 Law Decks

Fropert9 Law
Blair and Tony have lived together for years but are not married. Suppose that Connecticut does not recognize common law marriage, but does have status prohibiting discrimination based on marital status. One day, Blair goes out and stops at the local state-run casino during her lunch break. She drops a nickel in a slot machine and wins the $2 million jackpot. She uses the proceeds to buy Vaux Hall, which has a great Art Deco mansion on the property. Blair and Tony record the title and hold Vaux Hall as joint tenants. Tony also decides to make a million gambling, but promptly looses $500,000 playing three-card-monty. He needs to raise this cash to pay off his bookmaker or his knees will be broken. Tony manages to convince Bank-a-Lot Bank to enter into a mortgage against Vaux Hall with his signature alone. (Blair never signs the mortgage.) Seven months later, Tony dies in a tragic supermarket accident involving a jar of pickles and some pipe cleaners. The mortgage remains unpaid. Tony has no will and his only heir is his brother. Suppose Connecticut is a "lien theory" mortgage state. Who holds title to Vaux Hall? A. Blair

B. Blair and Bank a Lot


-

C. Blair and Tony's brother

D. Blair, Tony's brother and Bank-a-Lot

Answer: A
As a joint tenancy carries the right of survivorship, Blair takes full possession and title in Vaux Hall. Upon Tony's death, Blair owns the property in the entirety. B is wrong since most states believe that a mortgage is a lien on the title to the land. When Tony took a mortgage out against Vaux Hall, the joint tenancy was not instantly severed, but rather could only be severed if Bank-a-lot entered a foreclosure hearing against Vaux Hall. Bank-a-Lot's opportunity to enter into a foreclosure hearing passed when Tony died. Upon his death Tony's interest in the property terminated and Bank-aLot, therefore, no longer had any interest in Vaux Hall. C is wrong. Tony's brother has no interest in Vaux Hall. We have a joint tenancy, and in a joint tenancy the surviving tenant receives the interest upon the death of a co-tenant. Succession of rights does not occur to an heir following a co-tenant's death. A will only speaks at death, so only upon Tony's death could his brother potentially gain an inheritance. However, also upon Tony's death, his rights as a co-tenant terminate and with regard to Vaux Hall. Because of this, the outcome would still have been the same even if Tony had left a will, making Answer D wrong.

Blair owns Vaux Hall in full. So upon Tony's death, there are rights to pass to his brother

2007-2008 Law Decks

Fropert9 Law
On December 2nd, Zane takes out a mortgage on his house for $100,000 with WorldBank. On December 5th, he conveys his house to Sheryl for $500,000. On December 7th, WorldBank records the mortgage. On December 11th, Sheryl records the deed transfer for the house. Is Sheryl's hold on the house subject to the mortgage to WorldBank? A. In a race-notice jurisdiction, yes. B. In a race-notice jurisdiction, no. C. Yes, regardless of the type of recording statute. D. No, because Sheryl is a bona fide purchaser for value who purchased the house before WorldBank recorded.

Answer: A WorldBank's mortgage was recorded first, so Sheryl takes subject to the mortgage if operating in a race-notice jurisdiction, because it was recorded first. Note: All of the recording acts apply to both deed and mortgages, so a subsequent purchase takes subject to any and all mortgages unless there is a different result based on the recording act. Sheryl probably did not have notice of the prior mortgage (due to the close proximity in time) but she recorded after WorldBank recorded and, therefore, takes subject to WorldBank's mortgage. B is wrong since Sheryl failed to win the race to record, which is necessary in a racenotice jurisdiction for a bona fide subsequent purchaser to prevail. C is wrong. If we are operating a pure notice jurisdiction, Sheryl would not take the house subject to WorldBank's mortgage. A subsequent bona fide purchaser that does not have notice (actual or constructive) prevails over a prior mortgagee who failed to record. D is partially correct, but only in an true notice jurisdiction. In a race or race-notice jurisdiction, Sheryl takes subject to the mortgage of WorldBank.

2007-2008 Law Decks

Fropert9 Law
Suppose that Ray owns an impressive mansion in Houston. The mansion includes an ornate leather club chair made in 1992 by HUDDY 5000. Ray dies and leaves his personal property to his only son, Charles, and his real property to the city of Houston. The chair was part of a built in seating banquette in the house made from the same color leather. After probate on the will, Charles empties the mansion of all of his father's personal possessions, taking the special chair with him. Suppose a Houston official notices that the chair is gone after walking through the mansion. He asks for its return, stating that the chair was part of the real property of the house since it was specially designed and built for the house. Charles refuses to return the chair. If the city of Houston sues to replevy the chair, how will a court rule? A. For Charles, as the chair is personal property since it was not bolted down. B. For Charles, as the removal of the chair did not damage the real property.

C. For Houston, as the chair was part of a fixture in the house and cannot be removed.
D. For Houston, as the removal of the chair reduced the value of the house.

Answer: C
Houston will win since the banquette is a fixture to the house and the chair is integrally connected to the fixture. A chattel annexed to real property is converted from a personal piece of property to a real piece of property and passes with ownership in the land. To determine whether a chattel becomes a fixture, we must look to the intent of the annexor. What is: 1) the relationship between the annexor and the premises, 2) the degree of annexation, and 3) the nature and use of the chattel. Here, Ray was the fee owner of his mansion and had the banquette built specifically for him where 2) the banquette was built into the wall and could not be easily removed, and 3) the appearance of the banquette, and the leather club chair associated with it, were probably most important to Ray and not the actual seating it provided. In light of this, the chair probably became part of the real property and is no longer personal property and Houston will likely win the return of the chair. A is incorrect since permanence and bolting to the floor is not determinative of whether the leather club chair was a fixture. Here, the chair was an accession to the banquette and is, therefore, a fixture. B is incorrect. The ability to remove the chair without doing damage is not enough to give Charles

the right to take it. Here, the chair is an accession to the banquette and removal would damage the
banquette. D is incorrect. The test for whether an item is a fixture is NOT if removal would constitute harm to the parties. The relevant question is if removal of the item damages the real property, which, based on the present facts, would occur.
02007-2008 Law Decks

Fropert9 Law
Jag wants a garage built and hires Besnik. He wants the garage on the backmost region of his property right near the property line. Jag tells his neighbor, Lala, about the garage: "It's going to be 6 cars wide, so there will be plenty of room. You are more than welcome to park your car in it, too, once it is finished if you like." When Besnik starts digging for the foundation, Lala is home and takes great interest watching the workers dig. She is friendly as she stands around and watches, but makes it clear to Besnik that he and his workers may not go onto her property. After several hours of excavating, part of Lala's yard collapses into the excavated foundation taking Lala's garage with it. What is Jag's best defense? A. Jag gave Lala notice of the excavation, and she did not object. B. Besnik was hired based on his good reputation and numerous recommendations for his quality work.

C. The excavation was done in a workmanlike manner, and there would not have been a collapse except for the fact that Lala had a garage there.
D. Lala's garage was made of a 20 inch thick concrete floor and marble walls, and was way heavier than an ordinary garage.

Answer: C
Absolute liability does not attach if the land would not have subsided and collapsed, except for the fact that Lala had a building there. Liability in such a situation would only attach if Jag was acting in a negligent manner. In light of this, Answer C addresses the potential negligence and the fact that the building was there, making it the best answer. A is wrong. Jag giving notice and Lala failing to object to his garage, does not result in Jag being excused from liability. B is wrong since it only addresses negligence and fails to address the existence of Lala's garage. D is a decent answer, but not the best. Jag could still be liable if negligence is proved, regardless of the fact that Lala's marble garage was heavier than normal.

2007-2008 Law Decks

Property Law
Luna partners with Lupe to develop Wolf Moon Estates in 1990. In 1999 they acquire all of the required permits and begin to build Wolf Moon's residential development. The first portion is comprised of 100 houses on 250 acres of land. The second portion is a restaurant complex built on an adjoining 50 acres of land. Adjoining Wolf Moon Estates is land owned by Lupe alone, on which he intends to put up 200 single-family homes, with deed restrictions similar to Wolf Moon Estates where the residential nature will be preserved. If Lupe does this, will these be enforced?

A. No, since the applicable zoning restrictions will control. B. No, to have restrictions like this, the entire Wolf Moon Estates project must have been built at the same time. C. Yes, the restrictions will be equally enforceable for this new project in a similar manner as those directed to Wolf Moon Estates. D. Yes, if a court decides that this new project is part of the common development scheme of the original Wolf Moon Estates housing development.

Answer: D
D is correct. Upon a showing of a common scheme of development (as evidenced here) prior purchasers can enforce restrictions in deeds to subsequent pieces of property; if these subsequent pieces of property are from a common grantor, so long as that general scheme intended to benefit the entire subdivision. In lieu of this, Answer B is clearly wrong. A is wrong. The existence of zoning restrictions does not absolutely limit the enforceability of covenant deeds and restrictions. C is wrong. It fails to address a situation in which the court may find that there is no common development scheme, making the restrictions unenforceable.

2007-2008 Law Decks

Fropert9 Law
Luis owns Shin Mountain and in 1970 decides to subdivide it into 20 lots of 1 acre each. He keeps Lot 1 and sells Lots 2-20 with the deed restriction stating: "All lots will be used for single -family residential construction." Luis builds a house, as do owners of Lots 2-20. Luis lives in his home on Lot 1 until his death, at which time it is sold without any restrictions to Al. In 2001, Al dies and Lot 1 is sold to Henna without any deed restriction. In 2002, Henna decides to turn her house on Lot 1 into an adult video store and bondage emporium. Tasha tells Henna that she cannot do this and mentions the original deed restriction to her. Assume that when Henna buys Lot 1 from Luis's estate, she is given an abstract of title which states that the land has no restrictions. Henna, therefore, asserts that she cannot be bound to the negative restriction since she had no notice of it. Assume that Tasha brings suit. Is Henna's lack of notice defense sufficient? A. Yes, Henna lacks either actual or constructive notice of the restriction. B. Yes, Henna has clearly relied on the abstract of title and the representation of Luis in good faith. C. No, the property owners have a right to maintain the subdivision in the form they all originally agreed to. D. No, Henna is on notice in light of the residential nature of the subdivision.

Answer: D
Here, Henna is charged with inquiry notice as the facts indicate that Lots 2-20 all have residential single-family homes on them. Answer A is, therefore, clearly incorrect as Henna does have notice (inquiry notice). B is incorrect because reliance of the abstract of title does not negate the inquire notice that Henna is held to have. C is a bogus rule of law that has no standing in a court of law but rather appeals to the "fairness" arguments. Note: "Fairness" arguments, absent a legal reason, are always wrong answers.

2007-2008 Law Decks

Fropert9 Law
Leroux owns Lorien Gardens in fee simple and in 1970 converts the land "to Jamal for life and to the first child of Jamal's brother Kurt, who shall reach the tender age of 21 years." At the time of the conveyance in 1970, Kurt is unmarried. In 1976, Kurt marries and in 1978 Kurt has a daughter, Loriel. Assume that Kurt dies before Loriel becomes 21 and the jurisdiction does not have the common law rule of destructibility of contingent remainders. Who owns Lorien Gardens? A. Loriel B. Loriel's guardian ad lietum C. Leroux and his heirs in fee simple D. Leroux and his heirs hold the estate until Loriel turns 21, at which point her interest vests.

Answer: D Leroux holds a potential reversionary interest, and would exercise such an interest until Loriel reaches 21 years of age and can take for herself. A is wrong as Loriel cannot take possession until she reaches 21. Here, Loriel is not 21, has no right to take possession and, therefore, cannot actually take possession until she turns 21. Loriel cannot take possession through a guardian as set forth in Answer B, as Loriel has no right to possession until she is 21 years old. C is wrong in light of the correct answer. Additionally, Leroux does not hold a fee simple, but rather a reversionary interest until Loriel reaches 21 years of age.

2007-2008 Law Decks

Fropert9 Law
Jung owns land in fee simple. In 1990 she devises land to Ty for life with a written instrument that states: "To Ty for life; if Ed and Ned return home from looking for the Northwest Passage prior to Ty's death, to Ed and Ned and their heirs. If they do not return home before Ty dies then the remainder is to go to my heirs." In 2004 Jung and Ty both die in a tragic smoking accident. Ty's will left the estate to a pro-smoking club, and Ty's remaining relative is his brother, Jake. Assume for this question that Ed and Ned do find the Northwest Passage. At this point in time, what is the status of their interests? A. Tenants in common to a vested remainder B. Joint tenants to a vested remainder C. Tenants in common to a fee simple absolute

D. Tenants by the entirety to a fee simple absolute

Answer: A
Ed's and Ned's remainder each has vested as the condition precedent to their remainder has been satisfied. Based on common law principles, any conveyance to two or more persons is assumed to create a joint tenancy unless absent a showing of some rule to the contrary. In today's state of the law, however, such a conveyance creates a tenancy in common, as a joint tenancy requires wording such as "as joint tenants" or "in joint tenancy" to show the necessary intent. Therefore, Ed and Ned are tenants in common to a vested remainder. B is therefore wrong. C is wrong since the estate here is in a remainder interest as the life tenant is still alive. Therefore, we are not dealing with a fee simple estate. D is wrong for the same reason as Answer C. Additionally, a tenancy by the entirety can only be held by a husband and wife. Here, we are not dealing with spouses.

2007-2008 Law Decks

Fropert9 Law
Clay's basement is prone to flooding and Clay knows this, but the estimate to fix it is $10,000. Being cheap, Clay slaps a coat of paint over the damaged and water-stained walls. After spending $5,000 to have the place repainted, he puts the land and house on the market for $200,000, hoping that the house will sell before the next heavy rain. Josef looks at the house, likes it and offers $195,000. They enter into a written sale for purchase at $195,000. The sale is contingent on a home inspector signing off for the property. The inspector looks at the place, mentions that the basement is a nice color and looks new, and signs off. The inspector finds a few little problems which Clay agrees to fix, costing him a total of $500. The closing happens without a hitch. Two months later, it begins to rain and the basement floods, damaging Josefs collection of rare comic books. Josefs insurance covers the comic books, but will not repair the roof, noting that this was ordinary wear and tear. If Josef sues Clay for $15,000 (the cost of a new roof) how will the court rule? A. For Josef, as Clay's actions were a breach of the implied warranty of fitness for a particular purpose.

B. For Josef, as Clay knowingly concealed the defects of the house.


C. For Clay, since Josef had ample opportunity to inspect the house and Clay had no duty to disclose the house's defects. D. For Clay, since the property was not new construction built by Clay himself.

Answer: B
A court will likely find for Josef, since Clay knowingly concealed the defects. Even though the general rule says that property generally has no implied warranty for a particular purpose; here, we are dealing with a situation where Clay knowingly and intentionally concealed the defects. Had Josef known of the flood prone basement, he would have walked away from the deal altogether, or at least demanded it be fixed before closing. A is incorrect because the implied warranty of fitness only applies to new construction, or a house owned by a builder. Here, neither is met based on the given facts. We know that this is not a newly constructed home, and there are no facts indicating that Clay was a builder. C does, in fact, state the general rule, but its application is in error. A seller does not have to explicitly bring buyers' attention to existing defects, but at the same time he may not conceal them. Although D does, in fact, set forth grounds for a situation where the implied warranty is not applicable, the simple fact that Clay purposefully concealed the defects has resulted in a situation where Clay remains liable.
2007-2008 Law Decks

Fropert9 Law
Mickey buys Intrigue Forest and builds 10 log cabins. All the cabins are connected to a well drilled in Intrigue Forest. Skip buys Panda Estates (adjacent to the Intrigue Forest), builds a house on it and drills a well for the house. Skip uses a lot of water, due to his desire to take 10 showers a day. Six months later, the well in Intrigue Forest runs dry. (note, that we are dealing with percolating water). Oddly enough, Skip still has water from his well, though it has slowed down a bit. If Mickey sues Skip for an injunction to prevent Skip from drawing from the well and running out of water, how will a court rule? A. Skip must cut back on water usage so Mickey's cabins have enough water supply. B. Skip must pay money damages to Mickey. C. There must be a transfer of water from Skip's well to Mickey's well.
D. Nothing

Answer: D
Most states follow a reasonable use theory regarding water wells. As long as Skip's use was reasonable for the benefit of the land and not a malicious misuse, he is not responsible for doing anything to benefit Mickey. A is wrong. Skip may use as much water as he likes, so long as it is not a malicious misuse or waste. Note: The minority view prior appropriation rule, granting that the first to use the water has the most rights, might make this answer correct. We have no facts, however, to indicate that the minority rule is applicable. B is wrong under the majority view. However, this would be the best answer under the minority rule of prior appropriation view. Note: The minority rule states that domestic use such as Skip's has priority, so Skip may extract water. However, water rights are deemed property rights, making Skip liable to Mickey for damages. C is incorrect under all theories of water law.

2007-2008 Law Decks

Fropert9 Law
Florence conveys land, "To Lisa and her heirs, except if Lisa tries to convey the property, then to Eric and his heirs." During Eric's lifetime, which answer best describes his

interest? A. An executory interest B. A contingent remainder C. A vested remainder D. None of the above

Answer: D
Eric has no interest in the land. Lisa has a fee simple subject to a condition subsequent, while an executory interest is created in Eric. The condition subsequent, however, attempts to restrict the transferability of a legal interest in property. Therefore, it is deemed a restraint on alienation and is void, giving Lisa a fee simple interest, and Eric nothing. A is incorrect because, as explained above, Eric's interest is void. Answers B and C are also incorrect for the same reason. Additionally note that it is impossible for a remainder to follow a fee simple interest, as a remainder can only follow a fee tail or a life estate.

@2007-2008 Law Decks

Fropert9 Law
Romeo owns ocean front property and conveys it, "To the woman who is the most precious and special person I have every known, Juliet, for life, then to the heirs of my brother, Monty." At the time of the conveyance, Monty has not yet married, preferring the swinging singles life, and has no children that he knows of. At the time of Juliet's death, Romeo is still alive and Monty is unmarried and childless. What interest do Monty's heirs have? A. Shifting executory interest B. Springing executory interest C. Contingent remainder D. Nothing

Answer: C
Monty's heirs have a contingent remainder. By definition, a remainder is a future interest created in a transferee that is capable of taking in present possession and enjoyment, upon the termination of the preceding estate, which was created in the same conveyance. Remainders must be expressly created in the granting instrument and almost always follow life estates. A contingent remainder is such that it is created to benefit a party not yet born or ascertained. Here, Monty clearly has a remainder which is contingent, as his heirs are yet to be ascertained. A is incorrect. A shifting executory interest cuts short the prior estate, thereby divesting the prior estate's interest. Here, the interest of Monty's heirs does not cut short Juliet's estate, but rather simply follows it. B is incorrect because a springing executory interest follows a gap in possession, or divests the estate of the transferor. At the time of conveyance we do not have a gap in the transferor's estate, but rather, Monty's interest simply follows Juliet's interest without

a gap. Answer D is obviously incorrect in light of the aforementioned correct answers.

2007-2008 Law Decks

Fropert9 Law
Tybalt lives in South Carolina. Suppose that South Carolina has a recording statute that states: "No conveyance is good against a subsequent purchaser for value, without notice, who first records." Tybalt conveys land to Samson, his son, but never records the deed and leaves the deed in his underwear drawer. Two years later, Tybalt dies in a tragic accident involving swordplay at a fancy dress ball. In his will, all of his property is devised to Samson, his son, and Paris, his daughter. While going through his father's underwear drawer, Samson finds the deed, shows it to Paris and they agree not to record. Samson puts the deed in his sock drawer at his house, and a year later dies. Balthazar is executor of Samson's will, and finds the deed. He records it, then contracts with Peter to sell Tybalt's land. Paris finds out about this and sues, claiming an interest in her father's property. The court will rule: A. In favor of Paris, as the deed was not properly delivered. B. In favor of Paris, since Balthazar's actions were violative of the fiduciary duties of an

executor. C. Against Paris, as Balthazar and Peter are protected under the applicable recording act.
D. Against Paris, since she is not a bona fide purchaser for value.

Answer: A
Here, the deed was never properly delivered as there was not a clear expression of intent
that the grantor envisioned the passage of title to the grantee. Furthermore, Tybalt's failure to deliver continual possession of the deed sort of implies that there was no intention to relinquish control in the property. In light of this, Tybalt's land was part of his estate, and Samson and Paris take it as tenants in common.

B is wrong because a question of violation of fiduciary duties has no bearing on ownership of the land and is irrelevant.
C is wrong for two reasons: 1) Since this is a race-notice jurisdiction Samson cannot rely on recording statute protection. To prevail in a race-notice jurisdiction, a party must be a subsequent purchaser for value who does not have notice of an adverse claim, and must record first. Samson has not recorded and has not paid fair value, and further has notice that he is not a bona fide purchaser since he knows of Paris's will claim. 2) Balthazar cannot be protected under the subsequent recording as he is Samson's agent and Samson was a donee with notice.

D is wrong. As explained above, Paris inherited a 1/2 interest in Tybalt's land as a tenant
in common, following the will devise of Tybalt's land. In light of this, Samson cannot attempt a subsequent recording, as he clearly had notice of Paris's claim to the land.
2007-2008 Law Decks

Fropert9 Law
Edie and Simon own adjoining land. Edie's property is west of Simon's. Adjoining Simon's property on the other side to the east is Hot Spring Caves. In 1980 Edie buys inflatable rubber duckies for her children to use while soaking in the hot springs. She asks Simon if it would be okay for her and her children to walk across a 10 foot expanse of his property to gain access to the hot springs. Simon knows that the only other access is a circuitous 30 minute car ride over 15 miles, so he agrees to let Edie and her kids use the 10 feet of his property to get to the springs. Edie and her family use this strip for 19 years, at which point Simon sells his property to Art. Art sees Edie walking the strip every Monday, but says nothing, knowing that this is, in fact, his property. On the 9th month of noticing this, Art speaks up and prohibits Edie from using the strip. Edie and her family continue to use the land, regardless of the fight, and continue for 5 more years, at which point Art gets an injunction. The adverse possession period is 20 years. Which, from the following choices, best supports Art's argument to prevent Edie and her family from using the 10 foot expanse of land? A. The sale of the property terminated any potential adverse possession claim Edie might have had.

B. Edie was informed to stop using Art's land before the 20 year adverse possession period ended.
C. It is impossible for Edie to acquire an interest in the land as her agreement with Simon was originally an oral agreement. D. Edie's use of Simon's land was with the permission of Simon.

Answer: D
Here, we have permissive use. It is neither hostile nor adverse to Simon's interests, making an adverse possession argument inappropriate for the 19 year period before Art takes possession. A is wrong. Changes in ownership of the land do not terminate adverse possession. B is wrong. Even in light of the instruction to stop using the land, Edie continues to do so for a period after the fight. C is wrong as it is too broad to be applicable. While there was clearly no express easement due to the oral agreement between Edie and Simon, it is possible that an easement by prescription could have been created even in light of the oral agreement.

2007-2008 Law Decks

Fropert9 Law
Antoinette owns 2 homes on a plot of land that spans 10 acres. She enters into the following agreement with Samir, "I sell one house to Samir for $1 million, with enough land attached to build a garage and sauna. Received from Samir is a $1,000 deposit. Antoinette DeSanto, March 1st." On March 15th, Samir enters into an agreement with Marie to resell the house to Marie for $1.5 million. Assume that Antoinette hears of the deal with Marie and decides to sell to her directly for $1.5 million. Samir sues seeking specific performance. What is the outcome in a court of law? A. Antoinette will prevail, as the $1,000 consideration is not enough to support the proposed contract. B. Antoinette will prevail, as the contract is too ambiguous to be deeded sufficient to describe the land. C. Samir prevails, as he has an agreement in writing from Antoinette. D. Samir wins, as his contract with Marie is enough to clearly describe which house Marie is actually buying from him.

Answer: B The contract with Antoinette is too ambiguous. It fails to specify which of the two homes on Antoinette's property is being sold, and the site of the land granted for the garage and sauna is not set out. In light of this, we cannot go forward with specific performance. A is wrong because $1,000 can be deemed sufficient consideration, so long as there is a mutual agreement to buy and sell the land. C is wrong. A contract needs to include a description of what is actually being sold. A mere purchase price term alone is not enough. D is wrong. The introduction of evidence from the contract with Marie is irrelevant. What is at issue is the lack of description in the contract between Antoinette and Samir regarding which house is for sale.

2007-2008 Law Decks

Fropert9 Law
Bradley owns Buckland Shore. He builds his home on the northern parcel and develops a pear orchard on the southern parcel. There is a country road leading to his home on the north part of the northern parcel of land. Bradley extends this road from the northern parcel down to his southern part of land. He additionally runs above ground power lines on poles across the southern portion to get power to his house up north. After years of raising pears, Bradley gets pesticide poisoning. Just before dying, he deeds the northern part to Gary and the southern part to Carla. The deed to Gary reads, "To Gary for life and then to his widow for her life, remainder to Gary's children then alive." The deed to Carla is simply, "To Carla, Enjoy raising pears and don't drink the pesticide." Carla has an easement in the extended road that is plainly visible. Which act would extinguish the easement? A. Non-use for the statutory period applicable to prescriptive easements. B. Sale of the northern parcel to a purchaser for market value, who is unaware of Carla's easement.

C. Excessive use of the easement in a manner that unreasonably increases the burden on
Gary's northern parcel of land.
D. None of the above

Answer: D
The notable reason for extinguishing an easement occurs when the easement holder demonstrates by a physical action that they actually intend to permanently abandon the easement. Note, however, that this is not the case in the present fact pattern, making D the correct answer. A is incorrect. Non-use, absent a manifestation of intention to no longer use, will not result in an easement's extinguishment, even if such non-use continues for the statutory period applicable to prescriptive easements. Note: The statutory period of limitations is only applicable to termination of an easement by actions advanced by the holder of the servient tenement, not actions or non-actions of the holder of the dominant tenement. Here, the dominant tenement is held by Gary and the servient by Carla. B is incorrect as the burden of an easement appurtenant passes with the servient land when transferred or sold. The new owner is, therefore, bound by this easement as it is

clearly visible, putting any purchaser on inquiry notice of an easement.

C is incorrect. If faced with an excessive use complaint, a court would likely put a limit on the scope or use of the easement; but it would not revoke the easement altogether.
2007-2008 Law Decks

Property Law
Felix builds Cara Mia Estates, and includes the following in each lot deed, "Grantee agrees for himself/herself and assigns to only use this property for a single-family home construction, and to pay the monthly service charges to Cara Mia Estates for upkeep and security. Grantee also agrees not to put up any fences so that the indigenous cats may wonder the grounds as they please. Grantee will also agree never to paint a house '80s Green' since it is tacky." Felix sells lots to Minx, Luis and Nan, and each deed is recorded. Minx resells to Mike, but this deed has no language noting the restrictions and Mike has no notice of them. Mike then starts selling junk scrap metal out of his garage and keeps it on the side of his house. Luis resells to Kendra, who is aware of the monthly fees, but refuses to pay based on the suggestions of a ghost she claims to have met in her new house. Nan resells to Cindy who builds a 12 foot cinder block wall around her property and digs a mote, unaware of the fence restriction. Answer based on common law requirements. Assume that the Cara Mia Homeowners Association sues Kendra for the unpaid monthly fees. What is the outcome?

A. Kendra would win, because there is no privity between Kendra and the homeowners association. B. Kendra would win, as the covenant regarding fees does not touch and concern the land.

C. The homeowners association would win because the covenant regarding fees created an equitable
servitude which binds Kendra. D. The homeowners association would win because the covenant regarding fees created a burden that runs with the land at law.

Answer: D
Here, we have a burden that runs with the land. The requirements for running of a burden are present as the language of the covenant indicates that the parties' intended successors in interest to the covenantor shall be bound by the terms. Additionally, Kendra has actual and constructive notice of the covenant at the time of purchase. Horizontal privity exists between the promissory and the promisee at the time the covenant was entered into. Felix and Luis shared an interest in the land. Vertical privity also exists since Kendra takes the entire estate that Luis once held. Finally, the burden of the covenant touches and concerns the land, because the fees result in an increased obligation of a landowner for use of the land.
A is incorrect. When faced with a deed that is silent as to who may enforce the covenant,

and if faced with a general plan for the subdivision, any member of the subdivision is entitled to enforce the covenant. Such a common plan clearly existed, allowing any landowner, including the homeowners association as a whole, to enforce the covenant.
B is incorrect as it concludes that the covenant, here, fails to touch and concern the land.

As set forth in the correct answer, this is clearly not the case. C is wrong for giving the incorrect measure of damages. Breach of a real covenant is
remedied by an award of money damages, while breach of an equitable servitude is remedied by equitable relief such as an injunction or specific performance.
2007-2008 Law Decks

Fropert9 Law
Carerra and Andy live together as a couple, but are not married. They live in a common law marriage state. Andy depends on Carerra for monetary support, but Andy gets nervous as Carerra is older and is quickly losing his instincts as a lion tamer. Andy worries that he may become maimed or worse, and that his children from his first marriage will cut her out of any inheritance. Carerra understands her concerns and, a week later, shows Andy the deed to a hotel that he owns. He has made Andy the signed grantee on the deed. The deed is properly executed. Carerra hands the deed to Andy with language in the deed stating, "title to the property will be conveyed upon the death of the grantor named herein." Shortly, thereafter, Carerra is eaten by a lion when a woman in the front row coughs and distracts him. He does not leave a will. The administrator of Carerra's estate sues Andy seeking quiet title to the hotel. Who will win? A. The administrator will win, because the deed was improperly delivered. B. The administrator will win, because the transaction between Carerra and Andy violates the Statute of Frauds. C. Andy will win, so long as she produces additional evidence that Carerra intended for her to have an interest in the property while he was still alive. D. Andy will win, as she possessed a future interest, which has now become a possessory interest.

Answer: D
Andy now holds a possessory interest, as this deed (which contains an express provision that title will not pass until the death of the grantor) conveys a present possessory life estate in the grantor and a future estate in the grantee. Andy's future interest became possessory once Carerra died. The language within the deed is such that it is clear Carerra intended to make the deed legally effective when delivered. Therefore, we have a valid delivery and Andy can take possession of the estate. This makes Answer A incorrect. B is incorrect since the deed at issue created a conveyance of a future interest in the hotel during the lifetime of the grantor when delivered to Andy. Note: As Carerra intended that no interest would be transferred until his death, the deed would be testamentary in nature; thereby, requiring that the deed be executed in accordance with all of the requisite testamentary formalities such as, a signature in front of witnesses etc... Here, however, we can safely assume that the deed conveyed an interest to Andy immediately, and that the deed was not burdened by the testamentary

requirements of a will.

C is incorrect. As long as we find that the deed created the intention of an immediate future interest in Andy at the time of conveyance; we do not need to address the question of Carerra's intent that Andy receive an interest during his lifetime.
2007-2008 Law Decks

Fropertg Law
Green divides her property, Green Acre Commons, into 100 quarter-acre residential lots. She includes in the deed of all 100 grantees the following provision: "Grantee covenants for herself and her heirs and assigns, that all bottles, cans and grass clippings of grantee and her heirs and assigns shall be recycled. This covenant runs with the land and shall remain in effect as long as there is a recycling center within five miles of the Green Acre Commons Development." Green advertises the development as "An Environmentally Sound Place to Live," proudly referring to the recycling covenant to prospective buyers. Therefore, the development tends to attract "environmentally aware" persons. When Green sells her lots, there is a recycling center only 1 mile away. Hansel, an environmentalist, purchases a lot at 15 Spotted Owl Way. His deed contains the recycling clause and he happily recycles. Two years later, Hansel wins the lottery and moves to Brazil. He gifts his property to his niece, Ginger, whose deed also contains the recycling covenant. Shortly after Ginger takes possession, the recycling center moves to a new site 4 1/2 miles away. Ginger continues to recycle, but falls in love with Stan. They marry, and Ginger puts her house up for sale. On the advice of her real estate agent, she says nothing to prospective buyers about the recycling covenant. Vito Veteran, who lost the use of his legs in Vietnam, buys the house. He is impressed that the house is wheelchair-accessible with only a few minor alterations required. Vito's deed does not contain the recycling clause. He hires a gardener to keep the yard who bags all of the grass clippings. He also hires a garbage disposal service to pick up his trash and they deposit it in a landfill. Watching the disposal trucks drive up to Vito's property infuriates many neighbors. They inform Vito of his duty to

recycle. Vito explains that he knew nothing of the covenant and, because of his physical condition, must

rely on the disposal service. Unfazed, the neighbors file suit to require Vito to comply with the covenant, or pay damages. Which is Vito's best defense? A. Vito's deed did not contain the covenant. B. The covenant does not touch and concern the land. C. An intelligent inspection of the neighborhood would raise no inference that the covenant existed. D. Vito's physical condition requires a balancing of hardships by the court.

Answer: B
The covenant does not clearly "touch and concern" the land. While recycling may benefit the community at large, "touch and concern" involves the relationship between landowners at law. Recycling by Vito does not directly benefit the other landowners in the use and enjoyment of their land. Thus, B is correct. A is wrong. Even though Vito's deed does not contain the covenant, Vito has record notice because the restriction is in Vito's chain of title. C is wrong because servitudes implied from a common scheme apply only to negative covenants, and the recycling requirement is an affirmative covenant. Thus, this defense does not go to the point. D is wrong because it goes only to issues in equity. The suit includes a claim for damages at law. Regardless, balancing of hardships is not generally applied in such cases (although some courts might elect to do so).

2007-2008 Law Decks

Fropertg Law
A conveyance statute reads: "No conveyance is valid against any subsequent purchaser for value without

notice unless the conveyance is recorded. No lease of three years or more is valid against a subsequent purchaser for value without notice, unless the lease has been recorded."
Grimm is a successful businessman. He helps out Gip, a friend, by leasing him a convenience store that he owns. He figures that Gip will get rich selling lottery tickets. The lease is for 10 years. Grimm tells Gip, "You really ought to record this." Gip says, "Grimm, no need for me to record, any prospective purchaser will clearly realize that I'm in possession of the store. After all, Sticks n'Straw Convenience is open 24

hours a day. When a potential buyer comes to inspect the place, I'll tell him about the lease, and sell him a lottery ticket. No need to worry." Grimm eventually sells the property to B. B. Wolf, who drives by on
Christmas day, the only day the store is closed. She thinks the place looks good and continues with the transaction. Wolf has no notice in the recorded lease of Gip's lease. After she closes on the building, Wolf tries to kick Gip out. Gip refuses and asks where he should send the rent. The deed transferred from Grimm to Wolf was a warranty deed. What will be the outcome of an eviction action? A. Wolf will not be able to evict Gip, since the drive-by inspection will be deemed to confer actual notice of Gip's possession of the place.

B. Wolf will not be able to evict Gip, since she has a duty to adequately inspect the place.
C. Gip will be evicted, as Wolf was a subsequent purchaser for value with no notice due to Gip's own failure to record. D. Gip will be evicted, since Grimm knew of the statute and willfully failed to record.

Answer: B

Wolf's inspection was inadequate since a title search is not complete without an
examination of possession. In a situation where the possession of the property is unexplained by the record, the purchaser is obligated to make inquiry such as a thorough inspection. A purchaser is charged with knowing anything that would have been disclosed had they went to the property and checked it out. Therefore, Wolf is on constructive notice of Gip's possession. Wolf, is, therefore, not protected by the recording statute and she will take subject to Gip's lease. A is wrong because "actual notice" means that Wolf knew of Gip's possession of the property. Here, that is clearly not the case. The drive by inspection, additionally, has no bearing on the situation since it does not confer anything on Wolf. C is wrong since Wolf has constructive notice of Gip's possession of the property, and is not awarded protection under the recording statute. D is wrong. Gip's state of mind is irrelevant when dealing with an application of the recording statute. Note: Avoid this type of red-herring answer.

2007-2008 Law Decks

Fropert9 Law
Duke owns Corn Wall Hall, a huge compound with vineyards, cornfields and a great history. It is eventually bought by Lear, who runs it as a fantasy baseball resort. The property passes by inheritance to Cordelia, Lear's daughter, and from Cordelia to Cordelia's daughter Regan. Regan decides to sell the land to Kent, a developer, for $2 million. The contract between Regan and Kent is recorded and $1 million earnest money is put up by Kent. The closing is set for February 1st, 2 months after the contract signing. On January 1st, a crazy nephew of Lear burns the place to the ground. On February 1st, Regan shows up at the closing seeking to get the remaining million, with the signed deed in her hand. Kent refuses to hand over the money. Regan has already spent the cash on a new yacht and sues for specific performance. Kent counter-sues for return of the $1 million in earnest money. Both parties feel that the value of the property without the building that burned is $1.6 million. At trial, Regan will likely: A. Not prevail on the issue of specific performance, but will be allowed to keep the earnest money. B. Not prevail on the issue of specific performance, and will be ordered to return the earnest

money.

C. Prevail on the issue of specific performance, but the price will be reduced to $1.6 million. D. Prevail on the issue of specific performance for the full contract price.

Answer: D
Regan will likely prevail. Majority rule states that in a situation where property is destroyed before performance, but after the contract is signed, the risk of loss is on the buyer. Therefore, Kent must pay the contract price regardless of the decrease in value due to the fire. Note that the contract, here, was silent about loss; therefore, the risk of loss remains with the buyer. A and B are incorrect for falsely concluding that specific performance is not an applicable remedy. This is clearly contrary to the correct answer. B is also incorrect because it states that the original $1 million earnest money needs to be returned. As set forth prior, Regan is entitled to the entire contract price, so returning the earnest money is unnecessary. C is incorrect because it allows Kent to tender less than the agreed upon contract price, even though the risk of loss was on Kent. As set forth prior, there is no reduction in price and the contract must go forward at the originally agreed upon price.

2007-2008 Law Decks

Fropertg Law
Hood owns Red Cape Farm outside the town of Leaf Hollow. Red Cape Farm is 100 acres of flat land. Leaf Hollow has no land use regulations, a boon to developers. Hood sells Red Cape Farm to Cindy Ella (a developer) with no deed restrictions, and retires to Maine to lobster fish. Ella divides the property, into one 40 acre plot and one 60 acre plot. The 40 acre plot is further divided into 80, 1/2 acre plots for homes in accordance with any applicable zoning laws. No plot map is needed due to the lenient zoning laws, and Ella begins selling the 1/2 acre plots. Each buyer gets a deed with a "restrictive covenant" included, stating that the land may only be used for "homes with a 2 car garage maximum." Each buyer is also given a computer-generated photo of what the complete development will look like in a brochure. White, one of the first buyers and first to construct a house on one of the lots, sells to Beauty, without any deed restrictions. At this point, there are numerous homes, with the maximum 2 car garage attached. Ella then sells the 60 acre lot to Hunter, who intends to use it as a tree felling school for loggers. The transferred deed has no restrictions. Directly after Beauty moves into her new house, Hunter begins teaching chainsaw techniques to his students. Beauty can't sleep with all the noise and files suit to enjoin Hunter's use of the 60 acre lot. Which is the best argument in Hunter's defense? A. Ella never expressly promised in writing that she would restrict the remaining lots when she conveyed to White. B. Beauty and Hunter are not in the same chain of title. C. The deed received from White did not contain any restrictive covenants. D. The 60 acre lot was not part of a common scheme.

Answer: D
Here, the 60 acre lot was not part of a common scheme, therefore, Beauty cannot enjoin Hunter's use of his lot. Hunter's deed, or any in the chain of title to him, has no mention of a restrictive covenant that he should have been aware of. Beauty, therefore, must show some implied reciprocal negative servitude in Hunter in order to win. An implied equitable servitude (in a situation where a developer divided land into numerous plots) can be implied in a situation where: 1) there is a common scheme of development and 2) the grantee has some notice of the restrictive covenant. Here, both 1 and 2 are lacking, therefore, the common scheme requirement is not met and Hunter does not have the required notice to be bound by the covenants of Beauty's land. Furthermore, there is no inquiry notice, as this is not part of the same development.

A is wrong for the above reasons, and would additionally be wrong if there was a showing of a common
scheme. A prior purchaser of a burdened lot cannot enforce a promise against subsequent purchaser, unless the original developer promised the prior purchaser that they would restrict the remaining lots in the development. This does not need to be expressly written, but can be implied from things like maps, plans, plots or the overall appearance of the development. Here, we do have a brochure, as well as a general setup based on existing houses, from which we can infer that Ella intended to restrict her remaining lots of the 40 acre parcel.

B is wrong. Note that the 60 acre plot was part of a common scheme, and the fact that Beauty and Hunter are not in the same chain of title is meaningless. That is not a requirement for enforcing an equitable servitude. C is wrong. The fact that Beauty's deed did not contain a restriction is irrelevant, as the restriction is in
Beauty's record chain of title.
52007-2008 Law Decks

Fropert9 Law
Falstaff contracts to sell land to Henry for $1 million, to be delivered 90 days from contract signing. At this time, Henry delivers the $1 million. Thirty days later, Henry learns that Hal actually owns the land, not Falstaff. At closing, Falstaff tenders a deed and Henry refuses to accept, noting to Falstaff that he is not the record owner. Falstaff clarifies things by stating that he has been an adverse possessor for 40 years, 20 more than necessary and that everything is cool. Henry still refuses. If Falstaff sues Henry for specific performance, what is the outcome? A. He will prevail, since this is an action in rem, even though Hal is not a party. B. He will prevail, since Falstaff holds "good and marketable title" by adverse possession. C. He will not prevail, as failing to disclose the lack of record title is fraud.

D. He will not prevail, since a court will not make Henry buy a lawsuit, even if it is likely
that Henry would eventually win against Hal.

Answer: D

Historically, an adverse possession acquired title is not deemed to be a marketable title in the eyes of a court. Common law requires that at closing marketable title is passed. Here, the adverse possession title is not marketable as the holder may need to defend his claim in court and a court will not force a purchaser to subsequently take on a lawsuit. When dealing with an adverse possession claim to title; the adverse possessor must first sue for quiet title, in order for the title they hold to be deemed marketable. In light of this, Answer B is clearly incorrect. A makes no sense and is an illogical answer. A 3rd party with an interest in the land could clearly enforce that interest, following Falstaffs claim for specific performance. C is incorrect. Falstaffs action is not fraud, since marketable title can be based on a title obtained by adverse possession. However, Falstaff must first go to court seeking quiet title. Additionally, Falstaff never represented that he had marketable title.

@2007-2008 Law Decks

Fropert9 Law
Suppose Viola owns a 6 story building with a courtyard and street access in Sacramento. Olivia wants to open a boutique and leases the building from Viola for 10 years at a rent of $1,000/month. The lease states in part, "Olivia cannot lease or sublet or assign this lease without first receiving written permission from Viola. Any attempt to sublet or assign the lease without written permission from Viola will result in instant lease termination." Five years into the lease, the boutique is floundering. Cesario comes in and offers to sublet the store and try his hand at movie rentals. Olivia subleases the property to Cesario and he instantly takes possession. A week later, Olivia asks Viola for permission to allow a sub-lease. Viola agrees to execute the written document Olivia provides, as long as Olivia will extend her lease by 5 years. Olivia refuses, but maintains possession of the building. Viola never accepts any subsequent rent from Olivia and brings an eviction hearing against both Olivia and Cesario. The result of this action should be: A. Against Viola, since she has created a restraint on alienation for failing to consent to the sublease. B. Against Viola, the condition consent was in effect a waiver of the lease term requiring written

permission to sublease.

C. For Viola, since Olivia has clearly breached the terms of the lease agreement. D. For Viola, since her original oral consent to sublet was not enough to satisfy the Statute of Frauds.

Answer: C
Here, we have a clear breech of the lease due to Olivia's subletting which was a clear
violation. Generally speaking, the transfer is not automatically void when a tenant

transfers in violation of a lease term, but the landlord has the right to terminate the lease under either the lease terms or a statute. Here, the lease has a forfeiture clause and Viola is free to terminate the lease under this clause. A is incorrect. Clauses restricting assignment or subletting are not considered to be
restraints on alienation.

B is incorrect because a conditional consent is not a waiver in a situation where the condition is never agreed to. D is incorrect since oral consent would be enough, assuming that it was actually made. Here, the consent was conditional on a 5 year lease extension, which never occurred.

2007-2008 Law Decks

Fropert9 Law
Jean owns Aquifer Farm, which he divides into Fleur Farm and Spring Ranch. They are equal in size. Jean deeds Spring Ranch to Manon by warranty deed, which includes an easement over the northern portion of Fleur Farm for access to a river that flows into town. Manon acknowledges the deed and easement, and promptly records. The recording officer maintains a grantor-grantee index that is in alphabetic order only, and does not provide a tract index. Manon never uses the easement until after Marcel takes Fleur Farm. Manon continues to use the easement for another year. Note, however, that Marcel has no idea that Manon had used or currently uses the easement. A month later Marcel finds out about the easement and objects to it, but Manon just ignores Marcel thinking he is an old fool. Marcel sues Manon for quiet title to prevent her use of the easement, since she has reasonable access to a road on the opposite border of Spring Farm. Assume that Manon is successful. It is likely because: A. Since there was no tract index, Marcel must inquire regarding the riparian rights of owners abutting his property. B. Since there was common grantor for Marcel and Manon, whose covenants of title run with

the land, Marcel is stopped from denying Manon's title.

C. An easement is a legal interest that runs with the land it is attached to, and, therefore, binds successive owners of the servient estate regardless of notice. D. The easement is a legal interest in Marcel's chain of title even though there is no tract index.

Answer: D
If Manon wins, it is due to the fact that the easement is a legal interest in the chain of title held by Marcel. The present facts are a recording act problem.

Note: Answers A, B and C all misstate the recording act and the applicable law. Even if Marcel has no actual or inquiry notice, he has constructive notice due to the recorded easement by the grantor, Jean, regardless of the absence of a tract index.

2007-2008 Law Decks

Fropert9 Law
Ethan and Joel are brothers and hold land as joint tenants with rights of survivorship. Ethan moves out after conveying his interest to Wes, his adopted brother, by quitclaim deed. Wes never records the deed transfer. Two years later, Joel does record, leaving Sophia, his adopted daughter, as the only surviving heir to his estate. After Joel's death, Ethan asks Wes to return the deed and give up his interest in the land. Wes agrees and turns over the deed to Ethan, who burns it on Joel's funeral pyre. Who has title to the land? A. Wes and Sophia as co-tenants B. Ethan and Sophia as co-tenants C. Ethan as sole owner
D. Wes as sole owner

Answer: A
Wes because Ethan conveyed an interest in him and Sophia takes by inheritance, therefore, Answer C is clearly wrong. B is wrong as Ethan transferred his interest to Wes, and the destruction of the deed has no effect on this interest. Wes must deed an new interest back to Ethan for the transfer to be effective. D is wrong because when Ethan conveyed to Wes, the original joint tenancy was destroyed and the interest became a tenancy in common, which has no right of survivorship. In light of this, when Joel died the interest just passed to Sophia, Joel's heir.

2007-2008 Law Decks

Fropert Law
Enrique and Wendy are a married couple who own Casa Riverside as a tenancy by the entirety. Enrique decides to transfer his interest to Paul by a quitclaim deed. When Enrique does this, Wendy is away visiting her mother. When she gets back, she decides to mortgage her interest in Casa Riverside to Sue. Assume that Wendy dies and leaves her interest to Wences. Who in actuality will receive Wendy's interest? A. Paul B. Wences C. Enrique D. Wendy's heirs

Answer: C
A tenancy at will comes with a right of survivorship which cannot be defeated by a single spouse. Therefore, Wendy's husband Enrique receives her interest. In light of this, Answer B is clearly incorrect. A is wrong. Paul can not have any interests because Enrique cannot convey an interest to Paul, as this is an tenancy by the entirety. D is wrong as Wendy's husband, Enrique, succeeds to her interest. Additionally, Casa Riverside is not part of Wendy's estate, making it unavailable to Wendy's heirs.

2007-2008 Law Decks

Property Law
Oscar seeks to buy a house from Tyrone in a new housing development on Tony Town Hill. Tony Town is a group of subdivisions built by Tyrone with 4 types of homes (Art Deco Condo, Cool Ranch, Spanish Hacienda and McMansion), each of which have different prices and sizes within those groups. Oscar narrows his choice down to two available Spanish Haciendas. Both are identical and cost $1 million each, but Oscar cannot decide which suites him best: the one facing north, or the one facing south. Tyrone (a veteran professional in land development) prepares a deed describing the homes, but not definitely stating which one the deed is directed to, so that once Oscar decides he can quickly fill this in and be done. Tyrone tells him to fill in the address, record the deed and give him a call. Tyrone delivers this deed to Oscar. Oscar decides that he actually likes a third Spanish Hacienda with a pool, offered at a price of $1.5 million. He fills this address in his deed and records. Two weeks later, Oscar offers this house to Agatha for $1.25 million. A title search shows that Oscar has title free of any encumbrances. Agatha agrees to purchase and records her deed after paying with cash. Assume that Tyrone brings action against Agatha for quiet title. Which is the most probable reason for a judgment in Agatha's favor? A. Tyrone is guilty of unclean hands. B. Once Agatha recorded the deed form Oscar, Tyrone could not attack the validity of the recorded deed. C. Evidence of Tyrone's instructions to Oscar cannot be introduced under the Statute of Frauds. D. Agatha is a bona fide purchaser for value. Therefore, Tyrone cannot rely on the agreement with Oscar regarding filling in the blank deed.

Answer: D
Agatha, in the present case, has detrimentally relied on an agency in Oscar offered by Tyrone. Therefore, Tyrone cannot deny that the agency was not valid and the deed recording was not proper. As Tyrone cannot exercise this argument, the transfer to Agatha is valid and proper.
A is incorrect since the actions of Tyrone were not inequitable, making an unclean hands

argument inapplicable.
B is incorrect. The recording of a deed is not such that the validity of the deed can no longer be attacked once recorded. A recorded deed can, in fact, be attacked for validity.

C is incorrect because the Statute of Frauds is not applicable to the present facts. The enforceability of the conveyance would allow the introduction of Tyrone's instructions to Oscar.

(D2007-2008 Law Decks

Fropert9 Law
Suppose that Higuchi owns a 100 acre farm in Kansas City and the city annexes his land to build a residential shopping complex, and rezones it from farming to residential. This project is spearheaded by Hattie, a builder who buys the 100 acres from Higuchi on behalf of the city for a fair price. She files a subdivision plan to build 200 homes and 2 "city parks" and begins selling lots like hotcakes. In each deed of transfer is a covenant, restrictions and conditions relating to the nature of homes in relation to the subdivision. One covenant bans any commercial or industrial activity. Hattie sells all but 10 lots and 1 of the designated parks. All of her sales include the covenants. The remaining 10 lots are sold to CoCompany, without any of the deed restrictions. They resell the 10 lots (absent deed restrictions) and the remaining park to Clear & Free, a nuclear power company. The final sale to Clear & Free was made 2 years after Kansas City's purchase of land from Higuchi. Before the nuclear plant begins trucking in uranium, suppose that Kansas City names the remaining park "Reader's Memorial Park." The city solicits bids from local landscapers to decorate it with reading nooks for people to use on their lunch hour. Clear & Free learns of the city's plans and brings action for quiet title. How should a trial court rule on this action? A. For Kansas City, since Clear & Free had constructive notice that the parcel was to be used for park purposes.
B. For Clear & Free, since its deed had no mention of the park restriction.

C. For Kansas City, since Hattie initially dedicated the land for use as a park and the city accepted this plan. D. For Clear & Free, because laches applies to the present facts since Kansas City waited too long (2 years) before doing something with the land.

Answer: C
Once the subdivision plan was recorded including the park provisions, this is deemed an offer, and Kansas City's action in soliciting bids and naming the park is an acceptance. Therefore, Kansas City has good title.
Laches would not be a factor over a 2 year period, so D is incorrect.

Clear & Free had constructive notice of the intended proposed use of the land as a park through the chain of title. Therefore, Answers A and B are wrong.

2007-2008 Law Decks

Fropert9 Law
Antwone conveys to Maya and Patricia a huge hunk of land known as Maya Hall, as cotenancy in equal shares with right of survivorship. Maya conveys, "To Cindy, my undivided, one-half interest in Maya Hall." Maya dies shortly after. In an action between Patricia and Cindy in which both claim sole title to Maya Hall, Patricia can only prevail if: A. The co-tenancy created in Maya and Patricia was a tenancy by the entirety. B. Patricia was once the sole owner of Maya Hall. C. Maya and Patricia owned Maya Hall as tenants in common. D. Patricia had knowledge of the conveyance prior to Maya's death.

Answer: A
Here, we are faced with a vague description of the conveyance given in the problem, and might be dealing with either a joint tenancy with a right of survivorship, or a tenancy by the entirety. If we assume for a moment that we are dealing with a tenancy by the entirety, then further conveyance from Maya to Cindy does result in a severance and upon Maya's death, Patricia holds the land as the sole owner. In contrast, if we are originally dealing with a joint tenancy, then Maya's conveyance does not serve to sever the conveyance, resulting in a tenancy in common with Patricia and Cindy. B is irrelevant because the form of concurrent ownership determines the rights of the current parties, not the form of the prior holding. D is irrelevant because the present form of holding is what controls, and the knowledge (or lack of knowledge) of the subsequent holding is irrelevant. Under Answer C, Patricia and Cindy hold an undivided 1/2 interest, making this clearly an incorrect answer.

2007-2008 Law Decks

Fropert9 Law
Murray and Rodney are adjoining land owners. Murray decides to develop a campground on CampySwamp, and submits a development plan outlining the camping areas. The plan has access via a road around CampySwamp, which Murray is going to construct and turn into a public road that he refers to as CampySwamp Road. The plan is recorded. Rodney owns GolfClub Acres, which only has small dirt road as access to main roadways. Rodney sells GolfClub Acres to Kathleen. Kathleen's deed has similar language to prior recorded deeds, but contains additional language directed to road access which states, "bordered on the south by the new CampySwamp Road, soon to be constructed by Murray, CampySwamp's owner," and references Murray's plan for a public road. Kathleen seeks a building plan giving her access from Murray's new road to GolfClub, bypassing GolfClub's poor dirt road. Murray objects, stating that he never gave Rodney any rights originally, and that Kathleen, therefore, has no rights to use the new CampySwamp road. Murray's best argument is: A. The Statute of Frauds prevents the introduction of evidence that might prove the necessity for Kathleen to gain access to Murray's road on CampySwamp. B. Kathleen would be unjustly enriched if she was allowed to use CampySwamp Road.
C. Kathleen has to await the completion of CampySwamp Road and the opening of the road as a public

street used in gaining access to CampySwamp, as Kathleen has no easement - related rights to use the
CampySwamp road. D. Kathleen could only have use rights of CampySwamp Road by necessity, and the facts indicate that the access to CampySwamp Road is not an absolute necessity.

Answer: C
Kathleen's rights clearly depend on the existence of a public street. Here, she has street access via the poor existing dirt road, so Kathleen has no additional rights to use Murray's new road.
A is wrong since a Statute of Frauds issue is present. Here, we have no issue of a

conveyance of rights in road access by a person who was in the position to actually convey these rights.
B is wrong. Unjust enrichment does not occur in a situation where a person gains access to a public street. D is wrong since, if a public street exists, Kathleen does not need to assert an issue of necessity. Anyone is eligible to access a public street.

2007-2008 Law Decks

Fropert9 Law
Elkins leases 200 square feet from Fisher for a three year period for his commercial business. Fisher retains an option to renew for 3 additional years at the end of the lease should he like. Fisher updates the property for use in his tropical fish pet store by having $100,000 worth of giant salt-water fish tanks and their systems installed. During year 2 of Fisher's lease, Elkins borrows $50,000 from WoIrdBank, giving them a mortgage on the place, and does not inform Fisher. During year 3 of Fisher's contract, Elkins decides not to renew and begins removing his improvements. Elkins defaults on his mortgage and WorldBank begins foreclosure hearings 2 months before Fisher's lease will expire. WorldBank brings action to enjoin removal of Fisher's giant fish tanks and machinery, naming both Fisher and Elkins as defendants. Assume that a court refuses to grant the injunction; it is likely because: A. Under the Statute of Frauds, WorldBank cannot assert an interest in improvements that Fisher put into the place. B. Absent an agreement to the contrary, commercial tenants are free to remove any improvements they may have made to the property during their lease. C. A fixture installed for the purpose of a trade by a commercial tenant, may generally be removed so long as the tenant pays for any damages incurred by the removal. D. Fisher had no notice of the mortgage by Elkins.

Answer: C
As long as it is not an integral part of the premises, a commercial tenant may remove things he installed after moving in, assuming that these things are "trade fixtures" associated to the trade that he is practicing (i.e., giant salt-water fish tanks for a tropical fish pet store). A tenant must also pay any damages associated with the removal. A is wrong as the Statute of Frauds allows a mortgagee to claim an interest in property that has become a part of the premises. B is wrong. Generally speaking, personal property affixed to a location becomes a fixture of the property, with the exception of the trade fixtures that may be removed. D is wrong because as WorldBank (the mortgagee) has the same rights to the property as the original owner, regardless of the fact that Fisher (the tenant) had no knowledge of the mortgage.

2007-2008 Law Decks

Property Law
Pasquale enters into an option agreement to buy Philip's house and land. Pasquale is to pay a fixed monthly rent, plus taxes and maintenance charges for 5 years. This option can be exercised at any point during that 5 year term by giving Philip 30 days notice. Three years into the lease, Philip dies, leaving his estate to Rick, his brother. Just before Philip's death, Pasquale assigned his interest in land to Ashley using a written agreement. The option terms are set forth, but Pasquale failed to mention the payment of the real estate taxes. Months later, Rick is informed that there is a tax lien on the house. Rick pays off the lien and sues Ashley. Ashley decides to walk away from the land. Can Rick sue Pasquale for this breach of the lease? A. No, because Pasquale is not a tenant any more. B. No, because Pasquale was not a tenant when Rick gained interest in the land. C. Yes, because Pasquale's assignment to Ashley does not extinguish his liability. D. Yes, because this problem was actually caused by Pasquale's failure to include the tax clause in the assignment to Ashley.

Answer: C
The tenant-assignor (Pasquale) is still in privity of contract with the lessor (originally Philip, and now Rick) and is liable for the rent reserved in the lease. A and B are incorrect because the status of Pasquale as a tenant is meaningless when applied to Pasquale's liability. D is incorrect. Pasquale's failure to include the tax term is immaterial when viewed against his liability for the rent as the assignor under privity of contract

2007-2008 Law Decks

Froperty Law
Alan conveys his house on Elm Corner to Chris for life, then to Winnie and Robin as joint tenants with the right of survivorship. On New. Year's Eve, Chris, Winnie and Robin are driving along the highway on their way home from a party and hit a large donkey. An ambulance takes them all to the hospital. All of them are alive upon arrival. Robin dies at 1:20 AM, Winnie dies at 1:21 AM and Chris dies at 1.22 AM. There is no uniform simultaneous death act or any local modification of that act. Who takes the house on Elm Corner based on the prior facts? A. Chris's heirs B. Winnie's heirs C. Robin's heirs
D. Alan or his heirs (assuming that Alan is dead)

Answer: B
Winnie and Robin held an indefeasibly vested remainder as joint tenants. This remainder was not contingent on them surviving Chris. When Robin died before Winnie, Winnie took the property pursuant to the right of survivorship. When Winnie died, the remainder passed to her heirs. When Chris died, Winnie's heirs took the house on Elm Corner pursuant to the death of the life tenant. A is wrong because Winnie had a life estate with no potential to pass any interest after her death. C is wrong since Winnie survived longer and, therefore, owns all of the remainder that can pass to her heirs. Robin upon death has nothing, as it all went to Winnie. D is wrong. Alan has no potential interest in the house on Elm Corner as he has granted a life estate and an absolutely vested remainder.

2007-2008 Law Decks

Fropert9 Law
John leaves Rocky Top Mountain to Dave, his heirs and assigns (as long as the property is used exclusively for raising cattle), then to John's grandson Marcus. The remainder of John's property passes through his will to Marcus. Ten years after John's death, Dave begins to strip mine coal on Rocky Top Mountain. Marcus brings suit to quiet title and Dave counterclaims. Who should prevail? A. Dave, as the rule against perpetuities results in the condition imposed on his interest to be void. B. Marcus, since the condition imposed is valid and Marcus takes subject to the violation of this provision. C. Marcus, pursuant to the Residuary Clause.
D. Dave, as the condition imposed on his use was a restraint against alienation.

Answer: C
Marcus wins, as Rocky Top Mountain passes to him under the Residuary Clause. Note that the attempt to give Marcus an executory interest violates the rule against perpetuities and is void. Therefore, Answer B is clearly incorrect. A court will read the conveyance, absent the language directed toward the executory gift, leaving the possibility of reverter in the original grantor (John). Because of this, Answer A is clearly incorrect as Marcus succeeded to John's interest as grantor and, due to the residuary clause of the will, will prevail and hold Rocky Top Mountain. D is wrong because the will contains no restraint on alienation.

2007-2008 Law Decks

Fropert9 Law
Glen rents out a 2 story building from Raul. He uses the top floor for office space and subleases the remaining parts of the building to others. Glen pays Raul $2,000/month. Glen's 2 sub-tenants pay Glen $600/month each. After renting out the building for 5 years, Glen and Raul agree to a sale where Glen buys the building from Raul for $96,000 (to be paid in monthly installments of $2,000 for 4 years). Additionally, the parties agree that title will remain in Raul's name until Glen has paid $48,000 of the total price, upon which title will switch to Glen. Glen redecorates his office, spending $40,000, and over the next year gets new tenants that he charges $700/month. Two years after the agreement, Glen demands that Raul convey the building to him via deed. Raul refuses, claiming that there was no agreement for sale between he and Glen. Glen brings an action against Raul for specific performance and Raul stands behind the Statute of Frauds as a defense. Assume that Raul wins. It is likely because: A. Glen never obtained Raul's approval before making improvements to the building. B. There was an original violation of the Statute of Frauds, and this violation was incurable.

C. Glen's actions of making monthly payments and improvements are consistent with a tenant
using the place for commercial purposes. D. Raul has received no unconscionable benefit that would entitle Raul to equitable relief.

Answer: C
Under the Doctrine of Part Performance, a contract that is oral in nature can be enforced so long as facts are provided that unequivocally prove the existence of the contract. Here, however, Glen's monthly payments to Raul are the same amount as he was paying as a tenant, and the improvements he made, are in keeping within the actions of an ordinary tenant. A is incorrect because Glen does not need Raul's approval to make the improvements B is incorrect because if Glen had taken some actions that clearly indicate the presence of an oral contract, the Doctrine of Part Performance would have applied. As set forth in the correct answer, however, there is no indication that Glen's actions were in any manner indicative of part performance, and beyond that which an ordinary tenant would do.

for the purposes of part performance.

D is not a good answer. The benefit that might have been garnered by Raul is irrelevant

2007-2008 Law Decks

Fropert9 Law
Pasquale enters into an option agreement to buy Philip's house and land. Pasquale is to pay a fixed monthly rent, plus taxes and maintenance charges for 5 years. This option can be exercised at any point during the 5 year term by giving Philip 30 days notice. Three years into the lease Philip dies leaving his estate to Rick, his brother. Just before Philip's death, Pasquale assigned his interest in land to Ashley using a written agreement. The option terms are set forth, but Pasquale failed to mention the payment of the real estate taxes. Months later, Rick is informed that there is a tax lien on the house. Rick pays off the lien and sues Ashley. A court will find for: A. Ashley, as under the assignment from Philip she is not liable for the real estate taxes. B. Ashley, as the agreement to pay taxes is a collateral agreement that does not run with the land. C. Ashley, since the tax agreement can only be a covenant running with the land if it is

originally expressed in the original agreement.

D. Rick, as property tax payments touch and concern the land.

Answer: D
An assignee such as Ashley is in privity of estate with the lessor (Rick) and is liable for any covenants in the original lease that runs with the land. In light of this, Answer A is clearly wrong. B and C are wrong because an agreement to pay taxes touches and concerns the land, and thus runs with the land.

2007-2008 Law Decks

Fropert9 Law
Von wishes to give some of his property to Maria for use as an outdoor park. Von hands Maria a generic pair of deed grants, one which reads, "My property known as Musical Arbor, consisting of 10 acres of plowed land and an amphitheater." The second deed is, "Four acres of wooded land adjacent to Musical Arbor for use as an outdoor park." Both deeds contain standard warranties, including a covenant against encumbrances, the right to convey, and the right of quiet enjoyment. Von tells Maria that he is giving her the property, but wants to keep the deeds until his attorney can review them. Maria agrees to this, and gives the deeds back to Von so that his attorney can look them over. Unfortunately, Von is eaten by a horse the following day and dies, never having spoken with his attorney about the deeds to Maria. While dealing with Von's estate, Maria learns that Von had, in fact, deeded a portion of Musical Arbor years before. This purchaser, Rupert, was given an easement across Von's land to access the deeded portion of the land that he received. (Note: There is no evidence that Rupert had ever used the easement, and there exists no trail or evidence of his use.) Von's daughter, Louisa, introduces evidence stating that she (Louisa) has a right of first refusal over Musical Arbor. In light of this, a court will: A. Void the deed to Maria.

B. Require that Maria convey to Louisa if she tenders market price for Musical Arbor.
C. Not effect the deed held by Maria, but Louisa could potentially go after her father's estate. D. Not affect the title held by Maria, since Louisa has no claim against her or Von's estate.

Answer: C
If a "right of first purchase" is found, then a court will deem this right a promissory restraint, thereby, allowing Louisa to potentially go after Von's estate. Maria's interest is not affected, unless Maria had knowledge of this right in Louisa when Maria acquired the estate. Thus, Answers A, B and D are incorrect.

@2007-2008 Law Decks

Property Law
Friedrich and Liesl buy Gazebo Acres just before they marry. They hold title in both of their names. After 10 years of marriage, Gazebo Acres is still a disheveled vacant lot with nothing but an old gazebo on it, their dreams of a prewar mansion never having come to fruition. Friedrich decides that their son, Kurt, would have better luck with the ladies some day if he had some land to show for himself. Friedrich conveys a 1/4 interest in Gazebo Acres to Kurt. One month later, Friedrich realizes that he was being a chauvinist and conveys a 1/4 interest to their daughter, Marta, as well. Who owns Gazebo Acres? A. Friedrich and Liesl own Gazebo Acres, with right of survivorship, and Kurt and Marta have no rights. B. Kurt has a 'A interest, Marta a % interest and Liesl a 1/2 interest.
C. Kurt has a 1/4 interest, Marta a % interest and Liesl 1/2 interest with right of survivorship.

D. Liesl owns Gazebo Acres outright.

Answer: B
First, we must presume that Friedrich and Liesl are tenants in common, and not joint tenants. To be joint tenants there must be an express creation of such tenancy, which is absent here. Neither are we dealing with a tenancy by the entirety, since Friedrich and Liesl were not a married couple at the time of purchase. Each tenant in common has an undivided interest, which can be conveyed by an inter vivos transfer. Here, Friedrich has a 1/2 interest to start, and following the conveyances to Kurt and Marta, he has nothing left. Making, Answer A incorrect. There is no right of survivorship in a tenancy in common; therefore, C is also incorrect. D is not supported by the facts presented here.

2007-2008 Law Decks

Property Law
Mame is affluent and in her will dated March 1, 1950 leaves everything to Patrick. Mame dies in 1991. When Patrick meets with Mame's executors, Patrick learns that 2 pieces of property were deeded to others: Rough Ride Ranch to Brigitta in 1975, and Vino Vicarage to Rosalnd in 1990. The deed to the ranch includes the clause: "for as long as the property is used to house sick parakeets. If the property is used for any other purpose, then it goes to the Able Aviary Association." The deed to Vino Vicarage includes: "as long as the vicarage is used as a headquarters in campaigning for a tax cut for the wealthy

to pass by 2004. However, if the tax cut is passed before 1990, then the property should pass to Patrick at that time." Mame died leaving only one son, Fred, who was a bastard who never called his mother and
so received nothing at all in Mame's will. This is a common law jurisdiction, and the state's probate laws provide that future interests, or estates in real property, may be passed by will or descent in the same manner as present or possessory interests. (Note: Neither Brigitta, nor Rosalind qualify as a charitable institution.) In 2005, Brigitta approaches Fred, asking him to join in the sale of the ranch to Yuki for use in constructing a fancy fast food restaurant. They enter into a contract for sale, but just before closing Brigitta decides against the sale. Yuki sues both Brigitta and Fred for specific performance. The requested relief will be: A. Denied, because the Able Aviary Association did not join in the sale.

B. Denied, because Patrick never joined in the sale.


C. Granted, because Brigitta has the power to sell her interest. D. Granted, because Brigitta and Fred's interest merge, resulting in a fee simple interest that they are free to sell.

Answer: B The interest of the Able Aviary Association is void as violative under the rule against perpetuities, because it might vest beyond lives in being plus 21 years. Note: While there is a charity-to-charity exception, it is inapplicable here since Brigitta is not a charity. In light of this, Mame retains a possibility of reverter which passes to Patrick upon Mame's death. In light of this, Answers A and D are clearly wrong. C is wrong. While Brigitta is free to sell the interest she has, it is not a fee simple, but an absolute interest directed toward the entire property.

2007-2008 Law Decks

Fropert9 Law
Sadie owns Eatmore Estate in Rhode Island. In 2000 she executes the following deed: "All of Eatmore to Jude, my best friend and confidant." Sadie puts the deed in the kitchen cupboard and tells her trusted attorney, Wallace, to turn the deed over if Jude survives Sadie. By 2004, Sadie hates Jude, and gives Eatmore to Julia in a quitclaim deed for $1,000. Sadie told Julia about the earlier deed and further told her that she would take great pleasure in burning it that evening. Sadie never did burn it, however. She dies in 2005. Jude is the only surviving heir, and Wallace hands the deed over to Jude as instructed to do so by Sadie when she was alive. Jude builds another house on Eatmore and moves in. The house costs $100,000 to build, but it is an ugly house and adds no value to the land. The deed from Sadie to Julia was: A. Effective as a conveyance of title when delivered. B. Effective upon recordation, to cut off Jude's interest in the property.

C. Ineffective against Jude, because Julia has knowledge of the deed from Sadie to
Jude. D. Ineffective against Jude, because Sadie took Eatmore by quitclaim deed.

Answer: A
The deed conveyed title to Julia instantly upon delivery since it was a quitclaim deed and a quitclaim deed delivers whatever rights the grantor has to the grantee. Sadie failed to deliver the original deed to Jude, making the deed ineffective against Julia. Upon transfer of the quitclaim deed, Julia took the property without having to worry if Jude had a greater right. Julia's acquiring of title occurred, therefore, in 2004 when she received her deed. B is incorrect in light of the correct answer, as title was conveyed to Julia upon delivery of the deed. Additionally, an answer such as B, relating to the recordation, is irrelevant. C is wrong. The earlier deed is irrelevant since it was not an effective transfer (never being properly delivered) and Sadie had no intention of relinquishing control upon granting it to Jude.

A quitclaim gives Julia all the rights of Sadie (she stands in her shoes) namely, a fee
simple absolute. Note: Because Jude's warranty deed was never delivered, it is meaningless to this analysis.
2007-2008 Law Decks

D is wrong as the quitclaim deed of Julia is quite effective against Jude's warranty deed.

Fropert Law
Raphael buys land in 1975 from Michael by warranty deed. Raphael does not know that Michael really doesn't own Purple Plume Hill, but that it is actually owned in fee simple by Norman. After buying from Michael, Raphael builds a house and digs a pool on the northern tip of Purple Plume Hill. The remaining parts of the property are vacant. In 1980, Raphael builds a garage on a 10 acre plot that he thinks is part of Purple Plume Hill. In reality, these 10 acres belong to Leo. Leo lives out of state and doesn't know that a garage was built on his property. In 1990, Leo dies, having not been back to Purple Plume Hill since Raphael built the garage. Leo leaves his property to Lisa via a will. In 2000, Lisa dies leaving all her property to David. Adverse possession period in this state is 20 years. Raphael brings an action for quiet title on Purple Plume Hill. How should a court decide? A. Raphael is the owner. B. David is the owner of Purple Plume Hill since, in relation to him, adverse possession only began in 2000. C. David owns Purple Plume Hill since Raphael only occupied a portion of it. D. David owns Purple Plume Hill assuming that Raphael never paid the property taxes on it.

Answer: A
Using an application of the rule of constructive adverse possession, Raphael is the owner. The rule states that where the claimant takes possession of a portion of land under the belief that he has color of title, if the adverse possession period is met, a court will find the person in possession has title to the entirety. In light of this, Answer C is clearly incorrect. B is incorrect because a transfer in ownership does not interrupt the statutory period for adverse possession (i.e., tacking is permissible). D is incorrect because only a minority of states require that the adverse possessor pay taxes on the property in question.

2007-2008 Law Decks

Property Law
Lamar wills Hideout Creek to his daughter. The devise states, "To my feisty daughter, Madn, for life, the remainder to each of her children living at the time of my death." The will further provides that, "No child of Marin's may mortgage or sell Hideout Creek until at least 35 years of age. If any of them attempt such action, their interest terminates and is divided among the remaining children, who are alive at the time of my death, in equal shares." Lamar dies choking on a pretzel. Rusty, Marin's oldest son, wants to sell his interest, but Mindy (Marin's daughter), as well as Marin's other 3 remaining children, object to the sale. Assume that Rusty is under 35 years of age. If Rusty sues, stating that the restriction is invalid, what is the likely outcome? A. The suit would be dismissed, since Rusty is under 35 and has no interest in the property. B. The gift to Marin's children violated the rule against perpetuities and is void.

C. The restriction would be stricken, as it is violative of the alienation of real property.


D. The restriction is proper, since Rusty has nothing more than a defeasible estate.

Answer: C
Note that we are dealing with 2 gifts to children, a vested remainder that follows Marin's life estate, and an executory interest in the other children if one of them tries to sell while under 35 years of age. This second restriction is an attempt to retain alienation and is void, making Answer D clearly incorrect. A is further incorrect, as Rusty, Mindy, and the other 3 children have interests that are vested at the time of Lamar's death. B is wrong for a similar reason. The children's interests vested and their class closed at the time of Lamar's death, making their interests permissible when viewed under the requirements of the rule against perpetuities.

2007-2008 Law Decks

Fropertg Law
Jack owns Stone's Skip, a large rugged piece of land. Stone's Skip has a river running through it with fish, but it is not navigable. The river cuts Stone's Skip in half, making a northern portion and a southern portion. Jane is interested in the northern portion for raising wild dogs. Jack and Jane agree on a price and Jack conveys, "the northern part of Stone's Skip, as separated by the river, to Jane." If one was to determine the exact boundaries of the land owned by Jane, which is the most accurate description? A. Jack owns all of the stream. B. Jack and Jane own 1/2 of the stream each. C. Jane owns the entire stream, since it is necessary to interpret the language of a deed in the most stringent manner against the seller.
D. The description is invalid; therefore, it is impossible to determine ownership interest

in Jack and Jane.

Answer: B Each party owns 1/2 of the stream bed, since, generally speaking, when dealing with a stream such as this, a court states that Jack and Jane would each split it down the middle. In light of this, Answers C and A are clearly wrong. Answer D is incorrect for asserting that the description of the land is not valid. In fact, it is valid enough for a court to determine that Jack and Jane share the stream down the middle.

2007-2008 Law Decks

Fropert9 Law
Roman leases Metro Pond to Mitch for 99 years. Mitch builds a casino on Metro Pond, and sells the casino to Lorna, with Roman consenting to the assignment of the lease. Ten years into the lease, Roman conveys the property to his 2 sons, Harry and Lance. A month later, Roman has a huge fight with his sons and demands that they return the deed. The sons do so and Roman destroys the deed. Harry learns that he is dying and conveys to Tim "any interest I may have in Metro Pond." He then dies. Who owns the reversion in Metro Pond and the casino? A. Lance B. Roman C. Lance and Tim as tenants in common
D. Lance and Tim as joint tenants

Answer: C
Following the conveyance of a co-tenant's interest in joint tenancy property, the joint tenancy is severed and the remaining interest becomes a tenancy in common. In light of this, Answer D is clearly incorrect. A is incorrect. The joint tenancy with right of survivorship is severed before Harry dies. B is incorrect as the physical return of the deed does not confer rights back to Roman. Harry and Lance would have to re-deed the property to Roman for Roman to now hold it, which they did not do.

2007-2008 Law Decks

FroFert9 Law
Suppose that Erica owns a house on Kiawah Island and rents to Julian who intends to use the back of a house to live and the front to sell seashell art. Julian puts in a storefront, a seashell washing machine, a wet saw, a wall-mounted glue gun, etc. . . . Months later, Erica mortgages the house to Cat& Bank as part of a loan, which is recorded. Erica never tells Julian about it. Erica defaults, and begins a foreclosure hearing, at which point Julian learns of the loan. He begins removing the fixtures that he added and Cat& Bank seeks an injunction to prevent this. Under these circumstances, the court should deny Cat& Bank the injunction because: A. The agreement between Erica and Julian has no provision regarding this, and Julian is, therefore, allowed to remove anything he might have added. B. Caton Bank does not have a security interest in Julian's personal property. C. The equipment was installed for Julian's benefit with no intention that it would stay when he left. D. Julian had no notice of the mortgage.

Answer: C
Since the equipment was installed without an intention to benefit the landlord; Julian is free to remove it when the lease time terminates as long as he repairs any damages caused by the removal. A is incorrect. Personal property attached to real property may become a fixture (and, thus, part of the realty that is not to be removed) if the one who attached it intended it to become part of the real property. Note: It is presumed that items used in trade or business, are not intended to be permanent fixtures. B is incorrect. Julian's fixtures (the seashell washing machine etc. . .) are not considered part of the real property, only because they are considered trade fixtures. D is immaterial since notice to the tenant is not necessary.

2007-2008 Law Decks

Fropert9 Law
Ethan and Joel are brothers and hold land as joint tenants with right of survivorship. Ethan moves out after conveying his interest to Wes, his adopted brother, by quitclaim deed. Wes never records the deed transfer. Two years later, Joel does record, leaving Sophia, his adopted daughter, as the only surviving heir to his estate. After Joel's death, Ethan asks Wes to return the deed and give up his interest in the land. Wes agrees and turns over the deed to Ethan, who burns it on Joel's funeral pyre. Ethan then conveys a deed to Steven. The applicable recording act states: "A conveyance of an interest in land is not valid against any subsequent purchaser for value, without notice thereof, unless the conveyance is recorded." Stephen, runs down to the town recording office and records before the ink is dry. In a lawsuit between Stephen and Wes, Stephen prevails because:
A. Stephen recorded his deed.
B. Wes's interest terminated when he returned the deed to Ethan.

C. There has been no severance in the chain, as Wes was adopted by Joel in a court of law.
D. Stephen is a bona fide purchaser.

Answer: D
Stephen wins since he is a bona fide purchaser who recorded. Answer A is a close to right answer as it addresses the recording act, but the recording act requires Stephen to take without notice and pay value i.e., be a bona fide purchaser. Therefore, D is a better answer than A due to the bona fide purchaser issues. Answer A is further wrong as the recording act is a "notice" statute, therefore, Stephen would have prevailed against Wes even if he had not recorded. B is wrong because Wes's return of the deed to Ethan has no chance on the state or status of his interest. C is a bogus answer that should be avoided at all costs for being nonsensical.

2007-2008 Law Decks

Fropert9 Law
Blair and Tony have lived together for years but are not married. Suppose that Connecticut does not recognize common law marriage, but does have status prohibiting discrimination based on marital status. One day, Blair goes out and stops at the local state-run casino during her lunch break. She drops a nickel in a slot machine and wins the $2 million jackpot. She uses the proceeds to buy Vaux Hall, which has a great Art Deco mansion on the property. Blair and Tony record the title and hold Vaux Hall as joint tenants. Tony also decides to make a million gambling, but promptly looses $500,000 playing three-card-monty. He needs to raise this cash to pay off his bookmaker or his knees will be broken. Tony manages to convince Bank-a-Lot Bank to enter into a mortgage against Vaux Hall with his signature alone. (Blair never signs the mortgage.) Seven months later, Tony dies in a tragic supermarket accident involving a jar of pickles and some pipe cleaners. The mortgage remains unpaid. Tony has no will and his only heir is his brother. Suppose Connecticut is a "title theory" mortgage state. What is the state of Vaux Hall? A. Blair and Bank-a-Lot have title because the joint tenancy would have been severed. B. Blair has title since the joint tenancy was severed. C. Tony's brother has a legal interest in Vaux Hall. D. The result would be the same as under a "lien theory," since the mortgage theory of the jurisdiction has no effect on whether joint tenancies are severed.

Answer: A A mortgaging property in a "title theory" state severs a joint tenancy; since in a "title theory" state a mortgage is deemed an actual transfer of title to the property and not simply a lien on the property. Once a joint tenant mortgages the land, the legal title is transferred to the mortgagee (Bank-a-Lot) while severing the joint tenancy between Blair and Tony. Following the severance in light of the mortgage transfer, Tony and Blair hold title to the land as tenants in common. B is wrong because, as explained, the joint tenancy was severed. C is wrong. Tony's brother, at best, could inherit an equitable interest. In a mortgage title theory state, the mortgage of property results in the borrower retaining an equity of redemption. Following Tony's mortgage, he can again become a title holder with Blair by repaying the mortgage and, therefore, becoming a tenant in common with Blair again. Here, Tony's brother could, therefore, have (at best) inherited the right to continue repaying the mortgage and holding Vaux Hall with Blair as a tenant in common. Note that

this would be an equitable interest, not a legal interest in the property.

D is wrong because a state's mortgage theory has a great effect on the issue of severance: a mere lien on the property (lien theory) does not cause a severance, but a transfer of title (title theory) does.
2007-2008 Law Decks

Property Law
Sadie owns Eatmore Estate in Rhode Island. In 2000 she executes the following deed: "All of Eatmore to Jude, my best friend and confidant." Sadie puts the deed in the kitchen cupboard and tells her trusted attorney, Wallace, to turn the deed over if Jude survives Sadie. By 2004, Sadie hates Jude and gives Eatmore to Julia in a quitclaim deed for $1,000. Sadie told Julia about the earlier deed and further told her that she would take great pleasure in burning it that evening. Sadie never did burn it, however. She dies in 2005. Jude is the only surviving heir, and Wallace hands the deed over to Jude as instructed to do so by Sadie when she was alive. Jude builds another house on Eatmore and moves in. The house costs $100,000 to build, but it is an ugly house and adds no value to the land. If Julia prevails over Jude: A. Julia can keep the house and recover the reasonable rental value for Jude's use and possession.
B. Julia can keep the house, but reimburse Jude for $100,000 in construction costs. C. Jude is entitled to remove the house if he likes.

D. Jude is entitled to a lien on Eatmore Estate for $100,000, minus the fair rental value for the house.

Answer: A Jude is a trespasser and Julia can keep the house and receive rental income. Jude's good faith belief is meaningless, he remains a trespasser. Under many jurisdictions a trespasser is further liable for the reasonable rental value, making Jude liable in the present fact pattern for rental fees as well as allowing Julia to keep house. B is wrong as a good faith improver of property can occasionally recover some amount of the improvement so long as they add value to the property. Jude's house added no value, so he is unlikely to recover any amount. C is wrong because, as explained above, a trespasser may not recover his annexed item. D is wrong as Jude is entitled to no monetary value for the house, since it added no value to the land and Julia is also free to keep the house. Jude cannot place a lien against Eatmore as nothing is owed him.

2007-2008 Law Decks

Anda mungkin juga menyukai